Download as pdf or txt
Download as pdf or txt
You are on page 1of 315

https://krokology.

com/
1. A patient has gradually lost consciousness. The skin is pale Objective
and dry. There is a smell of ammonia from the mouth. ↑ Urea + lost consciousness = Uremic coma
Respirations are deep and noisy. Heart sounds are muffled,
pericardial friction rub is present. Blood pressure is 180/130
mm Hg. Blood test: Нb- 80 g/L, leukocytes 12 · 109/L, blood
glucose - 6.4 mmol/L, urea 50 mmol/L, creatinine - 1200 Uremia—syndrome resulting from high
mcmol/L, blood osmolarity - 350 mOsmol/L. No urinary serum urea.
excretion. Make the diagnosis:
A. Uremic coma Can present with:
B. Hyperglycemic coma 1. Pericarditis
C. Acute renal failure 2. Encephalopathy (unconsciousness)
D. Acute disturbance of cerebral circulation 3. Anorexia
4. Nausea
E. Hyperosmolar coma
(Ure- PEAN $)
2. A man works in casting of nonferrous metals and alloys for
12 years. In the air of working area there was registered high Objective
content of heavy metals, carbon monoxide, and nitrogen. Highlighted symptoms are typical for lead
During periodic health examination the patient presents with poisoning (Saturnism).
asthenovegetative syndrome, sharp pains in the stomach,
constipations, pain in the hepatic area. In urine: aminolevulinic
acid and coproporphyrin are detected. In blood: reticulocytosis,
low hemoglobin level. Such intoxication is caused by:
A. Lead and lead salts
B. Tin
C. Carbon monoxide
D. Nitric oxide
E. Zinc
Review
A 39-year-old man, a battery attendant, suddenly developed
weakness, loss of appetite, nonlocalized colicky abdominal
pains, and nausea. Objectively his skin is gray; there is a pink-
gray stripe on his gums; the stomach is soft and sharply painful.
Blood test detected erythrocytes with basophilic stippling and
anemia. The patient has a history of peptic ulcer disease of the
stomach. Constipation tends to occur every 3-4 days. What is
the most likely provisional diagnosis?
A. Saturnism (lead poisosning)
B. Acute appendicitis
C. Perforation of gastric ulcer
D. Acute cholecystitis
E. Chronic alcoholism
3. A 23-year-old woman without visible cause developed a
Objective
conflicting behavior at the work place. She accused the
Over excitation → manic episodes.
management of underestimating her, claimed that she can
be a deputy director, because she speaks four languages, is
very attractive, and can make useful connections for the
company. She has been dressing extravagantly, flirting with
her colleges, and singing loudly in her office. Infact, she
has only the training of a computer operator and speaks no
foreign languages. What is the likely clinical diagnosis?
A. Mild mental retardation
B. Depressive disorder
C. Manic episode
D. Epilepsy
E. Schizophrenia
A 26-year-old patient with affective bipolar disorder has Objective
developed a condition manifested by mood improvement, • Presentation →
behavioral and sexual hyperactivity, verbosity, active body • Manic episodes →
language, reduced need for sleep. Which of the following drugs • Neuroleptic drug (Aminazine).
would be most effective in this case?
A. Neuroleptics with sedative effect
B. Antidepressants with activating effect
C. Neuroleptics with activating effect
D. Tranquilizers
E. Antidepressants with sedative effect
A 23-year-old woman has been suffering from a mental disease
since the age of 18, the course of disease has no remission
periods. At a hospital the patient mostly presents with non-
purposeful foolish excitation: she makes stereotypic grimaces,
exposes herself, publicly masturbates with a loud laughter, Objective
repeates stereotypical abusive shouts. The patient should be Presentation →
prescribed: Schizophrenia / manic episodes→
A. Neuroleptics Neuroleptic drug (Aminazine)
B. Antidepressants
C. Tranquilizers
D. Nootropics
E. Mood stabilizers
4. A 17-year-old girl has been suffering from hepatic
Objective
cirrhosis for 3 years. Lately her periods of excitation have
• Liver failure:
been intermittent with depression, she does not sleep
• ↑ ammonia → Neurological symptoms
enough. Objectively her condition is severe, the girl is
(tremers, mode changes) →
sluggish, gives one-word responses, has tremor in her
• Hepatic encephalopathy.
extremities, her skin is icteric, with single hemorrhagic
• Kidney failure:
rashes. Name the likely complication of her disease:
• ↑ Urea → loss consciousness →
A. Bipolar affective disorder
• Uremic coma
B. Hepatic encephalopathy
C. Reye syndrome
D. Sepsis
E. Kidney failure
A woman undergoing in-patient treatment for viral
Objective
hepatitis type B developed headache, nausea, recurrent
Liver failure
vomiting, memory lapses, flapping tremor of her hands, The patient is infected with viral hepatitis which is
and rapid pulse. Sweet smell from her mouth is is a typical cause of acute liver failure.
detected. Body temperature is 37.6oC, heart rate is
89/min. What complication developed in the patient?
A. Acute liver failure Acute liver failure
• Acute liver failure is a serious and potentially life-
B. Ischemic stroke
threatening condition that occurs when the liver suddenly
C. Gastrointestinal hemorrhage
loses its ability to function properly.
D. Hypoglycemic shock
• When the liver fails, it can detoxify blood from ammonia
E. Meningoencephalitis toxins.
• Ammonia toxins will accumulate in the brain leading to
these symptoms.
• Headache, nausea, recurrent vomiting, memory lapses,
flapping tremor of her hands.
• Fetor hepaticus: sweet, musty, or moldy odor caused by
accumulation of ammonia in blood.
• Causes:
• Viral hepatitis.
• Toxins: ex, mushroom toxins
• Medication
• Ischemic liver injury.
Acute liver failure
• Acute liver failure is a serious and potentially life-
2 hours after eating unknown mushrooms, a 28-year-old threatening condition that occurs when the liver suddenly
man sensed a decrease in his mobility and deterioration of loses its ability to function properly.
his ability to focus. This condition was then followed by a • When the liver fails, it can detoxify blood from ammonia
state of agitation and agression. On examiantion he is toxins.
• Ammonia toxins will accumulate in the brain leading to
disoriented and his speech is illegible. 4 hours later he
these symptoms.
developed fetor hepaticus and lost his consciousness. What
• Headache, nausea, recurrent vomiting, memory lapses,
syndrome can be observed in this patient?
flapping tremor of her hands.
A. Acute hepatic failure • Fetor hepaticus: sweet, musty, or moldy odor caused by
B. Hepatolienal syndrome accumulation of ammonia in blood.
C. Portal hypertension • Causes:
D. Cholestatic syndrome • Viral hepatitis.
E. Cytolytic syndrome • Toxins: ex, mushroom toxins
• Medication
• Ischemic liver injury.
5. A 35-year-old woman came to the family doctor with Abnormal uterine bleeding
complaints of profuse menstruations that last up to 10 days. (Dysfunctional uterine bleeding)
Gynecological examinations shows that the uterine cervix
is without changes, the uterus is in anterflexio, has normal • What is it? Abnormal bleeding from the uterus
size, is mobile and painless. The uterine appendages on the that is not related to a specific menstrual period
both sides are without peculiarities. The family doctor
or pregnancy.
made the provisional diagnosis of the abnormal uterine • Causes: hormonal imbalances, structural
bleeding. What instrumental method of examination needs abnormalities, medications, or medical
to be performed first to diagnose this pathology? conditions such as polyps or fibroids.
A. Laparoscopy
• Diagnosis: Transvaginal ultrasound
B. Transvaginal ultrasound
C. Culdoscopy
D. Transabdominal ultrasound
E. Colposcopy
Abnormal uterine bleeding
(Dysfunctional uterine bleeding)

• What is it? Abnormal bleeding from the uterus


that is not related to a specific menstrual period
or pregnancy.
• Causes: hormonal imbalances, structural
abnormalities, medications, or medical
conditions such as polyps or fibroids.
• Diagnosis: Transvaginal ultrasound
Review Abnormal uterine bleeding
A 46-year-old woman came to the maternity clinic with (Dysfunctional uterine bleeding)
complaints of moderate blood discharge from the vagina, which
developed after the menstruation delay of 1.5 months. On • What is it? Abnormal bleeding from the uterus
vaginal examination: the cervix is clean; the uterus is not that is not related to a specific menstrual period
enlarged, mobile, painless; appendages without changes. Make or pregnancy.
the diagnosis: • Causes: hormonal imbalances, structural
A. Dysfunctional uterine bleeding abnormalities, medications, or medical
conditions such as polyps or fibroids.
B. Adenomyosis
• Diagnosis: Transvaginal ultrasound
C. Ectopic pregnancy
D. Submucous uterine myoma
E. Cancer of the uterine body
A 14-year-old girl has been delivered to a gynecological Abnormal uterine bleeding
department with complaints of profuse blood discharge from her (Dysfunctional uterine bleeding)
genital tract for 2 weeks. Anamnesis: menstruation since 13,
irregular, painful, profuse; the last one was 2 months ago. • What is it? Abnormal bleeding from the uterus
Objectively: pale skin and mucosa, BP- 100/60 mm Hg, Hb- that is not related to a specific menstrual period
108 g/l. The abdomen is soft and painless on palpation. Rectal or pregnancy.
examination revealed no pathologies of reproductive organs. • Causes: hormonal imbalances, structural
What condition is it? abnormalities, medications, or medical
A. Juvenile uterine hemorrhage (Dysfunctional) conditions such as polyps or fibroids.
B. Hypomenstrual syndrome • Diagnosis: Transvaginal ultrasound
C. Inflammation of uterine appendages (Pelvic inflammatory
disease)
D. Pelviperitonitis
E. Endometritis
6. A child was born at 40 weeks of gestation with the Physiological weight loss
weight of 3700g. The child’s Apgar score is 7/9. The baby • It’s normal for newborn to lose up to 7% of the
was put to breast immediately after birth and suckled birth weight within 10-14 days after birth
actively. On the 3rd day of life, the child’s weight • This weight loss is due to the loss of excess fluid
decreased to 3600g. What transitory condition is observed • In this case the child lost less than 7% of their
in this child? birth weight, it is within the normal range of
A. Uric acid infarction physiological weight loss.
B. Transient dysbiosis
C. Physiological jaundice
D. Physiological weight loss
E. Toxic erythema
7. A 19-year-old young man complains of cough with Objective
expectoration of purulent sputum in the amount of 100 mL per • Purulent cough and dilatation are typical for
day, hemoptysis, dyspnea, increased body temperature up to chronic bronchitis.
37.8oC, general weakness, weight loss. The patient’s condition • It’s one of the four COPD diseases:
lasts for 4 years. Exacerbations occur 2-3 times a year. The Emphysema, chronic bronchitis, asthma,
patient presents with malnutrition, pale skin, cyanosis of the Bronchiectasis
lips, drumstick (clubbed) fingers. Tympanic percussion sound in
the lungs, weakened respiration, numerous various moist
crackles in the lower pulmonary segments on the left can be
observed. In blood: erythrocytes - 3.2·1012/L, leukocytes -
8.4·109/L, ESR- 56 mm/hour. On X-ray: lung fields are
emphysematous, the left pulmonary root is deformed and
dilated. What is the most likely diagnosis?
A. Multiple bronchiectasis of the left lung
B. Chronic left-sided pneumonia
C. Chronic abscess of the left lung
D. Left-sided pulmonary cystic dysplasia
E. Suppuration of the cyst in the left lung
8. A 35-year-old forestry officer was delivered to the
hospital on the 7th day after the onset of the disease.
He complains of chills, elevated body temperature
up to 40.0°C, sharp headache, and myalgias. On
examination his face is puffy and hyperemic, the
tongue is dry, “chalk-dusted”. In the left inguinal
area, a sharply painful conglomeration of enlarged
lymph nodes can be palpated. The skin over the
conglomeration is hyperemic and tense. What
etiotropic therapy should be prescribed to this
patient?
A. Human immunoglobulin intravenously
B. Ribavirin
C. Streptomycin
D. Administration of heterologous serum
E. Ketoconazole

Objective
Presentation (painful ulcer and lymphodes) →
Francisella tularensis →
Streptomycin
9. A woman complains of a severe pain in her throat on the Peritonsillar abscess
left, difficult swallowing and mouth opening, elevated body • What is it? collection of pus surrounding
temperature, and general malaise. The onset of the disease tonsils. It is a common complication of
was 4 days ago after a case of tonsillitis. Examination tonsillitis and usually occurs as a result of a
detects a trismus of the masticatory muscles; the left tonsil bacterial infection.
is displaced toward the midline; the anterior palatal arch is • Findings:
infiltrated and protruding. The regional lymph nodes on the • Severe inflammation and edema.
right are enlarged and painful on palpation. Make the • Very painful.
diagnosis: • Difficulty swallowing and speaking
A. Lacunar tonsillitis • Trismus (difficulty in opening the mouth).
• Excessive salivation.
B. Infectious mononucleosis • Otalgia (ear pain).
C. Tonsillar tumor • Head tilts.
• Enlargement of lymph nodes.
D. Peritonsillar abscess
E. Acute pharyngitis
Peritonsillar abscess
• What is it? collection of pus surrounding
tonsils. It is a common complication of
tonsillitis and usually occurs as a result of a
bacterial infection.
A man complains of sore throat on the left, pain in his left ear, • Findings:
to up to 39oC, and nasal sound of his voice. Disease onset was 5 • Severe inflammation and edema.
days ago. Marked trismus and increased salivation are • Very painful.
observed. The head tilts to the left shoulder. Left side of the • Difficulty swallowing and speaking
soft palate presents with swelling, hyperemia, and infiltration. • Trismus (difficulty in opening the mouth).
Retromandibular lymph nodes on the left are acutely painful on • Excessive salivation.
palpation. Otoscopy results are normal. Make the diagnosis: • Otalgia (ear pain).
A. Left-sided peritonsillar abscess • Head tilts.
B. Retropharyngeal abscess • Enlargement of lymph nodes.
C. Parapharyngeal phlegmon
D. Peritonsillitis on the left
E. Cervical phlegmon on the left
A 58-year-old woman undergoing chemotherapy for her
oncologic disorder has developed sore throat. Examination
revealed necrotic areas on the mucosa of the pharynx and
tonsils. Many of her teeth are afflicted with caries. In blood:
Objective
neutrophilic granulocytes are practically absent against the
• Chemotherapy →
background of leukopenia. Leukocytes are represented mainly
• Leukopenia (agranulocytes) →
by lymphocytes and monocytes. What disease can be suspected
• Agranulocitar tonsillitis: a bacterial infection
in the given case?
that can occur when there is a decrease in
A. Agranulocitar tonsillitis
neutrophils
B. Lacunar tonsillitis
C. Pseudomembranous (Vincent’s) tonsillitis
D. Syphilitic tonsillitis
E. Diphtheria
Objective
Lacunar tonsillitis and follicular tonsillitis are two types of
tonsillitis, which is an infection or inflammation of the
tonsils.
A 16-year-old patient has made an appointment with an • Lacunar tonsillitis occurs when the small pits or crypts in
the tonsils, called lacunae, become inflamed and filled
otolaryngologist. He complains of elevated body temperature with pus.
and sore throat. Disease onset was 2 days ago, after the patient • Follicular tonsillitis: occurs when the infection affects the
ate two portions of ice-cream. Pharyngoscopy shows hyperemic tonsil's follicles, which are small pockets in the tonsils
mucosa of the palatine tonsils, with purulent exudate in the that produce white blood cells.
lacunae. Make the provisional diagnosis:
A. Lacunar tonsillitis
B. Follicular tonsillitis
C. Diphtheria
D. Acute pharyngitis
E. Pseudomembranous (Vincent’s) tonsillitis
An 18-year-old patient was admitted to a hospital with
complaints of headache, weakness, high fever, sore throat.
Objectively: enlargement of all groups of lymph nodes was Infectious mononucleosis
revealed. The liver is enlarged by 3 cm, spleen - by 1 cm. In Fever and sore throat + enlargement of:
blood: leukocytosis, atypical lymphocytes - 15%. What is the • Lymph nodes.
most probable diagnosis? • Spleen
A. Infectious mononucleosis • Liver
B. Acute lymphoid leukosis
C. Diphtheria
D. Angina
E. Adenoviral infection
Three weeks after a case of acute tonsillitis the patient is still
weak, inert, subfebrile, his retromaxillary lymph nodes are
enlarged. Tonsils are flabby, stick together with arches, there Objective
are purulent plugs in lacunae. What is the most probable 3 weeks, purulent discharge from lacunae →
diagnosis? Chronic tonsillitis
A. Chronic tonsillitis
B. Chronic pharyngitis
C. Acute lacunar tonsillitis
D. Paratonsillitis
E. Tonsillar tumour
10. A 30-year-old woman complains of itching skin, Arthropods (‫)ﺣﺷرات‬
predominantly in the evening and at night. The condition
lasts for 2 weeks already. On the skin of interdigital folds, Scorpians:
mammary glands, abdomen, buttocks, and thighs there are • 4 pairs of ambulatory legs
numerous fine papular and papulovesicular rashes located • Segmented abdomen ‫ﺑطن ﻣﻘﺳﻣﺔ‬
in pairs, excoriations. There is no rash on the face and
neck. Similar rash is observed in the husband of the patient. Karakurt spider / Steppe spider/ black widow:
What is the most likely diagnosis? • Roundish black abdomen. ‫ﺑطن ﺳوداء‬
A. Eczema • Two rows of red spots on its dorsal side.
B. Scabies • Four pairs of jointed limbs.
C. Epidermophytosis Phthiriasis:
D. Neurodermatitis • Feeds on the blood.
E. Herpes
• Location: hair near armpits armpit, beard, eyebrows.
Dog-louse:
• Encephalitis.
• Body louse (Pediculus humanus humanus):
• Features: ~ 3 mm and white
Scabies:
• Skin itching.
• Between fingers, and genital areas.
Wohlfahrtia magnifica:
• Infective form: Larvae.
• Disease: Myiasis
• Feature: tissue necrosis.
The dermatologist has an appointment with a 30-year- Arthropods (‫)ﺣﺷرات‬
old man that complains of severely itching rashes that
Scorpians:
especially disturb him at night. The rashes developed 2 • 4 pairs of ambulatory legs
weeks ago, after he had returned from a travel. • Segmented abdomen ‫ﺑطن ﻣﻘﺳﻣﺔ‬
Objectively on the lateral surfaces of his fingers, hands,
wrists, elbows, lower abdomen, genitals, and thighs Karakurt spider / Steppe spider/ black widow:
there are paired papulovesicles, single pustules, and • Roundish black abdomen. ‫ﺑطن ﺳوداء‬
scratch marks. What disease can be suspected? • Two rows of red spots on its dorsal side.
A. Scabies • Four pairs of jointed limbs.
B. Pyoderma Phthiriasis:
C. Dermatitis • Feeds on the blood.
D. Eczema • Location: hair near armpits armpit, beard, eyebrows.
E. Shingles Dog-louse:
• Encephalitis.
• Body louse (Pediculus humanus humanus):
• Features: ~ 3 mm and white
Scabies:
• Skin itching.
• Between fingers, and genital areas.
Wohlfahrtia magnifica:
• Infective form: Larvae.
• Disease: Myiasis
• Feature: tissue necrosis.
11. A 48-year-old man came to a doctor with complaints of
vomiting that brings no relief and a burning pain in his left Chronic pancreatitis
subcostal region that irradiates to the left lumbar region. • What is it? long-term inflammation of the
These signs appeared after a meal. The Ortner’s and Mayo- pancreas that results in permanent damage to
Robson’s signs are positive. In the blood; leukocytosis and the organ and loss of function.
increased ESR. In the urine; elevated diastase levels. Make • Findings:
the diagnosis: • Pain location → in the epigastric region,
A. Chronic cholecystitis in the exacerbation stage
left subcostal area.
B. Chronic pancreatitis in the exacerbation stage • Abdominal distension.
C. Gastric ulcer Performation
• Diarrhea, and weight loss.
D. Chronic gastritis in the exacerbation stage • Complication:
E. Renal colic • Diabetes due to damage to the insulin-
producing cells in the pancreas.
• Cholelithiasis.
• Pancreatic insufficiency.
Objective • Treatment: Enzyme replacement (Pancreatin)
• Left subcostal region → pancreatitis • Association: chronic gastritis → white coating
• Ortner’s and Mayo-Robson’s: is a tenderness in the near the root of the tongue is a sign of chronic
left upper quadrant of the abdomen → pancreatitis gastritis which is often associated with chronic
pancreatitis.
Chronic pancreatitis
• What is it? long-term inflammation of the
pancreas that results in permanent damage to
A 45-year-old patient complains of pain in the epigastric region, the organ and loss of function.
left subcostal area, abdominal distension, diarrhea, loss of • Findings:
weight. He has been suffering from this condition for 5 years. • Pain location → in the epigastric region,
Objectively: the tongue is moist with white coating near the left subcostal area.
root; deep palpation of abdomen reveals slight pain in the • Abdominal distension.
epigastric region and Мауо-Robson’s point. Liver is painless • Diarrhea, and weight loss.
and protrudes by 1 cm from the costal arch. Spleen cannot be • Complication:
palpated. What disease can be primarily suspected? • Diabetes due to damage to the insulin-
A. Chronic pancreatitis producing cells in the pancreas.
B. Atrophic gastritis • Cholelithiasis.
C. Peptic stomach ulcer • Pancreatic insufficiency.
D. Chronic cholecystitis • Treatment: Enzyme replacement (Pancreatin)
E. Chronic enteritis • Association: chronic gastritis → white coating
near the root of the tongue is a sign of chronic
gastritis which is often associated with chronic
pancreatitis.
Chronic pancreatitis
• What is it? long-term inflammation of the
pancreas that results in permanent damage to
the organ and loss of function.
A 48-year-old man complains of constant pain in the upper
• Findings:
abdomen, predominantly on the left, which aggravates after
• Pain location → in the epigastric region,
eating, diarrhea, loss of weight. The patient has alcohol use
left subcostal area.
disorder. Two years ago he had a case of acute pancreatitis.
• Abdominal distension.
Blood amylase is 4 g/hour·l. Feces analysis: steatorrhea,
• Diarrhea, and weight loss.
creatorrhea. Blood sugar is 6,0 mmol/l. What treatment should
• Complication:
be prescribed?
• Diabetes due to damage to the insulin-
A. Panzinorm forte (Pancreatin)
producing cells in the pancreas.
B. Insulin
• Cholelithiasis.
C. Gastrozepin (Pirenzepine) • Pancreatic insufficiency.
D. Contrykal (Aprotinin) • Treatment: Enzyme replacement (Pancreatin)
E. No-Spa (Drotaverine) • Association: chronic gastritis → white coating
near the root of the tongue is a sign of chronic
gastritis which is often associated with chronic
pancreatitis.
Chronic pancreatitis
• What is it? long-term inflammation of the
A 45-year-old patient complains of pain in the epigastric region, pancreas that results in permanent damage to
the organ and loss of function.
left subcostal area, abdominal distension, diarrhea, loss of
• Findings:
weight. He has been suffering from this condition for 5 years.
• Pain location → in the epigastric region,
Objectively: the tongue is moist with white coating near the
left subcostal area.
root; deep palpation of abdomen reveals slight pain in the
• Abdominal distension.
epigastric region and Мауо-Robson’s point. Liver is painless
• Diarrhea, and weight loss.
and protrudes by 1 cm from the costal arch. Spleen cannot be
• Complication:
palpated. What disease can be primarily suspected?
• Diabetes due to damage to the insulin-
A. Chronic pancreatitis
producing cells in the pancreas.
B. Atrophic gastritis
• Cholelithiasis.
C. Peptic stomach ulcer • Pancreatic insufficiency.
D. Chronic cholecystitis • Treatment: Enzyme replacement (Pancreatin)
E. Chronic enteritis • Association: chronic gastritis → white coating
near the root of the tongue is a sign of chronic
gastritis which is often associated with chronic
pancreatitis.
Objective
Right subcostal area and history of acute
pancreatitis → chronic pancreatitis (memorize)

A 60-year-old woman complains of unbearable pain in her right


subcostal area. She has a history of acute pancreatitis.
Temperature is 38.2oC. Objectively her sclera are icteric. There
are no signs of peritoneal irritation. Ortner’s and Gubergrits’
symptoms are positive. Urine diastase is 320 U/L. What is the
most likely diagnosis?
A. Exacerbation of chronic pancreatitis
B. Acute cholangitis
C. Chronic cholecystitis
D. Acute cholecystitis
E. Pancreatic cancer
A 48-year-old woman has been suffering from chronic Objective
pancreatitis for the last 7 years. Lately she has been noticing an Chronic pancreatitis → Pancreatic insufficiency →
increase in daily feces with foul smell, abdominal distention, malabsorption
gurgling. The patient complains of diarrhea, weakness,
fatigability, loss of appetite, loss of weight. What syndrome can
be suspected in this case?
A. Malabsorption
B. Irritable colon
C. Maldigestion
D. Exudative enteropathy
E. Endocrine gland failure
12. A 25-year-old man was hospitalized with complaints of pain Acute appendicitis
in his lower abdomen and right lumbar area that appeared one • Acute inflammation of the appendix (blue
hour ago. Patient’s general state is moderately severe. Body arrow in ), can be due to obstruction by fecalith
temperature - 38.2oC, heart rate - 102/min. The tongue is dry. (in adults) or lymphoid hyperplasia (in
The abdomen is painful on deep palpation in the right iliac area children).
• Location: RLQ.
and in the Petit triangle. Aure-Rozanov and Gabay signs are
• Findings: Nausea, fever; may perforate →
positive. Make the provisional diagnosis:
peritonitis, rebound tenderness on exam,
A. Acute appendicitis
positive Aure-Rozanov sign.
B. Right-sided renal colic
C. Cecal tumor
D. Intestinal obstruction
E. Acute cholecystitis
13. A 45-year-old woman was hospitalized with Objective
complaints of periodical severe headaches against the Hypertension, hypokalemia, and hypernatremia →
background of elevated blood pressure up to180/90 mmHg, primary hyperaldosteronsim Ialdosteroma or
muscle weakness, and frequent urination (at night as well). secondary hyperaldosteronism.
Her anamnesis states that despite combining various
antihypertensive drugs and adjusting the dosage her arterial
hypertension cannot be corrected with drugs. The patient’s
blood serum potassium levels are 2.0mmmol/L, blood
serum sodium level are160mmmol/L. Ultrasound imaging
detects three dimensional formations approximately 1.0 cm
in diameter in the both adrenal glands. Selective
endovascular blood sampling from the suprarenal vein was
performed which revealed significant increase of cortisol
and aldosterone levels. Make the diagnosis:
A. Pheochromocytoma
B. Cushing's disease
C. Androsteroma
D. Aldosteroma
E. Cushing's syndrome
14. During winter epidemics of influenza caused
predominantly by virus A/California/04/2009 (H1N1), Objective
• Presentation →
on the 2nd day after the disease onset a 30-year old
hospitalized man presented with high fever, dry • Influenza (H1N1 strain) →
cough, myalgia, headache, and general weakness. • Neuraminidase inhibitors (oseltamivir)
What should be prescribed as an etiotropic treatment
in this case?
A. Immunoglobulin
B. Antibiotics
C. Neuraminidase inhibitors (oseltamivir)
D. Acyclovir
E. Interferon inducers
15. A 75-year-old man in a severe condition suffers from Objective
dyspnea at rest, marked weakness, and arrhythmia. 70/40mmHg and Abdominal aortic pulsation →
Abdominal aortic pulsation is observed, further on is a dissecting aortic aneurysm.
systolic murmur detected. Palpation reveals a volumetric
formation in the mesogastrium. Blood pressure is
70/40mmHg. There is no pulsation over the femoral
arteries. Oliguria is detected. Which diagnosis is the correct
one?
A. Pancreatic cyst
B. Acute pericarditis
C. Cardio sclerotic aneurysm
D. Dissecting aortic aneurysm
E. Acute cardiac aneurysm
A 65-year-old woman on abdominal palpation presents with a
tumor in the umbilical region and above it; the tumor is 13x8 Objective
cm in size, moderately painful, nonmobile, pulsing. On Aneurysm: is an abnormal bulging or ballooning
auscultation systolic murmur can be observed. What is the most of a weakened section of an artery. Rupture of
likely diagnosis? aneurysm is one of the most serious
A. Abdominal aortic aneurysm complications.
B. Gastric tumor
C. Arteriovenous aneurysm
D. Tricuspid insufficiency
E. Bicuspid insufficienc

A 65-year-old woman on abdominal palpation presents with a


tumor in the umbilical region and above it; the tumor is 13x8
cm in size, moderately painful, non-mobile, pulsing. On
auscultation systolic murmur can be observed. What is the most
likely diagnosis?
A. Abdominal aneurysm
B. Gastric tumor
C. Arteriovenous aneurysm
D. Tricuspid insufficiency
E. Bicuspid insufficiency
A week ago a 65-year-old patient suffered an acute myocardial
infarction, his general condition deteriorated: he complains of
dyspnea at rest, pronounced weakness. Objectively: edema of
the lower extremities, ascites is present. Heart borders are
extended, paradoxical pulse is observed 2 cm from the apex
beat to the left. What is the most likely diagnosis?
A. Acute cardiac aneurysm
B. Recurrent myocardial infarction
C. Acute pericarditis
D. Cardiosclerotic aneurysm
E. Pulmonary embolism

Objective
Myocardial infarction → highlighted findings →
cardiac aneurysm (ventricular aneurysm)
16. A 56-year-old woman was diagnosed with stage 2
Objective
hypertension of the 2nd degree. She belongs to the group of
Nonselective β blocker can help treating
moderate risk and has bronchial asthma. What group of drugs is hypertension by decreasing heart rate, however, it
CONTRAINDICATED to this patient? can lead to blocking of β receptors in the lung
A. β-blockers increasing asthma symptoms.
B. Angiotensin-converting enzyme inhibitors
C. Diuretics
D. Calcium antagonists
E. Imidazoline receptor antagonists
17. A dweller of the northern Dnieper area, a fisherman, for
the last several days has been complaining of a discomfort Opisthrochiasis
in his right subcostal region, periodical episodes of
• Feature: smallest egg.
diahhrea , intermittent with constipation, frequent skin
rashes. Abdominal ultrasound shows enlarged liver and • Transmission: common in population who
pancreatic head. Make the provisional diagnosis: eats freshwater raw fish.
A. Onchocerciasis
• Symptoms: usually asymptomatic but can
B. Ornithosis
lead to intestinal symptoms like diarrhea.
C. Trichinellosis
D. Taeniasis
E. Opisthrochiasis
Objective
Caviar and anemia → Diphyllobothriasis

A 28-year-old woman complains of nausea, stomachache, pain


in her tongue, and liquid feces. Three days ago she ate poorly
salted pike caviar. Objectively her skin is pale, the tongue looks
“lacquered” (bald tongue). Pulse is 100/min., with muffled
heart sounds and systolic murmur over the cardiac apex. Blood
pressure is 95/50 mm Hg. The liver is enlarged by 3 cm.
Hemogram shows anemia, eosinophils - 18%. Oval helminth
eggs were detected in feces. Make the provisional diagnosis:
A. Diphyllobothriasis
B. Trichinosis
C. Teniasis
D. Taeniarhynchosis
E. Ascaridiasis
18. A 9-year-old boy fell off a tree and hit the back of his Objective
head. A brief loss of consciousness was observed. The Fractures → Surgical department.
child’s condition is satisfactory, he has a headache and
vertigo. Skull X-ray scans shows depressed fracture of the
occipital bone in the area of the external occipital
protuberance. What treatment tactics is indicated for this
patient?
A. Hemostatic therapy
B. Surgical intervention
C. Anti-inflammatory therapy
D. Complex conservative treatment
E. Lumbar puncture to relieve the pressure
A 24-year-old patient had been delivered to a thoracic
department with a chest injury, fracture of the IV, V, VI ribs on
the right. Plan radiography showed the fluid level in the pleural
cavity to be reaching the III rib on the right. Puncture contained
blood clots. What is the optimal treatment tactics?
A. Emergency thoracotomy
B. Pleural puncture
C. Thoracentesis and thoracostomy
D. Hemostatic therapy
E. Medical thoracoscopy
A 9-year-old boy has been suffering from multiple
bronchiectasis since he was 3 years old. Exacerbations occur
frequently (34 times a year), after conservative therapy there are
short remission periods. The disease progresses, the child is
physically underdeveloped, presents with pale skin,
acrocyanosis, deformed nail plates in the shape of ”clock-face”.
Bronchography reveals saccular bronchiectases in the lower Objective
Failure of conservative treatment → Surgical
lobe of the right lung. What further treatment tactics should be
chosen? treatment.
A. Surgical intervention
B. Continuation of conservative therapy
C. Physiotherapy
D. Sanatorium-and-spa treatment
E. Physical training
19. A 34-year-old woman after rapidly changing her Forensic terms
position from horizontal to vertical suddenly paled, fell • Syncope: sudden and short term fainting (passing out) for
down, her skin became moist, her limbs are cold, her seconds or minutes caused by brain hypoxemia. Less
pupils are dilated. The pulse is rapid and thready, blood sever than collapse.
pressure is 50/25 mmHg. What condition has likely • Reasons: cns influence on vasomotor centers due to
developed in the patient? pain, hot, sight of blood, fright, etc.
A. Morgagni-Adams-stokes syndrome • Findings: pale, coldness and pupils dilated
B. Ventricular fibrillation • Collapse: sudden drop in blood pressured due to low
C. Shock blood volume or tone.
D. Coma • Reasons: intensive pain, bleeding and intoxication.
E. Collapse • Findings: similar to schock. Intensive weakness can
be still conscious. Better than shock and worse than
syncome.
• Shock: extreme grave condition (between life and death)
Why collapse? due to drop in blood pressure.
• Sudden drop in blood pressure. • Coma: prolonged unconsciousness.
• Loss of consciousness. • Preagony: final stages of life
• Agony: final moments of life
• Clinical death: 3 = no pulse or breathing and no brain
activity (pupil is dilated and not responsive). Organs can
be donated.
• Brain (cellular/biological) death: irriversable loss of brain
function even if other organs are kept alive artificially.
Organs can’t be donated.
Forensic terms
• Syncope: sudden and short term fainting (passing out) for
seconds or minutes caused by brain hypoxemia. Less
sever than collapse.
• Reasons: cns influence on vasomotor centers due to
pain, hot, sight of blood, fright, etc.
• Findings: pale, coldness and pupils dilated
• Collapse: sudden drop in blood pressured due to low
blood volume or tone.
• Reasons: intensive pain, bleeding and intoxication.
• Findings: similar to schock. Intensive weakness can
be still conscious. Better than shock and worse than
syncome.
• Shock: extreme grave condition (between life and death)
due to drop in blood pressure.
• Coma: prolonged unconsciousness.
• Preagony: final stages of life
• Agony: final moments of life
• Clinical death: 3 = no pulse or breathing and no brain
activity (pupil is dilated and not responsive). Organs can
be donated.
• Brain (cellular/biological) death: irriversable loss of brain
function even if other organs are kept alive artificially.
Organs can’t be donated.
Forensic terms
A 14-year-old girl has fainted during a meeting. The day • Syncope: sudden and short term fainting (passing out) for
before she complained of a headache. The skin is pale, the seconds or minutes caused by brain hypoxemia. Less
limbs are cold, shallow breathing, heart sounds are sever than collapse.
muffled; heart rate is 51/min.; BP is 90/50 mm Hg. The • Reasons: cns influence on vasomotor centers due to
abdomen is soft. Meningeal symptoms are negative. Make pain, hot, sight of blood, fright, etc.
the provisional diagnosis: • Findings: pale, coldness and pupils dilated
A. Collapse • Collapse: sudden drop in blood pressured due to low
B. Unconsciousness blood volume or tone.
C. Acute left ventricular failure • Reasons: intensive pain, bleeding and intoxication.
D. Acute right ventricular failure • Findings: similar to schock. Intensive weakness can
E. Respiratory failure be still conscious. Better than shock and worse than
syncome.
• Shock: extreme grave condition (between life and death)
due to drop in blood pressure.
Why collapse? • Coma: prolonged unconsciousness.
• Sudden drop in blood pressure. • Preagony: final stages of life
• Fainting (loss of consciousness). • Agony: final moments of life
• Clinical death: 3 = no pulse or breathing and no brain
activity (pupil is dilated and not responsive). Organs can
be donated.
• Brain (cellular/biological) death: irriversable loss of brain
function even if other organs are kept alive artificially.
Organs can’t be donated.
Forensic terms
During an outdoors school event in hot weather, a 10- • Syncope: sudden and short term fainting (passing out) for
year-old girl lost her consciousness. Body temperature - seconds or minutes caused by brain hypoxemia. Less
36.7oC. Objectively her skin is pale and cold to touch, sever than collapse.
her pupils are dilated. Blood pressure - 90/50 mm Hg. • Reasons: cns influence on vasomotor centers due to
Heart rate - 58/min. What pathology occurred in this pain, hot, sight of blood, fright, etc.
• Findings: pale, coldness and pupils dilated
case?
• Collapse: sudden drop in blood pressured due to low
A. Syncope
blood volume or tone.
B. Sympathicotonic collapse
• Reasons: intensive pain, bleeding and intoxication.
C. Paralytic collapse • Findings: similar to schock. Intensive weakness can
D. Sunstroke be still conscious. Better than shock and worse than
E. - syncome.
• Shock: extreme grave condition (between life and death)
due to drop in blood pressure.
Why Syncope? • Coma: prolonged unconsciousness.
• Sudden drop in blood pressure. • Preagony: final stages of life
• Fainting (loss of consciousness) • Agony: final moments of life
• Why not sympathicotonic collapse? No signs • Clinical death: 3 = no pulse or breathing and no brain
for sympathetic dysfunction → ignore choice activity (pupil is dilated and not responsive). Organs can
B. be donated.
• Brain (cellular/biological) death: irriversable loss of brain
function even if other organs are kept alive artificially.
Organs can’t be donated.
Other terms
• Sympathicotonic collapse: sudden drop in blood
pressure and syncope related to overactivity of
sympathetic nervous system.
• Weakness or paralysis: paralysis or weakness
from injury.
• Sunstroke: prolonged exposure to high
temperature → high body temperature (above
103°F or 39.4°C) → symptoms: confusion,
dizziness, headache, nausea, vomiting, rapid
heart rate, and rapid breathing. In severe cases,
sunstroke can lead to seizures, coma, or even
death
Shock ‫ھﺑوط ﺣﺎد ﻓﻲ اﻟدورة اﻟدﻣوﯾﺔ‬
Sever drop in the blood circulation (hypotension) →
A 33 year old male patient was brought to Emergency No blood supply → Ischemia → Hypoxemia → cell death
Department with the signs of cardiovascular collapse: BP - 1. Cardiogenic:
60/30 mm Hg, Ps - 140 bpm, the skin is pale and moist, diuresis • The heart doesn’t pump the blood →
20 ml/h, Hb - 80 g/l, red blood cell count - 2, 5 · 1012/l. The • No cardiac output → Sever Hypotension.
reduction of blood volume averages: 1. Management:
A. 30-40% • Dobutamine
B. 10-15% • Promedol
C. 15-20% 2. Septic shock:
D. 20-25% • Bacteraemia → toxins → massive vasodilation →
E. 25-30% shock
• Fever, excessive sweating, euphoria
3. Hypovolemic shock:
• Loss of blood because of hemorrhage (> or = 30%)
Objective →
The average reduction of blood volume in • Sever drop in blood pressure.
hypovolemic shock ranges from 30-40%, which • Management:
is the correct answer. • Adrenaline.
• Sodium chloride solution.
4. Anaphylactic shock:
1. Type 1 hypersensitivity.
2. Histamine → Vasodilation → sever drop in blood
pressure. (very fast)
Forensic terms
• Syncope: sudden and short term fainting (passing out) for
seconds or minutes caused by brain hypoxemia. Less
sever than collapse.
• Reasons: cns influence on vasomotor centers due to
pain, hot, sight of blood, fright, etc.
A victim of a traffic accident was received by the intensive
• Findings: pale, coldness and pupils dilated
care unit. The patient is in a grave condition that can be
• Collapse: sudden drop in blood pressured due to low
characterized as a severe pathologic process that leads to
blood volume or tone.
exhaustion of vital functions and puts the patient into the
• Reasons: intensive pain, bleeding and intoxication.
marginal state between life and death due to critical • Findings: similar to schock. Intensive weakness can
reduction of capillary circulation in the affected organs.
be still conscious. Better than shock and worse than
The patient is in the state of:
syncome.
A. Shock
• Shock: extreme grave condition (between life and death)
B. Collapse
due to drop in blood pressure.
C. Coma • Coma: prolonged unconsciousness.
D. Agony • Preagony: final stages of life
E. Preagony • Agony: final moments of life
• Clinical death: 3 = no pulse or breathing and no brain
activity (pupil is dilated and not responsive). Organs can
be donated.
• Brain (cellular/biological) death: irriversable loss of brain
function even if other organs are kept alive artificially.
Organs can’t be donated.
Forensic terms
• Syncope: sudden and short term fainting (passing out) for
seconds or minutes caused by brain hypoxemia. Less
sever than collapse.
• Reasons: cns influence on vasomotor centers due to
pain, hot, sight of blood, fright, etc.
A woman in her early- to mid-thirties has lost her • Findings: pale, coldness and pupils dilated
consciousness 3-5 minutes ago. On examination: the skin is • Collapse: sudden drop in blood pressured due to low
pale, no pulse over the carotid arteries, no spontaneous blood volume or tone.
respiration, pupils are dilated; the patient is nonresponsive, • Reasons: intensive pain, bleeding and intoxication.
presents with atony. The patient’s condition can be • Findings: similar to schock. Intensive weakness can
determined as: be still conscious. Better than shock and worse than
A. Clinical death syncome.
B. Natural death • Shock: extreme grave condition (between life and death)
C. Syncope due to drop in blood pressure.
D. Brain death • Coma: prolonged unconsciousness.
E. Comatose state • Preagony: final stages of life
• Agony: final moments of life
• Clinical death: 3 = no pulse or breathing and no brain
activity (pupil is dilated and not responsive). Organs can
be donated.
• Brain (cellular/biological) death: irriversable loss of brain
function even if other organs are kept alive artificially.
Organs can’t be donated.
A highly injured person has gradually died. Please choose Forensic terms
the indicator of biological death: • Syncope: sudden and short term fainting (passing out) for
A. Autolysis and decay in the cells seconds or minutes caused by brain hypoxemia. Less
B. Disarray of chemical processes sever than collapse.
C. Loss of consciousness • Reasons: cns influence on vasomotor centers due to
D. Absence of palpitation and breathing pain, hot, sight of blood, fright, etc.
E. Absence of movements • Findings: pale, coldness and pupils dilated
• Collapse: sudden drop in blood pressured due to low
During a surgery, a patient with acute appendicitis developed a
cardiac arrest. What signs are characteristic of clinical death? blood volume or tone.
Rapid respiration, weak heart sounds • Reasons: intensive pain, bleeding and intoxication.
B. Apneustic respiration, no cardiac activity • Findings: similar to schock. Intensive weakness can
C. Kussmaul respiration, no cardiac activity be still conscious. Better than shock and worse than
D. No respiration, thready pulse syncome.
E. No respiration, no cardiac activity • Shock: extreme grave condition (between life and death)
due to drop in blood pressure.
The doctor stated the absence of respiration and cardiac • Coma: prolonged unconsciousness.
activity in a traffic accident victim. This condition lasts for 1 • Preagony: final stages of life
minute already. This clinical presentation minute already.
• Agony: final moments of life
This clinical presentation corresponds with the following
• Clinical death: 3 = no pulse or breathing and no brain
terminal state:
A. Clinical death activity (pupil is dilated and not responsive). Organs can
B. Traumatic shock, torpid phase be donated.
C. Traumatic shock, erectile phase • Brain (cellular/biological) death: irriversable loss of brain
D. Preagony function even if other organs are kept alive artificially.
E. Agony Organs can’t be donated.
A woman in her early- to mid-thirties has lost her consciousness
3-5 minutes ago. On examination: the skin is pale, no pulse over
the carotid arteries, no spontaneous respiration, pupils are Objective
• Apparent death → vital signs can’t be
dilated; the patient is nonresponsive, presents with atony. The
detectable but still there, and the person can
patient’s condition can be determined as:
wake up alone.
A. Apparent death (memrise *)
• Clinical death (natural) → vital signs stopped
B. Natural death
completely, and can be reversable by CBR and
C. Syncope
resuscitation.
D. Brain death
E. Coma
20. A 6-year-old girl complains of body temperature up to 1. Pneumonia → Dullness (weak breathing)
39 C, rhinitis, dry cough, dyspnea. She has been presenting 2. Tracheitis → Resonance, leukopenia and
with these signs for 5 days already. On examination her dry cough
condition is of moderate severity. Her dyspnea is of mixed 3. Bronchitis → Resonance and sputum
genesis. Respirations are 28/min. Percussion produces a
cough
dull sound in the right lower segments; in the same area
auscultation detects weakened respiration and fine 4. Obstructive → Wheezing
vesicular wet crackles; coarse respiration can be detected 5. Bronchiolitis → Resonance and children
on the left. Make the provisional diagnosis:
A. Acute simple bronchitis
B. Stenosing laryngotracheitis
C. Acute obstructive bronchitis
D. Acute bronchiolitis
E. Right-sided community acquired pneumonia
Pneumonia vs Tracheitis vs Bronchitis Bronchiolitis vs obstructive
1. Pneumonia → Dullness (weak breathing):
2. Tracheitis → Resonance, leukopenia and dry cough
3. Bronchitis → Resonance and sputum cough
4. Obstructive → Wheezing
5. Bronchiolitis → Resonance and children
A 3-month-old girl has rhinitis, dyspnea, dry cough. She has
Why Bronchiolitis?
been sick for 2 days. Objectively: pale skin, acrocyanosis,
1. Children > 2: Bronchiolitis affect children
hypopnoe; breathing rate is 80/min.; over the whole pulmonary
under age of 2. Adults: have larger airways
surface there is vesiculotympanitic (bandbox) resonance
than children, which makes them less
observed with numerous bubbling crackles. The most likely susceptible. Adults → bronchitis.
diagnosis is: 2. Bandbox resonance: No dullness → No
A. Acute bronchiolitis pneumonia.
B. Pneumonia
C. Mucoviscidosis
D. Foreign body in airways
E. Acute bronchitis
A 9-month-old child presents with fever, cough, dyspnea. The symptoms
appeared 5 days ago after a contact with a person suffering from URTI. 1. Pneumonia → Dullness (weak breathing):
Objectively: the child is in grave condition. Temperature of 38oC, cyanosis 2. Tracheitis → Resonance, leukopenia and
of nasolabial triangle is present. Respiration rate - 54/min., nasal flaring dry cough
during breathing. There was percussion dullness on the right below the
3. Bronchitis → Resonance and sputum
scapula angle, and tympanic sound over the rest of lungs. Auscultation
revealed bilateral fine moist crackles predominating on the right. What is cough
the most likely diagnosis? 4. Obstructive → Wheezing
A. Acute pneumonia 5. Bronchiolitis → Resonance and children
B. URTI
C. Acute laryngotracheitis
D. Acute bronchitis
E. Acute bronchiolitis

4 days ago a 32-year-old patient caught a cold: he presented with sore


throat, fatigue. The next morning he felt worse, developed dry cough, body
temperature rose up to 38, 2oC, there appeared muco-purulent
expectoration. Percussion revealed vesicular resonance over lungs,
vesicular breathing weakened below the angle of the right scapula, fine
sonorous and sibilant wheezes.What is the most likely diagnosis?
A. Focal right-sided pneumonia
B. Bronchial asthma
C. Acute bronchitis
D. Pulmonary carcinoma
E. Pulmonary gangrene
Related
For the last 3 days a 2-year-old girl has been presenting with
fever of 38.0oC, runny nose, dry cough, inertness, and appetite
loss. Percussion sound has a bandbox resonance. Auscultation Acute tracheitis case
detects puerile breathing without wheezing. In the blood there
are leukopenia, lymphocytosis, and accelerated ESR. Make the 1. Bandbox resonance → No dullness → No
right diagnosis: pneumonia.
A. Acute tracheitis 2. leukopenia
B. Acute obstructive bronchitis 3. Dry cough
C. Recurrent bronchitis, exacerbation phase 4. Puerile breathing
D. Acute bronchitis • Also:
E. Bilateral microfocal pneumonia • More common in children.
• Without wheezing → No obstruction
A 3 y.o. girl has had a temperature rise up to 380 , rhinitis, dry
superficial cough, flabbiness, appetite loss. Palpation didn’t
reveal any changes over her lungs. Percussion sound has a
wooden resonance, auscultation revealed puerile breathing, no
rales. In blood: leukopenia, lymphocytosis, increased ESR.
What is the most probable diagnosis?
A. Acute simple tracheitis
B. Acute obstructive bronchitis
C. Recurrent bronchitis, acute condition
D. Acute simple bronchitis
E. Bilateral microfocal pneumonia
A 3-year-old child has a cough and runny nose. Two other family
members have the same signs. On the third day after the onset of the
disease, the cough intensified and became dry and persistent. The
temperature increased to 37.8oC. The act of breathing involves the
auxiliary muscles. Percussion produces a banbox resonance bilaterally in
the lungs. Breathing is harsh, expiration is prolonged, there are moderate
and large bubbling wheezes. The wheezes are diffuse and mostly wet,
though in some places they are dry. Make the right diagnosis:
A. Acute obstructive bronchitis
B. Acute bronchiolitis
C. Bilateral bronchopneumonia
D. Stenosing laryngotracheitis
Why obstructive?
E. Bronchial asthma • Bandbox resonance → No dullness → No
pneumonia.
A 52 y.o. hard smoker patient complains of persistent cough with purulent • Wheezing → No obstruction
sputum discharge especially in mornings, dyspnea provoked even by
slight physical exertion, wheezing chest, tahypnoe, general weakness. He
considers himself to be ill during 12 years. The overwritten conditions
appear 3-4 times per year usually after common cold and have tendency to
progress.What disease do you think about first of all?
A. Chronic obstructive lung disease
B. Bronchial asthma
C. Mucoviscidosis (cystic fibrosis)
D. Bronchoectatic disease
E. Aspergillosis
21. A 37-year-old woman received an occupational trauma Objective
that resulted in a severe vision impairment. Now she needs Occupational diseases → Occupational rehabilitation
to be trained for another occupation. What type of
rehabilitation should be doctor choose for the patient in this
case?
A. Occupational rehabilitation
B. Target rehabilitation
C. Psychological rehabilitation
D. Social rehabilitation
E. Medical rehabilitation
22. A 45-year-old man has been suffering from ankylosing Amyloidosis
spondylitis for 15 years. For the last 3 years he has been • What is it? Accumulation of amyloid protein
noticing facial swelling and edemas of the limbs. (misfolded proteins) in different organs. It can
Objectively he assumes a “beggar’s” position. X-ray shows be systemic or localised.
“bamboo spine” changes in the thoracic and lumbar • Diagnosis: can be made by finding amyloid
segments. Heart ultrasound shows aortic regurgitation. depositions in the gingiva.
Complete blood count: Hb-106g/L; leukocytes- 8.9 x 109 • Findings:
/L; ESR-40mm/hour. Daily proteinuria-9.6 grams per 24 • Kidney: most commonly involved organ
hours. Blood creatinine-230 mcmol/L. What is the cause of → nephrotic syndrome (proteinuria) and
kidney failure in this case? hypertension
A. Concomitant heart disease • Heart → heart failure
B. Pyelonephritis • Liver → liver failure
C. Renal amyloidosis • Spine → ankylosing spondylitis
D. Urolithiasis
E. Medicine side effects
A 46-year-old man notes swollen legs, weakness, sensation of Amyloidosis
fullness and heaviness in the right subcostal area; it is the first • What is it? Accumulation of amyloid protein
occurrence of these signs in the patient. The patient has 20-year- (misfolded proteins) in different organs. It can
long history of rheumatoid arthritis. The liver and spleen are be systemic or localised.
enlarged and dense. Blood creatinine 0,23 mmol/l, proteinemia - • Diagnosis: can be made by finding amyloid
68 g/l, cholesterol - 4,2 mmol/l, urine specific gravity - 1012, depositions in the gingiva.
proteinuria - 3,3 g/l, isolated wax-like cylinders, leached • Findings:
erythrocytes in the vision field, leukocytes - 5-6 in the vision • Kidney: most commonly involved organ
field. What is the most likely complication? → nephrotic syndrome (proteinuria) and
A. Renal amyloidosis hypertension
B. Chronic glomerulonephritis • Heart → heart failure
C. Acute glomerulonephritis • Liver → liver failure
D. Heart failure
E. Chronic pyelonephritis
Objective
A 34-year-old man complains of pale edema of the face, feet, The involvement of systemic organs in this
shins, and lumbar area, elevated blood pressure up to 160/100 patient, particularly the kidneys, indicates that the
mm Hg, and general weakness. He has a clinical history of patient may be suffering from systemic
nonspecific ulcerative colitis. Objectively: pulse - 84/min., amyloidosis.
rhythmic, blood pressure - 165/100 mm Hg; edemas all over the Amyloidosis
body; the skin is pale and dry, with low turgor. The kidneys • What is it? Accumulation of
cannot be palpated, on an attempt to palpate them they are amyloid protein (misfolded
painless. Blood test: erythrocytes - 3.0·1012/L, Нb- 100 g/L, proteins) in different organs. It can
erythrocyte sedimentation rate - 50 mm/hour. Urinalysis: be systemic or localised.
proteins 3.5 g/L, erythrocytes - 7-10 in the vision field, • Diagnosis: can be made by finding
leukocytes - 5-6 in the vision field. Daily proteinuria - 6 grams. amyloid depositions in the gingiva.
What analysis should be conducted additionally to verify the • Findings:
diagnosis? • Kidney: most commonly
A. Gingival biopsy for the diagnosis of amyloid disease involved organ → nephrotic
B. Radioisotopic examination of kidneys syndrome (proteinuria) and
C. Urinalysis for Bence-Jones protein hypertension
• Heart → heart failure
D. Renal ultrasound
• Liver → liver failure
E. Survey and excretory urography
A 54-year-old patient has been suffering from femoral
osteomyelitis for over 20 years. In the last month she developed
gradually progressing leg edemas. In the urine: proteinuria – 6.6
g/L. In the blood: dysproteinemia in the form of
hypoalbuminemia, increased globulin levels, ESR – 50
mm/hour. Make the right diagnosis:
A. Secondary renal amyloidosis
B. Acute glomerulonephritis
C. Systemic lupus erythematosus
D. Chronic glomerulonephritis
E. Multiple myeloma
Related

During pathomorphological kidney investigation of a patient, who for a


long time had been suffering from osteomyelitis and died from A man, who for a long time has been suffering from
progressing renal failure, the following was revealed: deposits of chronic mandibular osteomyelitis, died of chronic kidney
homogeneous eosinophilic masses in glomerular mesangium, arterial disease. Autopsy revealed large lardaceous kidneys.
and arteriolar walls, and stroma, which became red when stained with What process had occurred in the kidneys?
Congo red. What pathological process is this? A. Renal amyloidosis
A. Amyloidosis B. Arterial nephrosclerosis
B. Mucoid swelling C. Glomerulonephritis
C. Calcinosis D. Necrotic nephrosis
D. Carbohydrate degeneration E. Contracted kidney
E. Hyalinosis
On autopsy of a 40-year-old woman, who had been
A 45-year-old patient with an 8- year history of tuberculosis died in a suffering from rheumatoid arthritis, her liver is found to
hospital of chronic renal failure. At autopsy, the kidneys were enlarged, be dense and enlarged. On dissection its tissue is red-
the cross-section surface looked greasy, histological study revealed brown colored, with enlarged follicles resembling semi
profuse deposits of structureless homogeneous eosinophilic masses transparent grayish white granules. What is the most
exhibiting marked metachromasia when stained with Congo red. What likely pathological process?
pathological process developed in the kidneys? A. Sago spleen
A. Secondary amyloidosis B. Sugar-coated spleen
B. Hematogenous renal tuberculosis C. Lardaceous spleen
C. Acute glomerulonephritis D. Splenic hyalinosis
D. Toxic nephritis against the E. Porphyry spleen
background of antibiotic therapy
Nephrosclerosis
23. A 72-year-old man with pneumonia complains of marked
Objective
dyspnea, chest pain, severe cough with expectoration, to is 39.5-
Shock ‫ھﺑوط ﺣﺎد ﻓﻲ اﻟدورة اﻟدﻣوﯾﺔ‬
40oC, no urination for a whole day. Objectively the patient is Blood pressure of systolic less than 90 ! is a sign
conscious. Respiratory rate is 36/min. Over the right lower of shock and requires immediate supportive
pulmonary lobe percussion sound is dull; on auscultation there treatment. In addition, he has anuria and marked
is bronchial respiration and numerous moist crackles. Blood dyspnea.
pressure is 80/60 mm Hg. Heart rate is 120/min. Heart sounds
are muffled, there is tachycardia. What tactics should the family
doctor choose in the management of this patient?
A. Hospitalization into the intensive care unit
B. Outpatient treatment
C. Treatment in the day patient facility
D. Hospitalization into the pulmonology unit
E. Hospitalization into the neurology unit
24. A 3-month-old child with signs of rickets presents with Sings of hypocalcaemia
positive Chvostek, Trousseau, and Maslov signs. One day ago, • You need calcium in blood to suppress neurons
the parents witnessed a cyanotic attack in their child the child and relax muscles.
broke into a cold sweat, the eyes bulged, and respiratory arrest • No PTH → no calcium → no nerve inhibition
occurred. One minute later the child drew in a loud breath and → seizure / convulsions.
the child’s condition normalized again. What is the cause of the • Deficiency: Seizure → convulsions ‫ ﺗﺷﻧﺟﺎت‬and
described signs of the disease? Muscle spasm, positive Chvostek and
Trousseau signs.
A. Decrease of blood calcium levels
B. Increase of blood calcium levels
C. Decrease of blood phosphorus levels
D. Increase of blood phosphorus levels
E. Metabolic acidosis
Objective
25. A 38-year-old patient has been brought by an ambulance to Dotted hemorrhages on the skin and
the surgical department with complaints of general weakness, thrombocytopenia are typical presentation of one
indisposition, black stool. On examination the patient is pale, of the platelets disorders like thrombocytopenic
there are dotted hemorrhages on the skin of his torso and purpura.
extremities. On digital investigation there are black feces on the
glove. Blood test: Hb- 108 g/L, thrombocytopenia. Anamnesis Bleeding disorders
states that a similar condition was observed 1 year ago. Make Types of bleeding disorders
the diagnosis: • Coagulation factors disorders:
A. Thrombocytopenic purpura • Deep bleeding: Joint bleeding, deep tissue
B. Hemophilia bleeding
C. Ulcerative bleeding • Disorders:
D. Rectal tumor 1. Hemophilia A, B, or C: ↑ PT
E. Nonspecific ulcerative colitis 2. Vitamin K deficiency
• Platelets disorders:
• Superficial bleeding: Mucosal bleeding,
skin bleeding, petechiae
• Disorders: Bernard-Soulier syndrome,
Glanzmann thrombasthenia, Immune
thrombocytopenia, Uremic platelet
dysfunction, Thrombocytopenic purpura.
• Mixed: von Willebrand disease and
Disseminated intravascular coagulation
Objective
Low platelets count → platelet bleeding disorder
→ Thrombocytopenic purpura
A 28-year-old woman complains of skin hemorrhages after
minor traumas and spontaneous appearance of hemorrhages on
the front of her torso and extremities. On examination: the skin
is variegated (old and new hemorrhages), bleeding gums. Bleeding disorders
Blood platelets - 20 · 109/L; in the bone marrow there is Types of bleeding disorders
increased number of megakaryocytes and no platelet • Coagulation factors disorders:
production. Treatment with steroid hormones was effective. • Deep bleeding: Joint bleeding, deep tissue
What is the likely diagnosis? bleeding
A. Idiopathic thrombocytopenic purpura • Disorders:
1. Hemophilia A, B, or C: ↑ PT
B. Hemophilia
2. Vitamin K deficiency
C. Rendu-Osler-Weber disease (Hereditary hemorrhagic
• Platelets disorders:
telangiectasia)
• Superficial bleeding: Mucosal bleeding,
D. Disseminated intravascular coagulation
skin bleeding, petechiae
E. Acute vascular purpura
• Disorders: Bernard-Soulier syndrome,
Glanzmann thrombasthenia, Immune
thrombocytopenia, Uremic platelet
dysfunction, Thrombocytopenic purpura.
• Mixed: von Willebrand disease and
Disseminated intravascular coagulation
26. A 35-year-old woman had acute onset of the disease Objective
that started with fever up to 39.0°C and cough. 3days later Presentation → Pleural effusion →
a dyspnea at rest increased to 35/minute. Downward from thoracocentesis (Pleural tap)
her right shoulder blade angle, percussion detects a dull
sound. No vocal fremitus, respiratory sounds cannot be
auscultated. What is the treatment tactics?
A. Artificial lung ventilation
B. Antibiotic therapy
C. Pleural tap
D. Oxygen therapy
E. Physiotherapy
A 19-year-old student was urgently hospitalized due to marked Objective
dyspnea and chest pain on the left. Her body temperature is Pleural effusion →
38.8oC. She has been presenting with these signs for 3 days. thoracocentesis
Respiratory rate is 42/min., shallow. Percussion sound is dull to
the left from the center of the scapula, no respiration can be
auscultated. The left heart border is displaced outwards by 3
cm. Embryocardia and heart rate of 110/min are observed.
Palpation of the right subcostal area is painful. What urgent
measures should be taken in this case?
A. Urgent thoracocentesis
B. Prescription of penicillin antibiotics
C. Administration of furosemide
D. Administration of cardiac glycosides
E. Referral into thoracic surgery unit
Objective
• Pleurisy, also known as pleuritis, is a medical
condition characterized by inflammation of the
Review pleura, which is the thin tissue lining the lungs
A 2-year-old child with persisting cough and subfebrile body and chest cavity, usually due to infection.
temperature after a case of URTI developed dyspnea, cyanosis • Symptoms include, chest pain, Shortness of
of the nasolabial triangle, percussion dullness and weakened breath, and fever.
respiration in the lower lobe of the right lung, and a slight
mediastinal displacement to the left. What pulmonary pathology
is likely to cause this clinical presentation?
A. Pleurisy
B. Emphysema
C. Pneumonia
D. Atelectasis
E. Bronchitis
Objective
The patient suffers from pleural effusion is
accumulation of fluid in the pleural cavity.
• Types of pleural effusion:
Review • Exudate: fluid is reach in cells and
A 57-year-old patient complains of dyspnea at rest. The patient proteins ( > 25g/l)
presents with orthopnea, acrocyanosis, bulging cervical veins. • Transudate: fluid is clear (protein <25 g/l)
On percussion: dull sound over the lower lung segments. On
auscultation: no respiratory sounds. Heart rate is 92/min. Right-
sided cardiac dilatation is observed. The liver is +7 cm. Shins
are swollen. Pleural effusion is suspected. What indicator would
confirm the presence of transudate in this case?
A. Total protein content in the pleural fluid below 25 g/L
B. Presence of atypical cells
C. Total protein content in the pleural fluid exceeding 30 g/L
D. Specific gravity exceeding 1015
E. Positive Rivalta’s test
Objective
• Pneumothorax is accumulation of air in
pleural cavity.
• Chest pain, resonance ‫ﺻدى‬, absence of
During physical exertion a man suddenly developed acute chest respiration →Spontaneous pneumothorax.
pain on the right and dyspnea. Objectively he assumes forced
half sitting position in the bed, presents with diffuse cyanosis,
resting tachypnea of 38/min., the right side of the thorax is
enlarged and does not participate in the respiratory process;
percussion on the right reveals tympanic resonance and absence
of respiration. What is the most likely diagnosis in this case?
A. Spontaneous pneumothorax
B. Hemothorax
C. Lobar pneumonia
D. Pulmonary embolism
E. Acute pleurisy
27. Disease onset was acute. A child developed general
weakness, pain in the joints, and elevated temperature. Later
these signs became accompanied by itching skin rash Vasculitis
manifested as erythematous spots 25 mm in size. The rash 1. Buerger disease (Obliterating endarteritis):
gradually turned hemorrhagic. Large joints are painful and • Risk factors: Heavy tobacco smoking
swollen; pain attacks periodically occur in the paraumbilical history, males < 40 years old.
area; there are signs of intestinal hemorrhage. What is the most • Findings: intermittent claudication. May
lead to gangrene, autoamputation of
likely diagnosis?
digits, superficial nodular phlebitis.
A. Hemorrhagic vasculitis (Henoch-Schonlein purpura)
• Segmental thrombosing vasculitis with
B. Scarlet fever vein and nerve involvement.
C. Hemorrhagic meningoencephalitis 2. Immunoglobulin A vasculitis (Henoch-
D. Streptococcal impetigo Schonlein purpura):
E. Rheumatism • Most common childhood systemic
vasculitis. Often follows URI.
• Classic triad:
1. Hinge pain (arthralgias)
2. Stomach pain (abdominal pain
associated with intussusception)
3. Palpable purpura on buttocks/legs
A 32-year-old man complains of pain in his legs that intensifies
during walking, intermittent claudication, numbness of his toes,
extremity coldness, and inability to walk more that 100 meters. Vasculitis
When he sleeps, his leg usually hangs down. The patient has 1. Buerger disease (Obliterating endarteritis):
been smoking since he was 16. He drinks alcohol in excess. The • Risk factors: Heavy tobacco smoking
left leg is colder than the right one; the skin of the extremities is history, males < 40 years old.
dry. No pulse can be detected on the pedal arteries, while • Findings: intermittent claudication. May
pulsation of the femoral arteries is retained. What is the most lead to gangrene, autoamputation of
likely diagnosis? digits, superficial nodular phlebitis.
• Segmental thrombosing vasculitis with
A. Obliterating endarteritis
vein and nerve involvement.
B. Diabetic angiopathy
2. Immunoglobulin A vasculitis (Henoch-
C. Leriche syndrome (aortoiliac occlusive disease) Schonlein purpura):
D. Raynaud disease • Most common childhood systemic
E. Deep thrombophlebitis vasculitis. Often follows URI.
• Classic triad:
1. Hinge pain (arthralgias)
2. Stomach pain (abdominal pain
associated with intussusception)
3. Palpable purpura on buttocks/legs
28. A 16-year-old boy developed dizziness. His heart rate
Objective
is 35/min, blood pressure is 85/45mmHg, heart borders are P waves disconnected from QRS complex →
not enlarged. Heart sounds are loud and clear. ECG shows Complete atrioventricular block (III block)
P waves disconnected from QRS complex, dissociation and
different rhythm of atria and ventricles is accompanied by
varying location of P wave in relation to QRST complex.
This presentation is the most characteristic of the following
disease:
A. Atrioventricular block (II degree)
B. Extrasystole
C. Complete atrioventricular block (III block)
D. Atrioventricular dissociation
E. Sinus bradycardia
29. 40% of workers who, who polish the art glass, using an Objective
abrasive disk, and have a long record of employment, are Polish working with abrasive disk for along time→
diagnosed with ulnar neuritis, 21%- with vegetative involves a lot of vibration work → hammer disease /
polyneuritis, and 12% – with vegetomyofascitis of the vibration disease
upper limbs. These pathologies are associated with the
following harmful factor:
A. Noise
B. Vibration Vibration disease (hammer disease)
C. Electromagnetic field • Vibration disease is an occupational disease
D. Dust caused by prolonged and repeated exposure to
E. Microclimate vibrating tools or equipment.
• Symptoms include numbness, discoloration,
tingling, pain, and reduced sensitivity in the
affected areas.
A 50-year-old man, who works as a polisher at a combine-
building factory, addressed the factory’s sectorial doctor with
complaints of general fatigue, sensations of numbness and pain
in his fingers. Objectively: the skin of his fingers is pale.
Reaction to pain, tactile and thermal stimuli was revealed to be
slightly disrupted. No disruptions can be observed within the
other organs and systems. What disorder is most likely?
A. Pneumatic hammer disease
B. Multiple neuritis
C. Raynaud’s disease
D. Syringomyelia
E. Deforming arthrosis
A woman has been working as a polisher for a year and a half.
Her workstation is equipped with a grinding machine (grinding
wheels). She complains of white discoloration of her fingers and Vibration disease
• Vibration disease is an occupational disease
toes that appears when she is nervous. Objectively there are no
caused by prolonged and repeated exposure to
changes in the coloration of the distal segments of her limbs.
vibrating tools or equipment.
Grip strength measured with a dynamometer is 25 kg,
• Symptoms include numbness, discoloration,
algesimetry findings are 0.1; 0.3; 0.5. Cold stimulus is tingling, pain, and reduced sensitivity in the
extremely positive on the upper and lower limbs. Internal organs affected areas.
are without pathologies. Make the diagnosis:
A. Vibration disease
B. Raynaud disease
C. Syringomyelia
D. Raynaud syndrome
E. Polyneuritis
30. A 45-year-old woman undergoes an inpatient treatment. Urinary tract infection
She complains of elevated body temperature up to 39.0 C, • Cystitis
pain in her right lumbar area, turbid urine with blood. CT • Infection of bladder – Lower urinary tract
scan shows an area of low density within the parenchyma, • Symptoms: dysuria (pain with urination),
no difference between the cortical and medullary layers, an frequency (going a lot), urgency (always feel
increased density of the perinephric fat due to edema. What like you must go), and suprapubic pain.
is the diagnosis? • Pyelonephritis
A. Renal abscess • Infection of kidney – Upper urinary tract
B. Para nephritis • Symptoms: systemic symptoms (fever, chills),
C. Glomerulonephritis flank pain, and CVA tenderness.
D. Renal carcinoma • Most infections ascend: Urethra → Cystitis →
E. Pyelonephritis Pyelonephritis
• Diagnosis:
• Urinalysis: cloudy urine, leukocyte esterase
(produced by WBCs in urine) • Nitrites
>10WBC/hp
• Culture: bacteriological inoculation >100,000
CFUs
A woman is on the 32nd week of her second pregnancy. She Urinary tract infection
complains of fever, chills, nausea, vomiting, lumbar pain, and • Cystitis
dysuria. Costovertebral angle tenderness is present on both • Infection of bladder – Lower urinary tract
sides. Urine analysis: pyuria, bacteriuria. Blood test: • Symptoms: dysuria (pain with urination),
leukocytosis. What is the most likely diagnosis? frequency (going a lot), urgency (always feel
A. Gestational pyelonephritis like you must go), and suprapubic pain.
B. Cystitis • Pyelonephritis
C. Pyelitis • Infection of kidney – Upper urinary tract
D. Glomerulonephritis • Symptoms: systemic symptoms (fever, chills),
E. Latent bacteriuria flank pain, and CVA tenderness.
• Most infections ascend: Urethra → Cystitis →
Pyelonephritis
• Diagnosis:
• Urinalysis: cloudy urine, leukocyte esterase
(produced by WBCs in urine) • Nitrites
>10WBC/hp
• Culture: bacteriological inoculation >100,000
CFUs
A 9-year-old girl complains of fever up to 38.5oC, headache,
inertness, weakness, loss of appetite, stomachache, and Urinary tract infection
frequent painful urination. Provisional diagnosis of acute • Cystitis
pyelonephritis is made. Clinical urine analysis: specific gravity • Infection of bladder – Lower urinary tract
- 1016, no protein, leukocytes - 10-15 in the vision field. What • Symptoms: dysuria (pain with urination),
investigation method can verify the diagnosis of urinary tract frequency (going a lot), urgency (always feel
infection? like you must go), and suprapubic pain.
A. Bacteriological inoculation of urine • Pyelonephritis
B. Rehberg test (creatinine clearance test) • Infection of kidney – Upper urinary tract
C. Zymnytsky test (density measurement of daily diuresis) • Symptoms: systemic symptoms (fever, chills),
D. Complete blood count flank pain, and CVA tenderness.
E. Clinical urine analyses, dynamic testing • Most infections ascend: Urethra → Cystitis →
Pyelonephritis
• Diagnosis:
• Urinalysis: cloudy urine, leukocyte esterase
(produced by WBCs in urine) • Nitrites
>10WBC/hp
• Culture: bacteriological inoculation >100,000
CFUs
An 8-year-old girl complains of frequent painful urination in Urinary tract infection
small amounts and urinary incontinence. The signs have been • Cystitis
present for 2 days already. She explains her disease by • Infection of bladder – Lower urinary tract
overexposure to cold. Costovertebral angle tenderness is absent. • Symptoms: dysuria (pain with urination),
Complete blood count is without pathologies. Urine test: frequency (going a lot), urgency (always feel like
you must go), and suprapubic pain.
leukocytes - 20-30 in the vision field, erythrocytes - 40-50 in the
• Pyelonephritis
vision field, unchanged, bacteriuria. What is the most likely
• Infection of kidney – Upper urinary tract
diagnosis?
• Symptoms: systemic symptoms (fever, chills),
A. Cystitis
flank pain, and CVA tenderness.
B. Vulvitis • Most infections ascend: Urethra → Cystitis →
C. Pyelonephritis Pyelonephritis
D. Glomerulonephritis • Diagnosis:
E. Urolithiasis • Urinalysis: cloudy urine, leukocyte esterase
(produced by WBCs in urine) • Nitrites
>10WBC/hp
• Culture: bacteriological inoculation >100,000 CFUs
A man was brought into the admission room after an
overexposure to cold. He complains of sharp pain in the
small of his back and elevated body temperature up to
38oC. He took some aspirin. Blood test: leukocytes - 10.5
· 1012/L, eosinophils - 5%, band neutrophils - 8%,
segmented neutrophils - 51%, lymphocytes - 32%,
monocytes - 4%, erythrocyte sedimentation rate - 28
mm/hour. Urinalysis: protein - 0.6 g/L, leukocytes - cover Urinary tract infection
the whole vision field, large amount of mucus. What is • Cystitis
the most likely diagnosis? • Infection of bladder – Lower urinary tract
A. Acute pyelonephritis • Symptoms: dysuria (pain with urination), frequency
B. Chronic pyelonephritis (going a lot), urgency (always feel like you must go), and
suprapubic pain.
C. Acute glomerulonephritis
• Pyelonephritis
D. Tubulointerstitial nephritis
• Infection of kidney – Upper urinary tract
E. Subacute malignant glomerulonephritis
• Symptoms: systemic symptoms (fever, chills), flank pain,
and CVA tenderness.
• Most infections ascend: Urethra → Cystitis → Pyelonephritis
Note • Diagnosis:
Chronic pyelonephritis: happens due to recurrent • Urinalysis: cloudy urine, leukocyte esterase (produced by
episodes of pyelonephritis; fibrosis and long time WBCs in urine) • Nitrites >10WBC/hp
is a key. • Culture: bacteriological inoculation >100,000 CFUs
31. A patient is 45-year-old. He was referred for a
consultation with a psychiatrist due to complains of Somatoform autonomic dysfunction
abdominal pain and discomfort that occur in emotional • What is it? physical symptoms that cannot be
staining situation. Objectively, no changes of the explained by any known medical condition.
gastrointestinal tract was detected. The complaints emerged • The symptoms:
over 10 years ago against the background of severe alcohol • Pain, fatigue, dizziness, palpitations, and
poisoning. The patient has been repeatedly visiting gastrointestinal disturbances, among others.
gastroenterologists, who were unable to find any significant • symptoms are believed to be related to
psychological or emotional factors, rather than to
changes in the patient. The prescribed therapy was
any underlying physical illness.
ineffective. What is the likely conclusion?
• Somatoform → physical symptoms .
A. Organic brain disorder
• ِAutonomic dysfunction → symptoms related to ANS
B. Chronic alcoholism such as heart rate, blood pressure, and digestion.
C. No disorder
D. Somatoform autonomic dysfunction
E. Functional dyspepsia
A 20-year-old woman complains of sensation of air shortage,
lingering dull pain in the heart area, irritability. Objectively:
general condition is satisfactory, heart rate lability, BP is 130/60
mm Hg. ECG reveals disruption of repolarization proceses. The
patient is diagnosed with somatoform autonomic dysfunction of Objective
cardial type. Specify the conditions of the patient treatment: Somatoform autonomic dysfunction → follow with
A. Out-patient treatment out-patient treatment (no need for hospitalization).
B. In-patient treatment in therapeutics department
C. In-patient treatment in cardiology department
D. In-patient treatment in cardiac surgery department
E. In-patient treatment in psychiatric department
A 15-year-old girl complains of dizziness and sensation of lack
of air that she develops in emotionally straining situations. Somatoform autonomic dysfunction
• What is it? physical symptoms that cannot be
Relief occurs after she takes corvalol. Objectively: explained by any known medical condition.
hyperhidrosis and marble-like pattern of the skin of her palms • The symptoms:
and feet. Clinical and instrumental examination revealed no • Pain, fatigue, dizziness, palpitations, and
organic changes in the central nervous, cardiovascular, and gastrointestinal disturbances, among others.
respiratory systems. What provisional diagnosis can be made? • symptoms are believed to be related to
A. Somatoform autonomic dysfunction psychological or emotional factors, rather than to
B. Obstructive bronchitis any underlying physical illness.
• Somatoform → physical symptoms .
C. Bronchial asthma
• ِAutonomic dysfunction → symptoms related to ANS
D. Stenosing laryngotracheitis such as heart rate, blood pressure, and digestion.
E. Acute epiglottitis
A 30-year-old woman complains of pain in the heart area Objective
(”aching, piercing pain”) that arises primarily in the morning Physical symptoms (heart pain) with no
hours in autumn and spring. Pain irradiates into the neck, back, pathology seen in patients with depression →
abdomen and is attended by rapid heart rate and low vital Somatization.
tonus. This condition occurs independently from physical
exertion. In the evening her condition improves. Somatic and
neurologic state and ECG have no pathologies. What pathology
is likely to result in such clinical presentation?
A. Somatized depression
B. Rest angina pectoris
C. Pseudoneurotic schizophrenia
D. Somatoform autonomic dysfunction
E. Hypochondriacal depression
A 35-year-old woman complains of heart pain (”aching and
drilling”) occurring mainly in the morning in autumn and spring
and irradiating to the neck, back and abdomen; rapid heartbeat;
low vitality. Occurrence of this condition is not associated with
physical activity. In the evening, the patient’s condition Objective
improves. Study of somatic and neurological status and ECG • Physical symptoms (heart pain) with no
reveal no pathology. What pathology is most likely to have pathology seen in patients with depression →
caused these clinical presentations? Somatization.
A. Somatization depression
B. Resting stenocardia
C. Pseudoneurotic schizophrenia
D. Neurocirculatory asthenia
E. Hypochondriacal depression
Objective
A 20-year-old student after failing an exam developed
• Hysterical neurosis: a group of psychological
complaints of a sensation of a round foreign body in her throat,
disorders characterized by physical symptoms
difficult swallowing. She fixates on her condition, limits her
without an apparent physical cause.
diet, often cries, seeks attention, exhibits demonstrative attitude.
• Hypochondriacal neurosis: excessive anxiety or fear
She is highly susceptible to psychotherapeutic suggestion. What
about having a serious illness, despite having no or
psychiatric diagnosis can be made in this case?
only mild physical symptoms
A. Hysterical neurosis
• Depressive neurosis: prolonged and persistent
B. Hypochondriacal neurosis
feelings of sadness, hopelessness, and a loss of
C. Depressive neurosis
interest in activities that were once pleasurable.
D. Obsessive neurosis
• Obsessive neurosis: persistent and unwanted
E. Paranoid personality disorder
thoughts, impulses, or images (obsessions) that cause
significant distress and anxiety
A 32-year-old woman complains of episodes of intense fear that
• Paranoid personality disorder: type of personality
occur without visible cause and last for 10-20 minutes; the
disorder characterized by a pervasive distrust and
episodes are characterized by rapid pulse, sweating, labored
suspicion of others, including their motives and
breathing, and vertigo. Specify the likely diagnosis:
intentions.
A. Panic disorder
• Panic disorder is a type of anxiety disorder
B. Paranoid syndrome
characterized by recurring and unexpected panic
C. Manic syndrome
attacks. Symptoms such as sweating, trembling,
D. Simple schizophrenia
shortness of breath, chest pain, and a feeling of
E. Claustrophobia
choking or suffocation.
32. An unconscious patient was delivered to a hospital by
Objective
an ambulance. Objectively, his body temperature is 39.0°C, Somatoform autonomic dysfunction → follow with
he presents with convulsion and red dry skin. It is known out-patient treatment (no need for hospitalization.
that the patient works as a stoker in the boiler room. What
is the likely diagnosis?
A. CO poisoning
B. Heat stroke
C. food poisoning
D. Hypertensive urgency
E. Acute viral respiratory infection
33. An 18-year-old patient always obeys others and adapts his
needs to the demands of the people on whom he depends. He Objective
excessively defers to their wishes and makes them responsible The patient suffers from dependent personality
for his wellbeing, cannot defend his interests and needs support disorder (DPD) is a mental health condition
characterized by an excessive need for
from other people. Such psychic profile has been formed in the
reassurance, support, and guidance from others.
childhood, remains unchanged, and hinders adaptation. What
“always obeys others”.
psychic disorder is observed in this patient?
A. Dependent personality disorder
B. Anxiety (avoidant) personality disorder
C. Anankastic personality disorder
D. Markedly accentuated personality
E. Psychopathy-like state
34. A 22-year-old postparturient woman on the 12th day Lactational mastitis
after the normal childbirth informs of elevated body 1. Infiltrative mastitis:
temperature up to 39oC for the last 3 days and pain in her • Both: pus, fever, hyperemia, pain and
right mammary gland. The right mammary gland is enlargement of brest.
enlarged, hot to touch, tense, hyperemic, and painful. • Infliltrative is a milder form of mastitis.
• Respond well to antibiotics.
Palpation reveals there a dense infiltration 8x8 cm with a
2. Phlegmonous mastitis:
fluctuation in its center. What is the most likely diagnosis?
• More sever and potentially life threatening.
A. Postpartum period, day 12. Right-sided infiltrative-
• Multiple area of breast are involved with
purulent mastitis
pus and deformated.
B. Postpartum period, day 12. Right-sided serous mastitis • Usually requires hospitaliazation.
C. Postpartum period, day 12. Right-sided gangrenous
mastitis
D. Postpartum period, day 12. Right-sided phlegmonous
mastitis
E. Postpartum period, day 12. Right-sided lactostasis
Objective
Lactational mastitis
Breast feeding → increased risk of infection →
Review Lactational mastitis.
2 weeks after labour a parturient woman developed breast pain
being observed for 3 days. Examination revealed body
temperature at the rate of 39oC, chills, weakness, hyperaemia,
enlargement, pain and deformity of the mammary gland. On
palpation the infiltrate was found to have an area of softening
and fluctuation. What is the most likely diagnosis?
A. Infiltrative-purulent mastitis
B. Phlegmonous mastitis
C. Lactostasis
D. Serous mastitis
E. Mastopathy
Objective
Lactational mastitis
Breast feeding → increased risk of infection →
Lactational mastitis.
A postparturient woman, who has been breastfeeding for 3
weeks, made an appointment with the doctor. For the last 6
days she has been feeling unwell, complains of body
temperature of 38-39oC, general weakness; within the last 2
days she developed pain and redness in the area of her right
mammary gland. Examination revealed her mammary gland to
be significantly enlarged and deformed; breast tissue
fluctuations and lymphadenitis are observed. What type of
mastitis is the most likely?
A. Phlegmonous mastitis
B. Serous mastitis
C. Infiltrative mastitis
D. Lactostasis
E. Mammary edema
Lactational mastitis
1. Infiltrative mastitis:
• Both: pus, fever, hyperemia, pain and
On the 9th day after childbirth the obstetric patient developed
enlargement of brest.
high fever up to 38oC. She complains of pain in the right
• Infliltrative is a milder form of mastitis.
mammary gland. The examination revealed the following: a
• Respond well to antibiotics.
sharply painful infiltrate can be palpated in the right mammary
2. Phlegmonous mastitis:
gland, the skin over the infiltrate is red, subareolar area and • More sever and potentially life threatening.
nipple are swollen and painful. What is your diagnosis? • Multiple area of breast are involved with
A. Abscess of the right mammary gland pus and deformated.
B. Mastopathy • Usually requires hospitaliazation.
C. Cancer of the right mammary gland
D. Serous mastitis
E. Fibrous cystic degeneration of the right mammary gland
Mastitis abscess
• Abscess of the mammary gland is a localized
collection of pus within the breast tissue,
usually caused by a bacterial infection.
• Symptoms: Painful, swollen, and tender breast
with redness of the skin, fever, and chills.
• Treatment
• Drainage of the abscess.
• Antibiotics.
Objective
36,6oC → No mastitis → Lactostasis
A maternity patient breastfeeding for 1,5 weeks has attended a
doctor. She considers the onset of her disease to be when
• Lactostasis, also known as breast engorgement, is
proportional breast engorgement occurred. Mammary glands
a condition characterized by swelling, warmth,
are painful. Body temperature is 36,6oC. Expression of breast
and tenderness of the breast due to the
milk is hindered. The most likely diagnosis is:
accumulation of milk. It typically occurs in the
A. Lactostasis
early postpartum period but can also occur later
B. Infiltrative mastitis
during lactation.
C. Suppurative mastitis • Symptoms of lactostasis include breast pain,
D. Chronic cystic mastitis swelling, redness, and a sensation of fullness or
E. Gangrenous mastitis heaviness in the breast. There may also be a
decrease in milk production, fever, and general
malaise.
35. A woman complains of frequent, liquid stool (up to 9-10
times per day) with mucus and blood admixtures, dull pain in
the hypogastrium, weight loss of 4 kg within the last year.
Objectively: malnutrition, dry skin, low turgor, aphthous
stomatitis. The stomach is soft, the sigmoid colon is spastic and
painful on palpation. Occult blood test is positive.
Fibrocolonoscopy: edema, hyperemia, mucosal granulation,
pseudopolyps, small ulcers with irregular edges. Make the
diagnosis:
A. Nonspecific ulcerative colitis
B. Chronic enterocolitis
C. Colon cancer
D. Irritable bowel syndrome
E. Crohn’s disease (regional enteritis)
A 51-year-old female patient complains of
frequent defecation and liquid blood-streaked
stools with mucus admixtures, diffuse pain in
the inferolateral abdomen, 6 kg weight loss
within the previous month. Objectively: body
temperature - 37,4oC, malnutrition, skin is pale
and dry. Abdomen is soft, sigmoid is painful
and spasmodic, makes a rumbling sound. Liver
is dense, painful, extends 3 cm below the costal
margin. What is the most likely diagnosis?
A. Non-specific ulcerative colitis
B. Bacillary dysentery
C. Sprue
D. Intestinal enzymopathy
E. Helminthic invasion
Objective
A 32-year-old woman complains of increasing spastic pains in
Irritable bowel syndrome – ‫اﻟﻘوﻟون اﻟﻌﺻﺑﻲ‬
her lower abdomen that occur after emotional stress. Bowel • Irritable bowel syndrome is a recurrent
movements are intermittent: 2-3 defecations after waking in the abdominal pain with altered bowel habit
morning alternate with constipations that last for 1-2 days. without unique organic pathology.
Objectively body mass is retained, palpation of the sigmoid • Symptoms:
colon is moderately painful. Hb- 130 g/L, leukocytes 5.2 g/L, 1. Abdominal pain or discomfort
ESR- 9 mm/hour. Rectoromanoscopy is painful due to spastic 2. Gas and bloating.
condition of the intestine, intestinal mucosa is without changes. 3. Mucus in the stool.
Intestinal lumen contains large amounts of mucus. What is the 4. Changes in bowel habits: diarrhea,
most likely diagnosis in this case? constipation, or alternating bouts of both
A. Crohn’s disease (regional enteritis) • Aِِssociated with mood disorders (anxiety,
B. Nonspecific ulcerative colitis depression).
C. Irritable bowel syndrome
D. Acute mesenteric ischemia
E. Malabsorption syndrome
36. A 12-year-old girl after a case of respiratory infection Objective
developed dyspnea at rest, paleness of skin. Heart rate is Infectious myocarditis is a potential
110/min, BP is 90/55 mmHg. Heart sounds are muffled. complication of viral respiratory infections and
Borders of relative heart dullness: right- the left parasternal can cause pericardial effusion
line, upper- the III rib, left- 1,0 cm outwards from the
midclavicular line. Make the provisional diagnosis:
A. Functional cardiomyopathy
B. Somatoform autonomic dysfunction
C. Exudative pericarditis
D. Infectious myocarditis
E. Hypertrophic cardiopathy
An 8-year-old boy developed a temperature of 37,5oC two days
Nonrheumatic myocarditis
after his recovery from the case of URTI. He complains of • This presentation is typical for Nonrheumatic
suffocation, heart pain. Objectively: the skin is pale, myocarditis mainly because of the duration:
tachycardia, the I heart sound is weakened, short systolyc short duration between myocarditis and
murmur in the 4th intercostal area near the left edge of the infection; 2 days.
breastbone. What heart disorder such clincal presentation is • For rheumatic fever myocarditis duration is
characteristic of? usually 2 weeks after strep infection.
A. Nonrheumatic myocarditis
B. Primary rheumatic carditis
C. Myocardiodystrophy
D. Fallot’s tetrad
E. Cardiomyopathy
37. A 68-year-old woman with congestive heart failure and Objective
left ventricular ejection fraction of less than 40% receives Digoxin can increase risk of ventricular
the following pharmacotherapy scheme: ramipril, arrhythmia. In this case, it has to be
torasemide, bisoprolol, clopidogrel and digoxin. During discontinued from the therapy plan.
one of her regular examination, frequent polymorphic
ventricular extrasystole were detected in the patient. What
medicine should be removed from her therapy scheme?
A. Clopidogrel
B. Ramipril
C. Torasemide
D. Digoxin
E. Bisoprolol
38. You witnessed a car accident. When examining the
Bleeding first aid
place of the accident you noticed a man of about 30 years,
• Neck bleeding → Digital occlusion, Mikulich
who was hit by the car. He is unconscious. On his neck on
method
the left there is a profuse hemorrhage with bright-red
• Bleeding below the Poupart’s ligament (inguinal
blood. How to stop this hemorrhage? ligament) → compression band
A. Maximal hyperextension of the neck
• Thigh bleeding → Tourniquet above the wound
B. Digital occlusion, Mikulich method
C. Apply a plaster cast
D. Apply a neck brace
E. Put him in a stable position
Bleeding first aid
• Neck bleeding → Digital occlusion, Mikulich
method
• Bleeding below the Poupart’s ligament (inguinal
ligament) → compression band
• Thigh bleeding → Tourniquet above the wound
A 24-year-old patient received a puncture injury below the
Poupart’s ligament accompanied by intense arterial bleeding.
The best method to temporarily stop the bleeding in the patient
would be:
A. Compression band
B. Esmarch’s tourniquet
C. Maximum limb bending
D. Compressing a blood vessel with a clamp
E. Wound suturing
Bleeding first aid
• Neck bleeding → Digital occlusion, Mikulich
method
• Bleeding below the Poupart’s ligament (inguinal
ligament) → compression band
Review • Thigh bleeding → Tourniquet above the wound
A 30-year-old patient was in a car accident. He is unconscious,
pale, has thready pulse. In the middle third of the right thigh
there is an extensive laceration with ongoing profuse external
arterial bleeding. What urgent actions must be taken to save the
life of the patient?
A. Tourniquet above the wound of the right thigh
B. Tourniquet below the wound of the right thigh
C. Artificial lung ventilation
D. Precordial thump
E. Application of plaster bar
Restrictive and obstructive lung appearacne

Expiration problem Inspiration problem


39. A patient was brought into the pulmonology Idiopathic pulmonary fibrosis (IPF)
department with complaints of inspiratory dyspnea and dry • One of the restrictive disease (inspiration)
cough at the highest point of inhalation. On the • Clubbing → “Hippocratic fingers”→ sign of
examination the following is observed: pale skin, cyanotic hypoxemia which is typical in patients with IPF.
lips, “Hippocratic fingers”. Auscultation detects Velcro- • Velcro-type or cellophane-type crackles →
type crackles (like opening a Velcro fastener). X- ray shows stifness → fibrosis.
a “ground glass opacity” pattern. What is the most likely • Ground glass → Hazy grayish appearance →
diagnosis? increased lung density → Fibrosis.
A. Hand-Schuller-Christian disease
B. Exogenous allergic alveolitis
C. Idiopathic pulmonary hemosiderosis
D. Idiopathic pulmonary fibrosis
E. Pulmonary histiocytosis X
39. A patient was brought into the pulmonology Idiopathic pulmonary fibrosis (IPF)
department with complaints of inspiratory dyspnea and dry • One of the restrictive disease (inspiration)
cough at the highest point of inhalation. On the • Clubbing → “Hippocratic fingers”→ sign of
examination the following is observed: pale skin, cyanotic hypoxemia which is typical in patients with IPF.
lips, “Hippocratic fingers”. Auscultation detects Velcro- • Velcro-type or cellophane-type crackles →
type crackles (like opening a Velcro fastener). X- ray shows stifness → fibrosis.
a “ground glass opacity” pattern. What is the most likely • Ground glass → Hazy grayish appearance →
diagnosis? increased lung density → Fibrosis.
A. Hand-Schuller-Christian disease
B. Exogenous allergic alveolitis
C. Idiopathic pulmonary hemosiderosis
D. Idiopathic pulmonary fibrosis
E. Pulmonary histiocytosis X
A 52-year-old patient suffers from marked dyspnea during Idiopathic pulmonary fibrosis (IPF)
physical exertion, non-productive cough. The patient’s condition • One of the restrictive disease (inspiration)
has been persisting for 8 months. The patient has been a smoker • Clubbing → “Hippocratic fingers”→ sign of
for 30 years. In the lungs there are cellophane-type crackles hypoxemia which is typical in patients with IPF.
auscultated on both sides. Respiration rate is 26/min., oxygen • Velcro-type or cellophane-type crackles →
saturation of blood is 92%. On spirometry: moderate restrictive- stifness → fibrosis.
type disturbance of external respiration. What is the most likely • Ground glass → Hazy grayish appearance →
diagnosis? increased lung density → Fibrosis.
A. Idiopathic fibrosing alveolitis
B. Chronic obstructive pulmonary disease (COPD)
C. Chronic bronchitis
D. Community-acquired pneumonia
E. Sarcoidosis
40. A 31-year-old drug-addicted person complains of a Objective
cough with bloody expectorations, dyspnea, persistent Drug-addicted person →
fever, and leg edemas. The jugular veins are distended. Risk of endocarditis from staph →
There is a coarse pansystolic murmur detected above the Tircuspid stenosis (Pansystolic murmur left sternal)
base of the xiphoid process and in the second intercostal
space on the left, close to the edge of the sternum. Heart
sounds are clear, arrythmia is detected, heart rate is
128/min, pulse- 82/min, blood pressure is 100/70 mmHg.
What is the
most likely diagnosis?
A. Infective endocarditis
B. Lutembacher syndrome
C. Coarctation of the aorta
D. Community acquired pneumonia
E. Pulmonary embolism
41. A 48-year-old woman complains of disturbed menstrual Objective
cycle: her periods last for 7-9 days and are excessively profuse • Climacteric syndrome is an old name for
throughout the last half-year. She notes occasional hot flashes in menopause Age > 45 ‫ﺳن اﻟﯾﺄس ﻋﻧد اﻟﻧﺳﺎء‬.
her head, insomnia, irritability, and headaches. Her skin is of • Findings:
normal color. Blood pressure - 150/90 mm Hg, pulse - 90/min., • Hot flashes (very common).
rhythmic. The abdomen is soft and painless. Bimanual • Disturbed menstrual cycle.
examination shows no uterine enlargement, the appendages • Mood changes, including irritability,
cannot be detected. The vaginal fornices are free. What is the depression, and anxiety
most likely diagnosis? • Insomnia or sleep disturbances
A. Climacteric syndrome • Decreased libido or sexual desire
B. Premenstrual syndrome • Urinary incontinence or increased
urgency to urinate
C. Adrenogenital syndrome
• Joint and muscle pain
D. Stein-Leventhal syndrome (polycystic ovary syndrome)
• Weight gain or difficulty losing weight
E. Uterine myoma
• Memory problems or difficulty
concentrating
42. The body of a citizen was found at the place of his Forensics
dwelling. On his face, neck, and hands there were detected Discription of the wound → Bite wounds
irregular-shaped wounds, varying from 2×3 cm to 4×5 cm
in size. The skin and underlying tissues are absent in the
wounds. The margins of the wounds are uneven, with
major and minor scalloping along the edges and no signs of
bleeding. What is the initiating mechanism of these
wounds?
A. Blast injury
B. Animal bite wounds
C. Multiple stab and incised wounds
D. Local effect of cold
E. Pellet gunshot wound
43. A family doctor performed an external obstetrical Objective
examination on a pregnant woman and determined that her Uterine fundus is located at the level of the navel →
uterine fundus is located at the level of the navel. What is 20-26 → 24 is the correct answer
the most likely term of pregnancy in this woman?
A. 24 weeks
B. 8 weeks
C. 16 weeks
D. 40 weeks
E. 32 weeks
It is the 3rd day after the first normal term labor; the infant is Objective
rooming-in with the mother and is on breastfeeding. Physiological involution is a normal decrease of
Objectively: the mother’s general condition is satisfactory. size of uterus to the normal size after deliver of
Temperature is 36.4oC, heart rate is 80/min., BP is 120/80 mm baby; takes 6 weeks to complete.
Hg. Mammary glands are soft and painless; lactation is
moderate, unrestricted milk flow. The uterus is dense, the
uterine fundus is located 3 finger widths below the navel.
Lochia are sanguino-serous, moderate in volume. Assess the
dynamics of uterine involution:
A. Physiological involution
B. Subinvolution
C. Lochiometra
D. Pathologic involution
E. Hematometra
44. An 8-day-old boy was delivered to the hospital on the Phlegmon of the newborn
second day after the onset of the disease. His parents
(cellulitis of the newborn)
complain of his fussiness, regurgitation, body temperature
• What is it? A bacterial neonatal infection affects
up to 38.5 C, red skin with infiltration in the lumbar area.
the subcutaneous tissue, fascia, and muscles.
His medical history has no peculiarities. The child is in the
• Cuased by: Staphylococcus aureus and
severe condition, inert, pale, suckles poorly. In the lumbar
streptococcus pyogenes
area, on the sacrum and buttocks there is a tense infiltration
• Symptoms: fever, lethargy, poor feeding, and
with hyperemic and cyanotic areas and with a soft spot 8×7
swelling, redness, infilteration and tenderness of
cm in its center. the stool is 10 times in 24 hours, with
the affected area
green and mucous admixtures. What is the most likely
diagnosis?
A. Erysipelas
B. Congenital soft-tissue tumor
C. Phlegmon of the newborn
D. Adiponecrosis
E. Hemangioma
45. A 5-year-old child had acute onset of the disease that
manifested in body temperature up to 39.5° C, marked
chills, weakness, inertness, skin pallor, and headache. 8
hours later a hemorrhagic rash developed on the skin of the
buttocks and legs. The child is sluggish, the body
temperature has dropped, blood pressure is 80/40,
respirations are 28-30/min, diuresis is decreased. Make the
provisional diagnosis:
A. thrombocytopenic purpura
B. Hemorrhagic vasculitis (Henoch-Schonlein purpura)
C. Measles
D. Meningococcemia
E. Reye syndrome

Objective
Hemorrhagic rash and hypotension → Meningococcemia.
46. A 45-year-old man came to the hematologist with
Objective
complaints of general weakness, elevated body
CT would provide a more comprehensive view of
temperature, excessive sweating, enlarged cervical lymph
the entire neck region and surrounding tissues.
nodes. Objectively, his body temperature is 37.5 C, the skin
is pale and dry, the posterior cervical lymph nodes are
dense and elastic, up to 2cm in diameter, mobile.There are
no peculiarities in the patient’s heart and lungs.
Hepatosplenomegaly was detected. What examination is
necessary to determine the scope of pathologic process?
A. Bone scintigraphy
B. Abdominal X-ray
C. Complete blood count
D. Ultrasound of the cervical lymph nodes
E. Computed tomography
47. A 39-year-old man, a battery attendant, suddenly developed Objective
weakness, loss of appetite, nonlocalized colicky abdominal Highlighted symptoms are typical for lead
pains, and nausea. Objectively his skin is gray; there is a pink- poisoning (Saturnism).
gray stripe on his gums; the stomach is soft and sharply painful.
Blood test detected erythrocytes with basophilic stippling and
anemia. The patient has a history of peptic ulcer disease of the
stomach. Constipation tends to occur every 3-4 days. What is
the most likely provisional diagnosis?
A. Saturnism (lead poisosning)
B. Acute appendicitis
C. Perforation of gastric ulcer
D. Acute cholecystitis
E. Chronic alcoholism
48. A 28-year-old man, a teacher, after emotional stress
developed painful muscle spasms in his right hand that occur
during writing; now he has to hold the pen between the second
and third fingers. He has no problems with typing or writing on
the blackboard; no other motor disturbances or neurological
pathologies are detected. What is the most likely diagnosis?
A. Writer’s cramp
B. Cortical agraphia
C. Parkinsonism
D. Neuropathy of the right ulnar nerve
E. Neuropathy of the right radial nerve
49. An 18-year-old patient complains of skin rash. The patient
has been suffering from this condition for 5 years. The first Objective
instance of this disease occurred after a car accident. • Psoriasis is a chronic autoimmune condition
Objectively: the patient presents with papular rash covered in that affects the skin causing papules and
plaques with silvery scaling.
silvery scales, “thimble” symptom (small pits on the nails),
affected joints. What is the most likely diagnosis? • Positive Grattage test (Auspitz sign): pinpoint
bleeding when scaled off.
A. Psoriasis
B. Panaritium
C. Onychomycosis
D. Lupus erythematosus
E. Rheumatism
Review Objective
A 33-year-old man developed multiple rashes on the skin of his • Psoriasis is a chronic autoimmune condition
torso and extensor surfaces of his upper and lower limbs. The that affects the skin causing papules and
rashes itch and occasionally fuse together and form plaques. The plaques with silvery scaling.
elements of rash are covered with silver-white fine scales that • Positive Grattage test (Auspitz sign): pinpoint
easily flake off when scratched. Grattage test results in three bleeding when scaled off.
sequential phenomena: stearin spot, terminal film, and punctate
hemorrhage. What diagnosis can be suspected?
A. Psoriasis
B. Parapsoriasis
C. Pyoderma
D. Lichen ruber planus
E. Secondary papular syphilid
50. A 26-year-old woman was hospitalized into the Objective
gynecological department with the complains of body Vacuum aspirator (uterine curettage) can remove
temperature up to 38.2 C, fever, general weakness, and any remaining tissue, and antibiotics can help
dirty-red blood discharge from her genital tracts. She is prevent infection.
hemodynamically stable. Two days ago, she underwent a
medical abortion on the 8th week of pregnancy. Ultrasound
detects their mains of the fertilized egg in her uterine
cavity. What are the tactics of the patient management in
this case?
A. Pipelle biopsy
B. Uterine cavity treatment with antibiotic solutions
C. Revision of the uterine cavity with vacuum aspirator.
Antibiotic therapy
D. Laparotomy, extirpation of the uterus and tubes.
Abdominal drainage
E. Laparotomy, supravaginal uterine amputation.
Abdominal drainage
The gynecology unit received a patient with uterine bleeding Incomplete abortion
that started 6 hours after induced abortion at the term of 11-12 • What is it? type of abortion where some of the
weeks. Objectively the skin is pale, pulse is 100/min., blood fetal or placental tissue remains in the uterus
pressure is 100/70 mm Hg. On vaginal examination the uterus after the termination of the pregnancy.
is painless, its enlargement corresponds to the 10th week of • Findings:
pregnancy; uterine cervix is dilated enough to let in one finger, • Heavy bleeding, often with large blood
there are fragments of the fertilized ovum. What actions should clots.
be taken next: • Abdominal or pelvic pain
A. Urgent repeated curettage of the uterine cavity • Fever (infection)
B. Uterotonic drugs • Foul-smelling vaginal discharge
C. Treatment for acute anemia • Diagnosis: physical examination, ultrasound
D. Antibacterial agents imaging, and blood tests
E. Prescribe rest and continue to monitor the patient’s • Treatment: Dilation and curettage (D&C), a
condition surgical procedure to remove the remaining
tissue from the uterus.
Review questions about abortion

A woman with the pregnancy term of 8 weeks complains of


elevated temperature up to 37.6oC, skin rash that can be
characterized as macular exanthema, enlargement of posterior Objective
cervical and occipital lymph nodes, small amount of bloody • Rubella infection in the 1st trimester can cause
discharge from the genital tracts. She was examined by the serious congenital malformations including,
such as deafness, blindness, heart defects, and
infectious diseases specialist and diagnosed with rubella. What
intellectual disability.
tactics should the obstetrician-gynecologist choose?
• In some countries, rubella infection during the
A. Abortion first trimester of pregnancy can be considered
B. Prescription of antibacterial therapy as a justifiable reason for abortion.
C. Prescription of antiviral therapy
D. Treatment of incipient abortion
E. Prescription of hemostatic therapy
Review questions about abortion

A 25-year-old woman was brought into the gynecological


department with profuse bloody discharge from her genital
tracts. She is 12 weeks pregnant, the pregnancy is planned.
Within the last 3 days she was experiencing pains in her lower Objective
abdomen that eventually started resembling cramps, she Presence of fragments of the fertilized ovum is a
developed bleeding. Her skin is pale, pulse - 88/min., blood sign of abortion.
pressure - 100/60 mm Hg, body temperature - 36.8oC. Vaginal
examination: the uterus size corresponds with 11 weeks of
pregnancy, the cervical canal allows inserting 1 finger and
contains fragments of the fertilized ovum, the discharge is
bloody and profuse. What is the most likely diagnosis?
A. 12-week pregnancy, spontaneous abortion in progress
B. 12-week pregnancy, threatened spontaneous abortion
C. Disturbed menstrual cycle, hyperpolymenorrhea
D. Disturbed menstrual cycle, amenorrhea
E. Full-term pregnancy, term labor
Review questions about abortion

A 27-year-old woman complains of foul-smelling discharge


from her genital tracts, pain in her lower abdomen, and elevated Objective
temperature. The complaints arose 2 days ago. She has a history • Postabortal endometritis
of surgical abortion at the term of 8 weeks one week ago. Mirror • An infection of the lining of the uterus that
examination: the uterine cervix is clear, external orifice occurs after an abortion. The condition is
produces foulsmelling discharge. Vaginal examination: the usually caused by bacteria entering the uterus
uterus lies in anteflexion, is mobile, painful, and slightly during the abortion procedure.
enlarged. The appendages are without changes. Make the • Symptoms of postabortal endometritis may
provisional diagnosis: include fever, pelvic pain, foul-smelling
A. Postabortal endometritis vaginal discharge, and heavy bleeding.
B. Enterocolitis • If left untreated, postabortal endometritis can
C. Appendicitis lead to serious complications, such as pelvic
D. Acute respiratory disease abscesses, sepsis, and infertility.
E. Salpingoophoritis
51. A 32-year-old woman complains of general fatigue, low- Objective
grade fever persisting for 4 months, lumbar pain, and dysuria. Tuberculosis and renal symptoms →
Anamnesis includes frequent acute respiratory diseases, Nephrotuberculosis
overexposure to cold, lowcalorie diet, a case of pulmonary
tuberculosis in childhood. Clinical urine analysis: pH4.8,
leukocyturia, hematuria. Complete blood count: leukocytosis,
lymphocytosis, increased ESR. Urography concludes:
dilatation of renal pelvis and calyceal system of both kidneys,
foci of calcification in the projection of right kidney
parenchyma. What is the most likely diagnosis?
A. Nephrotuberculosis
B. Right renal cyst
C. Right renal carcinoma
D. Acute glomerulonephritis
E. Chronic pyelonephritis
52. A woman with the pregnancy term of 8 weeks complains of Objective
elevated temperature up to 37.6oC, skin rash that can be • Rubella infection in the 1st trimester can cause
characterized as macular exanthema, enlargement of posterior serious congenital malformations including,
cervical and occipital lymph nodes, small amount of bloody such as deafness, blindness, heart defects, and
discharge from the genital tracts. She was examined by the intellectual disability.
infectious diseases specialist and diagnosed with rubella. What • In some countries, rubella infection during the
tactics should the obstetrician-gynecologist choose? first trimester of pregnancy can be considered
A. Abortion as a justifiable reason for abortion.
B. Prescription of antibacterial therapy
C. Prescription of antiviral therapy
D. Treatment of incipient abortion
E. Prescription of hemostatic therapy
53. Forensic autopsy of a body of a 59-year-old man, who
died suddenly at home without signs of violent death,
shows pink skin and mucosa, liquid bright- red blood, and
bright-red plethoric internal organs. Forensic toxicology
testing detected 1.44% of ethanol in the blood and
carboxyhemoglobin levels of 55%. What is the cause of
death?
A. Carbon monoxide poisoning
B. Alcohol poisoning
C. Arsenic poisoning
D. Aniline poisoning
E. Potassium cyanide poisoning
40-50 minutes after the completion of repair works conducted
in a closed garage, with car engine running, the repair workers
developed severe headache in the temporal area, nausea,
tinnitus, vertigo, etc. These symptoms are characteristic of
acute poisoning with: Objective
A. Carbon monoxide Motor exhaust → Carbon monoxide
B. Aldehydes
C. Organochlorides
D. Hydrogen sulfide
E. Fluoride
A 37-year-old worker during a fire ended up in the area of high
CO concentration. He was delivered to a hospital in
unconscious state. Objectively: the skin of his face and hands is
crimson. Respiration rate is 20/min. ECG: alterations specific
for hypoxic myocardium. Hourly diuresis is 40 ml. Blood test:
erythrocytes - 4.5·1012/L, Нb- 136 g/L, color index - 0.9, ESR3
mm/hour, carboxyhemoglobin - 5%. What criterion allows
Objective
Fire → Carbon monoxide → Carboxyhaemoglobin
determining the severity of the patient’s condition?
A. Carboxyhemoglobin concentration
B. Respiratory disorders
C. ECG results
D. Extent of trophic disorders
E. Development of chronic renal failure
54. A patient underwent suture plication of the perforated
duodenal ulcer. On the 3rd day after the operation, he
Objective
Suture incompetence refers to the failure or
started producing a large amount of discharge from the
abdominal drain tube. The discharge contains bile and has weakness of surgical sutures to hold tissue
together adequately.
high amylase levels. What complication occurred in the
patient?
A. Hemorrhage from the ulcer
B. Suture incompetence of the ulcerative defect
C. Acute postoperative pancreatitis
D. Early postoperative adhesive obstruction
E. Acute cholecystitis
Diabetics mellitus
55. A 73-year-old woman came to the family 1. If glucose acumulates in eyes will cause retinal disorder → glucoma and
physician for one of her regular follow-up Microangiopathy → blindness.
2. Diabetic patients can’t use glucose → Switch into ketongensis→ Increased keto
examinations. Three months ago she was found to acids = Kussmaul’s respiration.
have type 2 diabetes mellitus. She was keeping to her 3. Polyuria.
4. Type 1 diabetes: beta cells of pancreas don’t secrete insulin.
diet and exercise plan and taking phytopreparations. 5. Has incomplete oxidation
On examination her fasting glucose was within the 6. Ketoacidic coma: increased ketone bodies → Aceton smell → Metabolic acidosis
range of 7.8-8.6 mmol/L, HbА1с - 7.9%. Height - 7. Hyperglycaemic / hyperosmolar coma: Glucose ~ 20 micromole/l
8. Deficiency of insulin → proteolysis → aminoacidemia
164 cm, weight - 83 kg. What blood sugar- 9. Renal diabetes: increased glucose levels in urine due to disturbance in glucose
controlling medicine should she be prescribed first in reabsorption from proximal convoluted tubules.
the course of her pharmacological therapy? Tests:
1. Blood sugar: Normal Glucose: 3.3 – 5.5. Diabetes: > 12.
A. Metformin 2. Glycated haemoglobin: the test use to detect history of diabetics (≥ 6.5).
B. Glibenclamide 3. Glucose tolerance: is measured after fasting glucose test.
Treatment:
C. Glimepiride 1. Insulin (replacment therapy).
D. Gliclazide 1. Use:
E. Insulin 1. Type 1 diabetes mellitus.
2. Inhibit the process of ketosis
3. Note: Rapid-acting insulin is used during coma.
2. Side effect: Hypoglycemia = coma (Carbohydrate starvation).
Objective 3. Antidote: Glucose or adrenaline.
Metformin is first line drug for treating type 2 2. Glibenclamide:
diabetes mellitus. 1. Use: type 2
2. Sulphonyl urease derivate
3. Stimulates generation of insulin from beta cells.
3. Metformin:
• Use: type 2 (drug of choice).
56. A 23-year-old woman came to the gynecological clinic. She
Objective
complains of pain, itching, and burning in her vulva, general
Genital herpes infection
weakness, indisposition, elevated body temperature up to • Symptoms: Painful penile, vulvar, or cervical
37.2oC, and headache. On examination in the vulva there are vesicles and ulcers with bilateral tender
multiple vesicles up to 2-3 mm in diameter with clear contents inguinal lymphadenopathy; can cause systemic
against the background of hyperemia and mucosal edema. Make symptoms such as fever, headache, myalgia
the provisional diagnosis: • Causes: HSV-2, less commonly HSV-1
A. Genital herpes infection • Treatment: acyclovir and ibuprofen.
B. Primary syphilis
C. Papillomavirus infection
D. Vulvar cancer
E. Cytomegalovirus infection
Review
During medical examination a cadet in the naval college was
detected to have a painless dense ulcer 1.5x0.5 in size in his
perianal area at the 2 o’clock position. The ulcer floor
resembles” old fat”. What is the provisional diagnosis?
A. Hard syphilitic chancre of the rectum
B. Rectal fissure
C. Rectal fistula
D. Anal cancer
E. Anal crypt suppuration Objective
Painless ulcer is in genitalia and rectum is a key
A patient complains of painless” sores” on his penis and feature of syphilis.
inguinal lymph nodes enlargement. Synthomycin emulsion that
the patient have been applying to the” sores” was ineffective.
Objectively: on the inner leaf of the foreskin there are three
closely situated rounded erosions, 0,5 cm in diameter, with
dense infiltration that can be palpated at their bases. Make the
preliminary diagnosis:
A. Primary syphilis
B. Herpes simplex (Herpes pro genitalis)
C. Candidiasis of the inner leaf of the foreskin
D. Shingles
E. Erythema multiforme
57. On ultrasound of the thyroid gland, a 47year-old woman Objective
presents with a hypoechoic node 1.6 cm in diameter with FNAB is considered the gold standard for
blurred margins and intranodular hypervascularization. The diagnosing thyroid carcinoma as spreading cancer
doctor suspects thyroid carcinoma. What method should be cells to adjacent structures is reduced
used to verify the diagnosis?
A. Fine-needle aspiration biopsy
B. Thyroid scintigraphy
C. Case monitoring
D. Determine TSH level in the blood
E. Positron emission tomography (PET)
58. A 55-year-old woman complaints of thyroid gland
enlargement that can be observed throughout the last two
years and a discomfort during swallowing. Objectively, she
has signs of hypothyroidism. The thyroid gland on
palpation is dense, non-fused with the surrounding tissues
and mobile on swallowing. The regional lymph nodes are
not enlarged. In the serum there are antithyroid antibodies
detected. What is the most likely diagnosis?
A. Hashimoto’s thyroiditis
B. Acute thyroiditis
C. Thyroid cancer
D. Endemic goiter
E. Midline cervical cyst
Thyroxin / triiodothyronine T3 T4 " #

1. From: follicular cells of thyroid gland


2. Contains: iodine
3. Function: regulate Basal metabolic rate. (‫)اﻟﺣرق‬
4. How? By process of disjunction of oxidation and
oxidative phosphorylation
5. Hyperthyroidism:
• Increase basal metabolic rate (‫= )اﻟﺣرق‬
1. Loss of weight.
2. Hyperthermia.
3. Tachycardia
4. Sweating.
5. Tremor ‫اﻟرﺟﻔﺔ‬
• Exophthalmia ‫ﺟﺣوظ اﻟﻌﯾن‬: by increasing
Glycosaminoglycans → water accumulation in the
eye.
6. Hyp0rthyroidism:
• Decrease basal metabolic rate (‫= )اﻟﺣرق‬
1. Weight gain.
2. Growth retardation.
3. Disproportional body build.
4. Bradycardia.
5. Physical and mental retardation.
7. Treatment: replacement therapy .
A 40-year-old female patient complains of having a bulge on Objective
the anterior surface of neck for 5 years. Objectively: Ps- • Bulge on the anterior surface of neck → Goiter
72/min., arterial pressure - 110/70 mm Hg, in the right lobe of • Mobile node 4x2 → nodular
thyroid gland palpation reveals a mobile node 4x2 cm in size, • Normal signs → Euthyroid
the left lobe is not palpable, the basal metabolic rate is 6%.
What is the most likely diagnosis?
A. Nodular euthyroid goiter
B. Nodular hyperthyroid goiter
C. Riedel’s thyroiditis
D. Mixed euthyroid goiter
E. Median cervical cyst
A 32-year-old woman complains of body weight loss despite her Objective
increased appetite, nervousness, and tremor of the extremities. Diffuse toxic goiter
Objectively: the skin is moist; the thyroid gland is diffusely (Graves' disease)
enlarged, painless, soft, and mobile. Blood test: increased level • Most common cause of hyperthyroidism.
of T3, T4, and thyroid-stimulating hormone (THS). What is the • Pathophysiology: Thyroid-stimulating
most likely diagnosis? immunoglobulin stimulates TSH receptors on:
A. Diffuse toxic goiter • Thyroid → hyperthyroidism, diffuse
B. Thyroid carcinoma goiter
• Dermal fibroblasts → pretibial
C. Autoimmune (Hashimoto’s) thyroiditis
myxoedema
D. Thyroid adenoma
• Orbital fibroblasts → Graves orbitopathy.
E. Diffuse nontoxic goiter • Lab Findings: ↑ T3, T4 and TSH
Objective
Tachycardia and loss of weigh are typical findings
in patients with hyperthyroid disorders like toxic
nodular goiter
A 48-year-old woman has been hospitalized due to development
of tachysystolic atrial fibrillation. She has lost 5 kg of body
Objective
weight within 2 months. On palpation there is a node in the left
Toxic nodular goiter
lobe of the thyroid gland. What pathology resulted in the • Focal patches of hyperfunctioning follicular
development of this condition? cells distended with colloid working
A. Toxic nodular goiter independently of TSH.
B. Aterosclerotic cardiosclerosis • Due to TSH receptor mutations in 60% of
C. Chronic thyroiditis cases.
D. Nontoxic nodular goiter • ↑ release of T3 and T4. Hot nodules.
E. Autoimmune thyroiditis • ‫اﻟﺜﺎﯾﺮوﯾﺪ ﺷﻐﻠﮫ ﻣﻦ دﻣﺎﻏﮭﺎ‬
59. The 5-year-old child has been ill for 2 weeks. Cough
attacks developed first and were then followed by reprises. Objective
During coughing the child’s face turns red and cervical The sever and long cough in this patient is typical
veins bulge. The cough attacks induce vomiting. X-ray for pertussis infection.
shows intensified bronchial pattern. Blood test: leukocytes-
16 x 109/L, lymphocytes-72%, erythrocyte sedimentation
rate- 4mm/hour. What is the most likely diagnosis?
A. Pneumonia
B. Foreign body
C. Pertussis
D. Obstructive bronchitis
E. Adenovirus infection Bordetella Pertussis (Whooping cough) - 100 days cough
Symptoms
1. Catarrhal stage: lasts one to two weeks and is characterized
by symptoms of an upper respiratory infection such as low-
grade fever, nasal congestion, and rhinorrhea.
2. The paroxysmal stage: lasts two to eight weeks and is
characterized by paroxysms of coughing followed by an
inspiratory whoop.
3. Convalescent stage: which may last for weeks to months and
is characterized by a subsiding cough.
The 5-year-old child has been ill for 2 weeks. Cough attacks Objective
developed first and were then followed by reprises. During The sever and long cough in this patient is typical
coughing the child’s face turns red and cervical veins bulge. The for pertussis infection.
cough attacks induce vomiting. X-ray shows intensified
bronchial pattern. Blood test: leukocytes 16 · 109/L ,
lymphocytes - 72%, erythrocyte sedimentation rate - 4
mm/hour. What is the most likely diagnosis?
A. Pertussis
B. Obstructive bronchitis
C. Pneumonia
Bordetella Pertussis (Whooping cough) - 100 days cough
D. Adenovirus infection
Symptoms
E. Foreign body 1. Catarrhal stage: lasts one to two weeks and is characterized
by symptoms of an upper respiratory infection such as low-
grade fever, nasal congestion, and rhinorrhea.
2. The paroxysmal stage: lasts two to eight weeks and is
characterized by paroxysms of coughing followed by an
inspiratory whoop.
3. Convalescent stage: which may last for weeks to months and
is characterized by a subsiding cough.
60. A 27-year-old man complains of pain in his leg joints,
purulent discharge from the eyes, and painful burning sensations
Objective
during urination. Disease onset was acute. He has a history of
influenza. The patient smokes and drinks alcohol in excess. In • Chlamydia is a sexually transmitted infection
his line of work he is often away on business trips. What is the (STI) caused by the bacterium Chlamydia
most likely etiological factor of this disease? trachomatis.
A. Chlamydia • Male symptoms:
B. Adenovirus • Pain or burning sensation when urinating.
C. Streptococci • Eye inflammation and discharge.
D. Staphylococci • Discharge from the urethra, which may be
E. Candida clear, white, or yellow in color.
• Pain or swelling in the testicles.
• Female symptoms:
• Vaginal discharge, which may be clear,
white, or yellow in color
• Dysuria: Pain or burning sensation when
urinating
• Dyspareunia: Pain during sexual
intercourse
• Abdominal pain or pelvic pain
• Intermenstrual bleeding: Bleeding
between periods or after sex
61. A 22-day-old infant developed subcutaneous red nodes from Objective
1.0 to 1.5 cm in size on the scalp; later the nodes suppurated. • Pseudofurunculosis is a purulent lesion of
Temperature increased up to 37.7oC, intoxication symptoms sweat glands that develops when staphylococci
appeared, regional lymph nodes enlarged. Complete blood or other pathogens enter them.
count: anemia, leukocytosis, neutrocytosis, increased ESR. • Pseudofurunculosis mainly affects newborns
What diagnosis can be made? and children under one year old
A. Pseudofurunculosis • Clinical dermatology refers pseudofurunculosis
to a group of purulent-inflammatory skin
B. Pemphigus
diseases — pyoderma. The most common
C. Vesiculopustulosis
infectious agent in pseudofurunculosis is
D. Scalp phlegmon
Staphylococcus aureus.
E. –
62. After playing with “mosaics”, a two-year-old child
suddenly developed cough, stridorous respiration, urges to
vomit, and cyanosis against the background of relative
stomatic health. What should the doctor suspect first when
examining the child?
A. Pertussis
B. Acute laryngotracheitis
C. Pneumonia
D. Foreign body aspiration
E. Acute obstructive bronchitis
A 5-year-old child was brought to the ENT department by an Objective
ambulance. The child presents with cough and difficult If a child shows symptoms of airway obstruction
respiration. From the patient’s history it is known that the child after playing with a toy, it indicates that a foreign
was playing with a toy construction set, when suddenly started object may have caused the obstruction.
coughing and developed labored breathing. Examination detects
periodical cough, labored expiration, and respiratory lag in the
left side of the child’s thorax. Auscultation: diminished
respiration on the left. Percussion: tympanitis. X-ray shows a
displacement of the mediastinal organs to the right. Make the
diagnosis:
A. A foreign body in the left bronchus, valvular
bronchostenosis
B. A foreign body in the right bronchus, valvular
bronchostenosis
C. A foreign body in the trachea
D. A foreign body in the left bronchus, complete
bronchostenosis
E. A foreign body in the right bronchus, partial bronchostenosis
63. A 45-year-old woman complains of increasing body Objective
weight throughout the last year. Examination revealed • Findings →
moon face syndrome, brittle hair, hirsutism, stretch marks • Cushin syndrome →
on the abdomen, and disproportionally thin limbs. The • Abnormal accumulation of fat depositions
patient’s height is 162 cm, her body weight is 94 kg, her of neck and abdomin →
body mass index is 35.8 kg/m2. What type of obesity it? • Dysplastic
A. Alimentary-constitutive
B. Gynoid
C. Dysplastic
D. Cerebral
E. Android
A 32-year-old patient complains of excessive appetite, excess
Shape of obesity
weight, dyspnea during physical exertion. There are fat deposits • Android/apple/abdominal/male obesity:
in the area of abdomen and shoulder girdle. The skin is pale- accumulation of fat above the waist. waist
pink, adult male pattern of hair distribution is observed on the
circumference of 100 or more.
torso, no stretch marks. Heart rate is 90/min., BP is 120/80 mm • Gynoid/pear/glueal femoral/femal obesity:
Hg, body build index equals 35. Blood sugar is 4,9 mmol/l, accumulation of fat below the waist.
cholesterol is 6,2 mmol/l. On ophthalmoscopy: fundus of the
eye without changes. What provisional diagnosis can be made?
A. Primary alimentary constitutive obesity, android type
B. Primary alimentary constitutive obesity, gynoid type
C. Secondary hypothalamic obesity
D. Secondary neuroendocrine obesity
E. Secondary endocrine hypo-ovarian obesity
Objective
• Alimentary Constitutive Obesity: excessive
A 13-year-old girl has 30% of excessive body mass, she started appetite → overeating and a lack of physical
to gain weight at the age of 3. She has a family history of activity.
obesity. Her height and sexual development are normal for her • Hypothalamic Obesity: hypothalamus damage
age. The appetite is excessive. She complains of periodical → dysregulation of satiety or hanger centres →
excessive appetite and obesity.
headaches. Blood pressure - 120/80 mm Hg. Subcutaneous fat is
• Adrenal Obesity: overproduction of cortisol
evenly distributed, she has no stretch marks. There is juvenile
(Cushing's syndrome) → excessive appetite and
acne on her face. What type of obesity is it?
obesity.
A. Alimentary constitutive obesity • Hypothyroid Obesity: underactive thyroid gland
B. Hypothalamic obesity → slow metabolism and weight gain
C. Adrenal obesity • Hypothalamic Syndrome of Puberty: rare
D. Hypothalamic syndrome of puberty genetic disorder called Prader-Willi syndrome
E. Hypothyroid obesity → insatiable hunger and a slow metabolism. It
typically presents in childhood → severe
obesity.
A 24 year old patient complained about putting on weight, Shape of obesity
limosis. Objectively: the patient’s constitution is of • Android/apple/abdominal/male obesity:
hypersthenic type, body weight index is 33,2 kg/m2, waist accumulation of fat above the waist. waist
circumference is 100 cm. Correlation of waist circumference to circumference of 100 or more.
the thigh circumference is 0,95. What is the most probable • Gynoid/pear/glueal femoral/femal obesity:
diagnosis? accumulation of fat below the waist.
A. Alimentary constitutional obesity ofthe I stage, abdominal
type
B. Hypothalamic Itsenko-Cushing obesity of the II stage,
gynoid type
C. Alimentary constitutional obesity of the III stage, gynoid
type
D. Alimentary constitutional obesity of the II stage, abdominal
type
E. Hypothalamic Itsenko-Cushing obesity of the I stage,
abdominal type
Types of obesity
• Primary:
A child is 10 years old. The weight is 46 kg. Since birth the • Reason for obeisity: Life style,
child has been gaining excessive weight. The parents are exogenic constitutive, constitutive.
fullbodied. The child has undergone the following tests: • Treatment: Exercise and reducing diet
carbohydrate tolerance, level of 17-ketosteroids, blood and
electrolytes, US of adrenal glands, cranium X-ray. The tests • Secondary:
revealed no pathologies.The diagnosis of exogenic constitutive • Reason for obeisity: Endocrine
obesity has been made. What direction of therapy should be (hypothyroidism or cushing) cerebral
prioritized? obesity (brain tumor).
A. Reducing diet and exercise • Treatment: Treat underlying pathology.
B. Sanatorium-and-spa treatment
C. Anorectic drugs Shape of obesity
D. Dehydration therapy • Android/apple/abdominal/male obesity:
E. ”Fat-burning” methods accumulation of fat above the waist. waist
circumference of 100 or more.
• Gynoid/pear/glueal femoral/femal obesity:
accumulation of fat below the waist.
A 30-year-old woman complains of increased body weight and Objective
problems with physical exertion. Her parents are of increased 1. Weight is 87 kg and height is 165 cm →
body weight as well; typical meals in their family are high in 2. BMI = BMI = 87 / (1.65)^2 = 31.93 →
fats and carbohydrates. Objectively her blood pressure is 3. Obesity class I : BMI between 25 and 29.9 →
135/80 mm Hg, pulse is 89/min., weight is 87 kg, height is 165 4. Prevention measures, such as dietary treatment
cm. The patient’s skin is clear, with even distribution of and graduated exercise.
subcutaneous fat; the thyroid gland is not enlarged; there are no
menstrual cycle disturbances. What obesity prevention methods
would be the most advisable in this case?
A. Dietary treatment, graduated exercise
B. Intensive training regimen
C. Gastroplasty or gastrojejunal shunt Body mass index (BMI)
D. Inhibitors of gastrointestinal lipases • What is it? is a measure of body fat based on a
E. Anorectic drugs person's weight and height
• Calculation = weight (kg) / (height (m))^2
• Categories:
1. Underweight: BMI below 18.5
2. Normal weight: BMI between 18.5 and 24.9
3. Overweight: BMI between 25 and 29.9
4. Obesity class I: BMI between 30 and 34.9
5. Obesity class II: BMI between 35 and 39.9
6. Obesity class III: BMI 40 or greater
64. A woman came to the gynecologist to plan her
pregnancy. She was advised to increase her intake of the Objective
One of the cuases of nural tube defects is folic
products rich in folic acid, particularly soy beans, bread
made of coarsely ground flour, fruits, leafy green acid deficiency
vegetables. Such changes in her diet will work toward the
prevention of:
A. Rickets
B. Polyhydramnios
C. Non- closure of the neural tube in the fetus
D. Iron- deficiency anemia in the pregnant woman
E. Non-closure of the fontanel
65. During examination of a patient, the doctor detected in Objective
him disorders of the eyes (hemeralopia, Bitot’s spots), skin Findings of skin, eyes and GIT defects and
and skin appendages, mucosa, ang gastrointestinal tract. He Prasad’s syndrome are typical for Zinc deficiency.
was provisionally diagnosed with Prasad’s syndrome. What
causes the development of this pathology?
A. Manganese deficiency
B. Vanadium deficiency
C. Zinc deficiency
D. Iron deficiency
E. Copper deficiency
66. What should be prescribed as secondary prevention Objective
drugs for a patient with atrial fibrillation after an ischemic • Patient with atrial fibrillation are at risk of →
stroke caused by cardiac embolism? • Developing thrombosis →
A. Oral anticoagulants • Second Prevention →
B. Aspirin or clopidogrel • Oral antigoaulants.
C. Beta blockers
D. Nootropics
E. Calcium antagonists
Vitamin B1(thiamine/ TPP)
67. During regular medical examination a lyceum 1. Function:
student presents with signs of cheilitis that manifests • Is the cofactor for transferases enzymes.
as epithelial maceration in the area of lip seal. The • Cofactor for pyruvate dehydrogenase / decarboxylase.
2. Deficiency:
lips are brightred, with single vertical cracks covered 1. Wernicke-Korsakoff syndrome (Common with alcoholics)
with brown-red scabs. These clinical signs are most 2. Beriberi: Muscle weakness and mental disability.
likely caused by insufficient content of the following Vitamin B2 (riboflavin)
in the diet: Function: Component of FAD.
A. Riboflavin Deficiency: Cheilosis and Corneal vascularization.
Vitamin B3 (Niacin / Nicotinamide/ PP)
B. Ascorbic acid
1. Function: Makes the cofactor NAD.
C. Retinol 2. Deficiency: Pellagra (4D disease): diarrhea, dementia, symmetric
D. Thiamine dermatitis and death.
E. Calciferol 3. Note: Hartnup disease causes less absorption of tryptophan which
means B3 deficiency.
Biotin B7
• Function:
1. Acetyl-CoA carboxylase
2. Pyruvate carboxylase
3. Propionyl-CoA carboxylase
• Deficiency: Alopecia
• Causes of deficiency:
• Excessive consumption of raw egg contains avidin → binds
biotin in the intestinal lumen → inhibition of biotin resorption.
68. A 25-year-old woman was brought into the Incomplete abortion
gynecological department with profuse bloody discharge • What is it? type of abortion where some of the
from her genital tracts. She is 12 weeks pregnant, the fetal or placental tissue remains in the uterus
pregnancy is planned. Within the last 3 days she was after the termination of the pregnancy.
experiencing pains in her lower abdomen that eventually • Findings:
started resembling cramps, she developed bleeding. Her • Heavy bleeding, often with large blood
skin is pale, pulse - 88/min., blood pressure - 100/60 mm clots.
Hg, body temperature - 36.8oC. Vaginal examination: the • Abdominal or pelvic pain
• Fever (infection)
uterus size corresponds with 11 weeks of pregnancy, the
• Foul-smelling vaginal discharge
cervical canal allows inserting 1 finger and contains
• Diagnosis: physical examination, ultrasound
fragments of the fertilized ovum, the discharge is bloody imaging, and blood tests
and profuse. What is the most likely diagnosis? • Treatment: Dilation and curettage (D&C), a
A. 12-week pregnancy, spontaneous abortion in progress surgical procedure to remove the remaining
B. 12-week pregnancy, threatened spontaneous abortion tissue from the uterus.
C. Disturbed menstrual cycle, hyperpolymenorrhea
D. Disturbed menstrual cycle, amenorrhea
E. Full-term pregnancy, term labor
69. A 20-year-old man suffers from headache, general
weakness, and face edema that aphe had a case of
tonsillitispears in the morning. 18 days earlier. Objectively,
his skin is pale, there are edema under his eyes. Hear rate is
60/min, blood pressure is 185/100 mmHg. The sign of
costovertebral angle tenderness (punch sign in the lumbar
region) is negative. Urinalysis: color of “meatslops”,
protein - 4.5 g/, altered erythrocytes- 40-45 in the vision
field, leukocytes- 5-6 in the vision field. 24-hour diuresis is
400 mL. What is the most likely diagnosis?
A. Acute pyelonephritis
B. Renal amyloidosis
C. Acute glomerulonephritis
D. Systemic lupus erythematosus
E. Urolithiasis
70. A 19-year-old woman complains of severe pain in the Hidradenitis suppurativa
axillary crease. Condition onset occurred a week ago after her • chronic inflammatory skin condition
swimming in a cold river and epilation. The next day a painful characterized by recurrent painful nodules,
”boil” appeared. The ”boil” was increasing in size every day abscesses, and draining sinus tracts in the
and became a plum-sized tumor. Upon examination there are apocrine gland-bearing areas, such as the
nodular conical growths joined together detected, the skin armpits, groin, and buttocks. It is a debilitating
covering them is bluish-red in color. Some nodules have condition that can significantly impact quality
fistulous openings producing thick purulent mass. Body of life.
temperature is 38,5oC, general malaise. What is the most likely • Symptoms of HS typically include painful,
diagnosis? inflamed bumps or nodules in the affected
A. Hydradenitis areas, which may break open and drain pus or
B. Carbuncle blood. These lesions may be accompanied by
C. Cutaneous tuberculosis a foul odor and can cause scarring and skin
D. Necrotizing ulcerative trichophytosis changes in the affected area. HS can also be
E. Pyoderma chancriformis associated with significant pain and
discomfort, as well as social isolation and
depression.
Review Objective
A 64-year-old man suddenly sensed pain in his occipital area, ACE inhibitors (ex, captopril) is one of the first
dizziness, general weakness. He has a 15-year-long history of line therapy for blood pressure.
hypertension. Objectively the skin and mucosa are of normal
color. Auscultation reveals vesicular respiration across the lung
surface. At the cardiac apex the I heart sound is weakened, the
II heart sound is accentuated over the aorta. Pulse is 84/min.,
blood pressure is 180/100 mm Hg. Other body organs and
systems are unaffected. What drug should be prescribed in the
first place?
A. Captopril
B. Ramipril
C. Urapidil
D. Perindopril
E. Amlodipine
71. A 38-year-old woman has been suffering from Objective
glomerulonephritis for 20 years. For approximately16 years Edema and refractor hypertension → loop
she has been presenting with progressin grenal (Torasemide)
parenchymal arterial hypertension that became refractory
and accompanied by leg edemas. She receives a
combination of 100mg losartan and 20mg lercanidipine
with insufficient antihypertensive effect. What medicine
can she be recommended for intensification of the
antihypertensive effect of her therapy?
A. Doxazosin
B. Bisoprolol
C. Torasemide
D. Lisinopril
E. Urapidil
72 For 20 years the role of excessive weight in ischemic Objective
heart disease development among the working age male • What is it? Compares a group with a given
production over 40 was studied. It was determined that exposure or risk factor to a group without such
overweight men developed ischemic heart disease more exposure.
often. What type of epidemiological study is it? • Example:
A. Cohort study • Overweight men developed ischemic heart
B. Experimental study disease more often than men with normal
C. Case report weight.
D. Case-control study • Smokers had a higher risk of developing
E. Vase series report COPD than people who don’t smoke.
73. When planning treatment of a patient, it was decided to Level A
use a medicine with evidence level A. what trials produce • What is it? The highest level of evidence, based
the evidence that allows to classify the medicine level as on well-designed randomized clinical trials
A? (RCTs) with consistent positive outcomes.
A. Case-control studies • Examplee:
B. Expert consensus • Use of statin medications (e.g., atorvastatin,
C. Data obtained from one randomized clinical trial simvastatin) for the prevention of
D. Data obtained from several randomized clinical trials cardiovascular events in patients with high
E. Data obtained from many non-randomized trials cholesterol. This recommendation is based
on multiple randomized clinical trials that
consistently demonstrate the effectiveness
of statins in reducing the risk of heart
attacks, strokes, and other cardiovascular
events.
74. A newborn has a round red formation in the suprapubic Objective
region. Examination shows that urine is being discharged in Presentation of red formation in the suprapubic
pulses from the two orifices located in the lower part of this and urine discharge is typical for bladder
formation. Name this developmental anomaly: exstrophy.
A. Bladder agenesis
B. Urachal cyst
C. Vesico-umbilical fistula
D. Bladder diverticulum
E. Bladder exstrophy
75. A patient 1 year had a Q wave myocardial infarction of Objective
the posterior wall of the left ventricle. For the last 2 weeks • Presentation of atrial fibrillation and
he has been suffering from daily attacks of atrial fibrillation bradycardia episodes →
and bradycardia episodes, accompanied by bouts of • Electrical activity defect →
vertigo. What tactics is the most advisable in this case? • Pacemaker implantation.
A. Prescription of amiodarone
B. Prescription of procainamide
C. Prescription of bisoprolol
D. Pacemaker implantation
E. Prescription of digoxin
76. On the 5th day after giving birth a post parturient Prevention of Mastitis and Nipple Complications
woman complains of a pain in her left mammary gland and • Feeding on demand: Breastfeeding on demand which
body temperature up to 38.1C. Examination shows that her means allowing the baby to nurse whenever they show
mammary gland is enlarged and painful on palpation, the signs of hunger helps ensure effective milk removal from
nipple is edematous and has fissures, the upper external the breast. This helps prevent milk stasis and engorgement,
quadrant of the gland is hyperemic. Name the measures reducing the risk of mastitis.
that would have prevented the development of this • Expression of breast milk: If the baby is unable to
complication in the patient effectively nurse or if there is excess milk production,
A. Feeding on demand, expression of breast milk, expressing breast milk manually or using a breast pump
prevention of nipple fissures can help maintain milk flow and prevent engorgement.
B. Stop breastfeeding when fissures appear • Prevention of nipple fissures: Proper latch and positioning
C. Feeding no longer that 10 minutes through an overlay during breastfeeding are essential to prevent nipple
D. Constant expression of breast milk damage and fissures. Ensuring a deep latch, with the baby's
E. Feeding on schedule mouth covering a significant portion of the areola, can help
prevent nipple trauma.
• Avoid stopping breastfeeding when fissures appear: It is
important to continue breastfeeding even if nipple fissures
develop, as stopping breastfeeding can lead to further
complications and exacerbate engorgement. Addressing
the underlying issue, such as improving latch or seeking
assistance from a lactation consultant, can help heal the
fissures while continuing to breastfeed.
77. A 34-year-old multipara was brought to the labor ward
Objective
with regular labor activity. Her pelvic size 26- 29-32-22
• According to krok center, findings here are normal
cm. Vaginal examination shows 6 cm cervical dilation, the
despite presence of breech presentation →
amniotic sac is unbroken. The fetus is in the breech
• Delivery through the natural birth canal.
presentation, with buttocks pressed to the entrance into the
lesser pelvis. The promontory cannot be reached, no
exostoses. Feta heart rate is 140/min, expected fetal weight
is 2800g. What labor tactics should be chosen?
A. External obstetric version of fetus
B. Fetal extraction from the pelvic end
C. Classic combined external-internal version of the fetus
D. Delivery through the natural birth canal
E. Urgent cesarean section
A parturient woman is 23 years old. Internal obstetric
examination shows the uterine cervix to be completely open.
Fetal bladder is absent. Cephalic presentation is observed in the
plane of the small pelvic outlet. Sagittal suture is at the
longitudinal section of the small pelvic outlet, small fontanel is
situated closer to the uterus. What cephalic position will the
newborn have during birth in this case?
A. Minor oblique lie
B. Longitudinal lie
C. Transverse lie
D. Medium oblique lie
E. Major oblique lie
Review Acute deep vein thrombosis
A 74-year-old patient was delivered into admission room with • What is it is? formation of a blood clot in the
clinical presentations of acute deep vein thrombosis of the shin. deep veins of the body, most commonly in the
What symptom is the most typical of this pathology? legs.
A. Homans’ sign • Findings: unilateral leg swelling, pain,
B. Rovsing’s sign Homans’ sign and tenderness.
C. Courvoisier’s sign • Diagnosis: ultrasound imaging or venography.
D. Mayo-Robson’s sign • Treatment: anticoagulation therapy with
E. Grey Turner’s sign heparin and warfarin, thrombolytic therapy.
78. A 65-year-old man underwent a left hemicolectomy Objective
due to a malignant tumor in the descending colon. On the • Thrombosis, specifically deep vein thrombosis
4th day after the surgery, he developed pain and edema in (DVT) and pulmonary embolism (PE) is a
his left shin. The Homans sign is positive on the left. What common complication that can occur after
postoperative complication developed in this patient? surgery.
A. Acute postoperative thrombosis of the deep veins in the • Risk is high due to: immobility, tissue injury, and
left shin changes in blood flow patterns.
B. Postoperative allergic reaction • This patient shows signs of DVT: pain and
C. Acute postoperative lymphangitis of the left shin edema, and positive Homans sign.
D. Acute disturbance of the cerebral blood flow
E. Acute postoperative thrombosis of the popliteal artery
on the left
79. A 19-year-old young man was diving and hit his head Objective
on the bottom of the pool. He complains of pain in the • Highlighted presentations → Uncomplicated
neck, head movements are limited and painful. During cervical fracture.
examination his head is bowed forward and to the right and • Complicated fructrues will show signs of spinal
the patient supports it with his hands. Palpation detects cord and nerve damage.
tense neck muscles and protruding spinous process of the
IV cervical vertebra (C4). When pressure is applied to this
process and to the head (axial load), the pain intensifies.
Make the provisional diagnosis:
A. Cervical contusion
B. Uncomplicated cervical fracture
C. Neck muscle injury
D. Spinal root injury
E. Complicated cervical fracture
80. A 58-year-old woman has type 2 diabetes mellitus that Objective
is compensated with diet and metformin. She prepares for • Type 2 diabetes mellitus can take insulin in 2
cholecystectomy. Objectively, her height is 164 cm, weight cases:
is 90 kg, heart rate is 72/min, blood pressure is 130/80 1. When the pancrease insulin production
mmHg. Her abdomen is soft, painful in the right subcostal becomes insufficient.
region. The liver is not enlarged. Fasting glucose-6.2 2. When the patient is going into stress that
mmol/L. Glycated hemoglobin- 6.5%. what further tactics can raise blood glucose levels like surgery.
of sugar lowering therapy should be chosen in this case?
A. Prescription of a short-acting insulin
B. Prescription of an insulin mixture
C. Continue the present scheme of therapy
D. Prescription of a long-acting insulin
E. Prescription of glurenorm (gliquidone)
A 72-year-old woman suffers from diabetes mellitus type II, lactic acidosis
concomitant diseases are stage II hypertension and stage IIB • What is it? Serious condition →
heart failure. She takes metformin. Hypertensic crisis had hyperlactatemia → decrease blood pH
occurred the day before, after which the patient developed (academia) → organ dysfunction and failure
extreme weakness, myalgias, thirst, dry mouth, polyuria. BP is → Coma → Death &.
140/95 mm Hg, heart rate is 98/min., no edemas or smell of • ↑ Risk factors:
acetone detected. What measures should be taken to prevent 1. Heart failure: less blood supply → less
development of comatose state in the patient? oxygen → anaerobic glycolysis → lactic
A. Stop metformin, prescribe short-acting insulin acidosis.
B. Double the dosage of metformin 2. Drugs: metformin
C. Apply hypotonic solution of sodium chloride 3. Hypertension: organ ischemia → less
D. Additionally prescribe long-acting insulin oxygen → anaerobic glycolysis → lactic
E. Prescribe glibenclamide acidosis.
4. Dehydration (ex, polyuria): decreased
Objective blood volume → less oxygen →
• Lactic acidosis is a known side effect of anaerobic glycolysis → lactic acidosis.
Metformin. In case the there is a high risk of 5. Exercise: increased lactate production in
developing lactic acidosis as this patient, the muscles → myalgia.
Metformin should be stopped to prevent further 6. Others: diabetic ketoacidosis, infections,
production of lactic acid. liver disease, and alcoholism.
• Also, prescription of short-acting insulin will
prevent hyperglycemia and inhibit production of
ketoacids to further prevent lactic acidosis.
81. hours after eating unknown mushrooms, a 28-year-old
Acute liver failure
man sensed a decrease in his mobility and deterioration of
• Acute liver failure is a serious and potentially life-
his ability to focus. This condition was then followed by a
threatening condition that occurs when the liver suddenly
state of agitation and agression. On examiantion he is loses its ability to function properly.
disoriented and his speech is illegible. 4 hours later he • When the liver fails, it can detoxify blood from ammonia
developed fetor hepaticus and lost his consciousness. toxins.
What syndrome can be observed in this patient? • Ammonia toxins will accumulate in the brain leading to
A. Acute hepatic failure these symptoms.
B. Hepatolienal syndrome • Headache, nausea, recurrent vomiting, memory lapses,
C. Portal hypertension flapping tremor of her hands.
D. Cholestatic syndrome • Fetor hepaticus: sweet, musty, or moldy odor caused by
E. Cytolytic syndrome accumulation of ammonia in blood.
• Causes:
• Viral hepatitis.
• Toxins: ex, mushroom toxins
• Medication
• Ischemic liver injury.
A woman undergoing in-patient treatment for viral
Objective
hepatitis type B developed headache, nausea, recurrent
Liver failure
vomiting, memory lapses, flapping tremor of her hands, The patient is infected with viral hepatitis which is
and rapid pulse. Sweet smell from her mouth is detected. is a typical cause of acute liver failure.
Body temperature is 37.6oC, heart rate is 89/min. What
complication developed in the patient?
A. Acute liver failure Acute liver failure
• Acute liver failure is a serious and potentially life-
B. Ischemic stroke
threatening condition that occurs when the liver suddenly
C. Gastrointestinal hemorrhage
loses its ability to function properly.
D. Hypoglycemic shock
• When the liver fails, it can detoxify blood from ammonia
E. Meningoencephalitis toxins.
• Ammonia toxins will accumulate in the brain leading to
these symptoms.
• Headache, nausea, recurrent vomiting, memory lapses,
flapping tremor of her hands.
• Fetor hepaticus: sweet, musty, or moldy odor caused by
accumulation of ammonia in blood.
• Causes:
• Viral hepatitis.
• Toxins: ex, mushroom toxins
• Medication
• Ischemic liver injury.
Reiview Peritonitis
On the day 4 after the cesarean section a woman developed • What is it? is a serious medical condition that
fever with body temperature up to 39oC and abdominal pain. occurs when there is inflammation and infection
Pulse - 104/min. She vomited twice. The patient is sluggish, her of the peritoneum.
tongue is dry and has gray coating. The abdomen is distended. • Etiology: perforation in the gastrointestinal tract
Signs of peritoneal irritation are positive in all segments. (such as a ruptured appendix or a perforated
Peristalsis cannot be auscultated. No passage of gas occurs. ulcer), a traumatic injury to the abdomen, or a
complication of abdominal surgery ex,
Uterine fundus is located at the level of the navel. The uterus is
caesarean section
painful on palpation. The discharge is moderate and contains
• Findings: abdominal pain or tenderness,
blood and pus. What is the most likely diagnosis?
abdominal distension, fever and chills, nausea
A. Diffuse peritonitis and vomiting, anorexia, gastrointestinal
B. Metroendometritis dysfunction manifesting as diarrhea or
C. Progressive thrombophlebitis constipation, fatigue or weakness, tachycardia,
D. Pelvic peritonitis and hypotension.
E. Parametritis
82. On the 3rd day after the artificial abortion the woman Objective
was hospitalized into the gynecological department in a Positive signs of peritoneal irritation →
severe condition with signs of intoxication, abdominal Pelviperitonitis.
pain, and purulent discharge from the vagina. Objectively,
the patient’s condition is severe, her body temperature is
38.8°C, pulse is 100/min, blood pressure is 110/70 mmHg,
the uterus is soft, the uterine fundus is located at the level
of navel, there are positive signs of peritoneal irritation.
What is the most likely diagnosis? Peritonitis
A. Uterine perforation
• What is it? is a serious medical condition that
B. Acute metro endometritis
occurs when there is inflammation and infection
C. Ectopic pregnancy
of the peritoneum.
D. Pelviperitonitis
• Etiology: perforation in the gastrointestinal tract
E. Acute suppurative salpingo-oophoritis
(such as a ruptured appendix or a perforated
ulcer), a traumatic injury to the abdomen, or a
complication of abdominal surgery ex,
caesarean section
• Findings: abdominal pain or tenderness,
abdominal distension, fever and chills, nausea
and vomiting, anorexia, gastrointestinal
dysfunction manifesting as diarrhea or
constipation, fatigue or weakness, tachycardia,
and hypotension.
Obstetrical paralysis
83. A full-term newborn (born with the body weight
Klumpke palsy
of 3900 g at gestational age of 39 weeks) on the first
• Erb-Duchenne palsy, is a type of lower brachial plexus injury that
day of his life developed respiratory disturbances:
dyspnea, arrhythmic respiration, cyanosis attacks. On occurs during childbirth or trauma and effects the upper arm.
• How the injury happens?
examination there is paradoxical respiration observed
1. Stretching of the baby’s arm and shoulder during delivery →
and left side of the chest lags behind in the act of
2. Damage to lower brachial plexus nerves →
breathing. On auscultation the respiration is
3. Paralysis of weakness or paralysis of the hand and forearm,
weakened in the lungs on the left. Neurologist
diagnosed the patient with left- sided Erb-Duchenne along with a loss of sensation in the affected area.
• Findings: Grasping defect.
palsy. Complete blood count shows no changes. What
is the most likely diagnosis? Erb-Duchenne palsy
A. Left-sided diaphragm paresis • Erb-Duchenne palsy, is a type of upper brachial plexus injury that
B. respiratory distress syndrome
occurs during childbirth or trauma and effects the upper arm.
C. Transient tachypnea of the newborn • How the injury happens?
D. Congenital pneumonia 1. Stretching of the baby's neck and shoulder during delivery or
E. Left-sided pneumonia trauma →
2. Damage to upper brachial plexus nerves →
3. Paralysis of the shoulder and upper arm, along with a loss of
sensation in the affected area (hands is not involved).
4. Diaphragm paresis on the damaged side → respiratory
disturbance
• Findings:
• Inward rotation of shoulder
• Arm extended and pronated
84. A 17-year-old girl complains of a pain in her knee and
ankle joints and body temperature up to 39c. 2 weeks ago,
she had a case of acute tonsillitis. Objectively, her joints are
swollen, sharply painful and their mobility is reduced. On
the skin of her truck and limbs there are barely visible
circle-shaped pale pink spots. Heart rate is 95/min, blood
pressure is 90/60 mmHg, heart sounds are weakened, there
is a soft systolic noise over the apex. Make the provisional
diagnosis:
A. Acute rheumatic fever
B. Rheumatoid arthritis
C. Systemic scleroderma
D. Erythema nodosum
E. Reactive arthritis
85. A 30-year-old woman made an appointment with the family Objective
doctor for scheduled vaccination of her 2-year-old child. What • Primary healthcare:
type of healthcare provides such medical services? • Primary healthcare refers to the first point
A. Primary healthcare of contact that individuals have with the
B. Emergency aid healthcare system, where they receive a
C. Secondary healthcare wide range of basic healthcare services
D. Tertiary healthcare such as preventive care, health education,
E. Palliative care routine check-ups, and vaccinations.
• Primary healthcare is often provided by
general practitioners and family doctors.
• Secondary healthcare: specialized ex, internal
medicine.
• Tertiary healthcare: highly specialized ex,
neurosurgery and cancer treatment.
• Palliative care: improving the quality of life
and relieving the suffering of individuals who
are living with serious or life-limiting illnesses.
Objective
• Primary healthcare:
• Primary healthcare refers to the first point
of contact that individuals have with the
A 52 year old man came to see his family physician healthcare system, where they receive a
complaining of pain in the chest. After taking history and wide range of basic healthcare services
performing physical exam the doctor decided to direct the such as preventive care, health education,
patient to cardiologist for a consultation. What level of medical routine check-ups, and vaccinations.
care is being proposed to the patient? • Primary healthcare is often provided by
A. Secondary healthcare general practitioners and family doctors.
B. Emergency healthcare • Secondary healthcare: specialized ex, internal
C. Primary healthcare medicine.
D. Tertiare healtcare • Tertiary healthcare: highly specialized ex,
E. Palliative care neurosurgery and cancer treatment.
• Palliative care: improving the quality of life
and relieving the suffering of individuals who
are living with serious or life-limiting illnesses.
Review
An 8-year-old girl periodically has sudden short-
term heart pains, sensation of chest compression,
epigastric pain, dizziness, vomiting. Objectively:
the patient is pale, respiratory rate - 40/min.,
jugular pulse is present. Ps- 185/min., of poor
volume. BP- 75/40 mm Hg. ECG taken during an
attack shows ectopic P waves, QRS wave is not
deformed. At the end of an attack a compensatory
pause is observed. The most likely cause of the
attack is:
A. Paroxysmal atrial tachycardia
B. Sinus tachycardia
C. Paroxysmal ventricular tachycardia
D. Complete AV-block
E. Atrial fibrillation
86. A 45-year-old man with a history of myocardial Objective
infarction developed a brief attack of palpitation, • ECG findings (extended QRS complex) and
accompanied by the sensations of lack of air, fear, and drop in blood pressure indicates →
vertigo. His blood pressure is 90/60mmHg. ECG during the • Abnormal ventricular depolarization →
attack shows extended QRS complex (0.13 seconds) with • Paroxysmal ventricular tachycardia
heart rate of 160/min, discordant shift of ST segment and T
wave, dissociation of atrial and ventricular rhythm. What
disturbances of cardiac rhythm it is?
A. Paroxysmal supraventricular tachycardia
B. Ventricular fibrillation
C. Frequent ventricular extrasystole
D. Atrial fibrillation
E. Paroxysmal ventricular tachycardia
Treatment for Paroxysmal supraventricular
tachycardia

Adenosine triphosphate may be expected to convert


which of the following arrhythmias to sinus
rhythm?
A. Paroxysmal supraventricular tachycardia
B. Paroxysmal ventricular tachycardia
C. Atrial fibrillation
D. Atrial flutter
E. Ventricular fibrillation

ECG revealed the following in a 10year-old child:


sharp acceleration of the heart rate - 240/min., P
wave overlaps with T wave and deforms it,
moderate lengthening of PQ interval, QRS complex
is without alterations. What pathology does this
child have?
A. Paroxysmal atrial tachycardia
B. Atrial hypertrophy
C. Ventricular hypertrophy
D. WPW syndrome
E. Extrasystole
Stages of delivery
1. First Stage: begins with the onset of uterine contractions and continues until the cervix
is fully dilated to 10 centimetres. The first stage is divided into two phases:
1. Latent Phase:
1. Mild contractions that occur every 5-20 minutes.
2. The cervix begins to soften and gradually dilate up to around 6 cm.
3. This phase can last for several hours and is usually the longest phase of
labor.
2. Active Phase:
1. Cervix dilates more rapidly, up to 10 cm.
2. Contractions become stronger, more frequent, and more painful, occurring
every 2-3 minutes and lasting up to 60 seconds.
3. This phase typically lasts between 3-8 hours.
2. Second Stage:
1. This stage begins when the cervix is fully dilated and ends with the birth of the
baby.
2. Characterized by strong contractions that push the baby through the birth canal.
3. This phase typically lasts between 20 minutes to 2 hours.
3. Third Stage:
1. This stage begins with the delivery of the baby and ends with the delivery of the
placenta.
2. After the baby is born, the uterus continues to contract, which helps to separate
the placenta from the uterine wall.
3. The placenta is then delivered through the vagina. This stage usually takes about
10-20 minutes.
87. A 20-year-old woman, gravida 2, para 1 has been First Stage
in labor for 4 hours. Her condition is satisfactory. begins with the onset of uterine contractions and continues until the
Moderately painful contractions occur every 3 cervix is fully dilated to 10 centimetres. The first stage is divided
minutes and last for 35-40 seconds. The waters have into two phases:
not burst yet. The fetus is in longitudinal position. 1. Latent Phase:
Fetal heartbeats are 136/min., clear and rhytmic. 1. Mild contractions that occur every 5-20 minutes.
2. The cervix begins to soften and gradually dilate up to
Major segment of the fetal head is engaged to the
around 6 cm.
pelvic inlet. Vaginal examination shows smooth
3. This phase can last for several hours and is usually
cervix of 6cm, amniotic sac is intact, sagittal suture is the longest phase of labor.
in the left oblique diameter, occipital fontanel is on 2. Active Phase:
the right near the symphysis pubis. What stage of the 1. Cervix dilates more rapidly, up to 10 cm.
labor is it? 2. Contractions become stronger, more frequent, and
A. Active phase of the first stage of normal labor more painful, occurring every 2-3 minutes and lasting
B. Latent phase of the first stage of normal labor up to 60 seconds.
C. The second stage of normal labor 3. This phase typically lasts between 3-8 hours.
D. Precursors of childbirth
E. Preliminary stage
88. After the pregnant woman’s water broke, it was noted
that they are significantly contaminated with meconium. Meconium Aspiration Syndrome (MAS)
Upon birth, the baby is not breathing, remains inert, the • What is it is? respiratory distress occurs when
skin is cyanotic and covered in meconium, heart rate is a newborn inhales meconium-stained amniotic
98/min. What resuscitation measures should be taken after fluid. Meconium is the early stool passed by a
the baby is born? the newborn.
A. Tactile stimulation of the newborn • Findings:
B. Give adrenaline intravenously • Respiratory distress: Meconium in the
C. Artificial pulmonary ventilation with a mask and airways causes airway obstruction and
Ambubag chemical irritation of the lungs.
D. Direct laryngoscopy, intubation, sanation of the trachea • Cyanosis labored breathing, grunting
E. Sanation of the upper respiratory tracts with a rubber sounds, wheezing or crackling sounds in
Balloon the chest, and poor feeding.
• Tretment:
• Immediate and appropriate respiratory
support to ensure adequate oxygenation
• In case of obstruction: Direct
laryngoscopy and intubation.
• Clearing the airways by suctioning
(sanation).
89. During a regular examination, an 8-year-old girl with
type 1 diabetes mellitus presents with a swelling on the
Objective
• What is it?
anterior surface of her hip. The swelling is 3 cm in
diameter, dense, painless on palpation. The skin over this • Hypertrophic lipodystrophy (Lipoatrophy)
is benign tumor like swelling of fatty tissue
formation has normal color and temperature. Localization
of the swelling matches the place where the girl usually at the injection site secondary to lipogenic
receives her insulin injections. What is the most likely effect of insulin
• Occurs commonly in children with DM-
cause of this clinical presentation?
A. Development of atrophic lipodystrophy
type1.
• Was common before discovery of
B. Development of hypertrophic lipodystrophy
C. Formation of post-injection abscess
recombinant human insulin
D. Allergic response
E. Formation of post-injection infiltration
90. A district doctor has diagnosed one of his patients with Documents
dysentery. What accounting document reflects this type of 1. Prevalence → prophylactic examinations
morbidity? register.
A. Urgent report 2. Dysentery (infectious) → Urgent report.
B. Statistical report 3. To assess the death of child during first month
→ Child development history
C. Report on a major non-epidemic disease
D. Certificate of temporary disability
E. Control card of a patient registered for regular check-
ups
Chief physician of a polyclinic encharged a district doctor with Documents
a task to determine the pathological prevalence of disease N in 1. Prevalence → prophylactic examinations
his district. What document allows to estimate the disease register.
prevalence in the population of amedical district? 2. Dysentery (infectious) → Urgent report.
A. Prophylactic examinations register 3. To assess the death of child during first month
B. Statistic coupons (+) → Child development history
C. Statistic coupons (-)
D. Statistic coupons (+) and (-)
E. Vouchers for medical appointments

In a rural health care area there is a case of child dying during


the first month of life. To analyse this situation, among other
measures, an expert assessment of medical records is
performed. What medical document should be considered first?
A. Child development history
B. Infant development history
C. Vaccination card
D. Outpatient medical record
E. Child’s medical record
Related
There were registered 500 cases of urolithiasis per 10000 Statistical concepts
inhabitants. What kind of statistical indices is presented? • Prevalence is how many individuals in a
A. Prevalence rate population have a disease over a specific
B. Correlation coefficient period of time. For example, number of people
C. Index of visualization infected with Covid 19 today is 2 million.
D. Incidence rate • Incidence is how many new cases of disease
E. Index of compliance occur in a population over a specific period of
time. For example, number of new cases of
Estimation of community health level involved analysis of a Covid 19 today is 1000.
report on diseases registered among the population of district • Morbidity: This refers to the overall burden of
under charge (reporting form 12). What index is calculated disease in a population, including both the
based on this report? prevalence of existing cases and the incidence
A. Prevalence of new cases.
B. Index of pathological affection • Mortality: This refers to the number of deaths
C. Index of morbidity with temporary disability that occur in a population as a result of a
D. Index of hospitalized morbidity particular disease or condition.
E. Index of basic non-epidemic morbidity
91. A 48-year-old woman has been hospitalized due to
Objective
development of tachysystolic atrial fibrillation. She has lost 5 kg
Tachycardia and loss of weigh are typical findings
of body weight within 2 months. On palpation there is a node in
in patients with hyperthyroid disorders like toxic
the left lobe of the thyroid gland. What pathology resulted in the nodular goiter
development of this condition?
A. Toxic nodular goiter
B. Aterosclerotic cardiosclerosis Objective
C. Chronic thyroiditis Toxic nodular goiter
D. Nontoxic nodular goiter • Focal patches of hyperfunctioning follicular
E. Autoimmune thyroiditis cells distended with colloid working
independently of TSH.
• Due to TSH receptor mutations in 60% of
cases.
• ↑ release of T3 and T4. Hot nodules.
• ‫اﻟﺜﺎﯾﺮوﯾﺪ ﺷﻐﻠﮫ ﻣﻦ دﻣﺎﻏﮭﺎ‬
Thyroxin / triiodothyronine T3 T4 " #

1. From: follicular cells of thyroid gland


2. Contains: iodine
3. Function: regulate Basal metabolic rate. (‫)اﻟﺣرق‬
4. How? By process of disjunction of oxidation and
oxidative phosphorylation
5. Hyperthyroidism:
• Increase basal metabolic rate (‫= )اﻟﺣرق‬
1. Loss of weight.
2. Hyperthermia.
3. Tachycardia
4. Sweating.
5. Tremor ‫اﻟرﺟﻔﺔ‬
• Exophthalmia ‫ﺟﺣوظ اﻟﻌﯾن‬: by increasing
Glycosaminoglycans → water accumulation in the
eye.
6. Hyp0rthyroidism:
• Decrease basal metabolic rate (‫= )اﻟﺣرق‬
1. Weight gain.
2. Growth retardation.
3. Disproportional body build.
4. Bradycardia.
5. Physical and mental retardation.
7. Treatment: replacement therapy .
Neonatal respiratory distress syndrome
A boy was born at 32 weeks of gestation. 2 hours after the (Hyaline membrane disease)
birth he developed respiratory distress (RD). The RD
severity assessed by Silverman score was 5. The • What is it? condition that occurs primarily in
respiratory disorders progressed, respiratory failure could premature infants due to a lack of surfactant in
not be eliminated by Martin-Bouyer CPAP (continuous the lungs.
positive airway pressure). X-ray of lungs shows reticular • Pathogenesis:
and nodular pattern, air bronchogram. What is the most 1. Lung immaturity due to ex, premature
likely cause of respiratory distress syndrome? delivery specially before week 36 →
A. Hyaline membrane disease 2. Low surfactant levels →
B. Segmental atelectasis 3. Alveoli collapse and become stiff →
C. Bronchopulmonary dysplasia 4. Formation of formation of hyaline
D. Congenital pulmonary emphysema membranes “reticular and nodular
E. Edematous hemorrhagic syndrome pattern”
• symptoms: tachypnea, cyanosis, nasal flaring,
and grunting respirations.
• Treatment: administration of exogenous
Surfactant is a substance that helps to reduce surface tension in surfactant, cortisole (stimulate surfactant
the lungs and keep the alveoli (air sacs) open during breathing. production), oxygen therapy, and mechanical
ventilation.
92. A pregnant woman at 32 weeks of gestation with the Objective
risk of preterm labor undergoes a treatment to prevent fetal Fetal respiratory distress syndrome → cortisol
respiratory distress syndrome. What medicine was she (Dexamethasone).
prescribed?
A. Dexamethasone
B. Misoprostol
C. Gynipral ( hexoprenaline)
D. Oxytocin
E. Progesterone
Review Objective
A child is 1 year old. After solid food was introduced into the Patients with celiac disease should exclude glutin
diet, within the last several months the child developed loss of cotaining food from their diets such as wheat, oats,
appetite, diarrhea with large amount of feces, and occasional bread, pasta, cereals
vomiting. Body temperature remains normal. Body weight is 7
kg. The child is very pale, has leg edemas and extremely
distended abdomen. Feces analysis detects high levels of fatty Celiac disease
acids and soaps. Diagnosis of celiac disease was made and Gluten-sensitive enteropathy
gluten-free diet was prescribed. What should be excluded from • Definition: chronic autoimmune disorder that
affects the small intestine in response to the
the diet in this case?
ingestion of gluten.
A. Cereals - wheat, oats
• Pathophysiology: gluten
B. Milk and dairy products
C. Fruits
D. Animal protei
E. Easily digestible carbohydrates
93. After semolina was introduced into the diet, a1-year
old child for 2 months has been presenting with loss of Objective
• After eating smolina (contains glutin) →
appetite, irritability, loss of body mass, and loss of
previously learned skills. The feces are copious and foul • Symptoms of Celiac disease occurred →
smelling. The skin is pale and dry, the hair is brittle. The • Celiac disease
abdomen is distended, while the limbs are thin. Stool test
shows high levels of fatty acids. What is the most likely
diagnosis?
A. Celiac disease
B. Lactase deficiency
C. Functional diahhrea
D. Irritable bowel syndrome
E. Mucoviscidosis
94. In a pediatric clinic, located in a rural area, there are 9 Methemoglobinemia
children, who simultaneously fell ill. The following signs • What is it? Nitrates convert normal structure of
were detected: low physical activity, acrocyanosis of the hemoglobin Fe+2 into → methemoglobin Fe+3
nasolabial triangle and fingertips, mucosal cyanosis,
which can’t bind effectively with oxygen leading
tachycardia, dyspnea. It was determined that all the sick to hypoxemia.
children were fed with a formula that was dissolved in the
• Symptoms:
water taken from a dug well. Laboratory analysis revealed • Cyanosis (bluish discoloration of the skin,
high levels of methemoglobin in the blood of the children. mucous membranes, and fingertips),
These signs can be caused by increased content of a certain
• Low physical activity, dyspnea (difficulty
element in the water. Name this element: breathing), tachycardia (rapid heart rate),
A. Selenium
and fatigue.
B. Sulfates
• Severe cases can lead to altered mental
C. Methyl mercury
status, seizures, and even death.
D. Nitrates
• Treatment: methylene blue and vitamin C.
E. Lead
95. A 48-year-old woman has arrived to the surgical unit
with wounds in her thigh. On examination the wound
surface has dirty-gray coating with unpleasant sweet smell. Objective
Wound content resembles raspberry jelly. Skin tissues Clostridium perfringens is an anaerobic bacteria
around the wound are glossy and turgid. Palpation reveals that cause gas gangrene wounds. Symptoms may
moderate crepitation in the tissues. What microflora is the include edema, pain, gas with crepitation, foul-
smelling exudates, intense coloration of the site.
most likely to cause such inflammation?
A. Anaerobic clostridial
B. Anaerobic non-clostridial
C. Streptococci
D. Staphylococci
E. Blue pus bacillus
Clostridium Family
1. Anerobic bacteria ‫ﻻ ھواﺋﯾﺔ‬
2. Secretes Exotoxins.
3. Hate air (anerobic).
4. Kitt-Tarozzi method.
5. Neutraliztion test.
6. Spore forming.
7. Tennis racket
8. Transmission: Canned food and honey.
Clostridium botulinum:
1. Secretes botulinum exotoxin: a neurotoxin which inhibits the
release Acetylcholine =
2. No muscle contraction (flaccid paralysis ‫)ﺷﻠل ارﺗﺧﺎﺋﻲ‬.
1. No swallowing, No movement, No respiration, No eye
accommodation.
3. Note: Botox is also used in cosmetics to remove wrinkles ‫اﻟﺗﺟﻌدات‬.
Clostridium tetani:
1. Secretes tetanus exotoxin: a neurotoxin which inhibits the release
of inhibitory neurotransmitter =
1. No inhibition to the muscles =
2. Muscles spams (Spastic paralysis).
Clostridium perfringens.
1. Causes: Gas gangrene
2. Produces Homolysis in the blood.
4 days after a patient received a gunshot wound to the soft Objective
tissues of middle third of the thigh, his condition suddenly Highlighted feature are typical for Clostridium
began deteriorating. There are complaints of bursting pain in the perfringens infection(Gas gangrene) which is Anaerobic
wound; pain has been increasing during the last 12 hours.
Edema of skin and hypodermic tissue quickly grows. Body
temperature is 38,2oC, heart rate is 102/min. The wound edges
gape, are dull in color; the muscles, viable as of day before, now
protrude into the wound, look boiled, are dull in color, have
dirty-gray coating, and fall apart when held with forceps. What
infection has developed in the wound?
A. Anaerobic
B. Aerobic gram-negative
C. Putrid
D. Aerobic gram-positive
E. Diphtheria of the wound
96. A 45-year-old man underwent a cardiac surgery one week
Objective
ago. His general state has been deteriorating since then: dyspnea
Pericardial friction rub, dyspnea, chest pain with
at rest, retrosternal pain that irradiates to the neck, marked
irradiation to the neck is a typical finding in acute
weakness. Objectively his body temperature is hectic. His pericarditis.
cardiac borders are expanded, apical beat is weakened.
Auscultation detects pericardial friction rub. What is the most
likely diagnosis?
A. Acute pericarditis
B. Acute cardiac aneurysm
C. Myocardial infarction
D. Acute myogenic dilatation of the heart
E. Pulmonary embolism
Diabetics mellitus
97. A 23-year-old woman with type 1 diabetes 1. If glucose acumulates in eyes will cause retinal disorder → glucoma and
mellitus during the 2nd week of community- Microangiopathy → blindness.
2. Diabetic patients can’t use glucose → Switch into ketongensis→ Increased keto
acquired pneumonia developed nausea and
acids = Kussmaul’s respiration.
vomiting. In the evening she has lost her 3. Polyuria.
consciousness and was hospitalized. Objectively, 4. Type 1 diabetes: beta cells of pancreas don’t secrete insulin.
the patient’s skin is pale and dry. Her respiration is 5. Has incomplete oxidation
6. Ketoacidic coma: increased ketone bodies → Aceton smell → Metabolic acidosis
long, the tongue is dry, with brown deposit. Her 7. Hyperglycaemic / hyperosmolar coma: Glucose ~ 20 micromole/l
heart rate is 129/min, blood pressure is 8. Deficiency of insulin → proteolysis → aminoacidemia
85/50mmHg. Palpation of the patient’s abdomen 9. Renal diabetes: increased glucose levels in urine due to disturbance in glucose
reabsorption from proximal convoluted tubules.
provokes no response. the liver is +3 cm. Acetone
Tests:
test is markedly positive, blood glucose is 26 1. Blood sugar: Normal Glucose: 3.3 – 5.5. Diabetes: > 12.
mmol/l. Make the provisional diagnosis: 2. Glycated haemoglobin: the test use to detect history of diabetics (≥ 6.5).
A. Hyperosmolar coma 3. Glucose tolerance: is measured after fasting glucose test.
Treatment:
B. Infectious toxic shock 1. Insulin (replacment therapy).
C. Hepatic coma 1. Use:
D. Lacticaemic coma 1. Type 1 diabetes mellitus.
2. Inhibit the process of ketosis
E. Ketoacidotic coma
3. Note: Rapid-acting insulin is used during coma.
2. Side effect: Hypoglycemia = coma (Carbohydrate starvation).
3. Antidote: Glucose or adrenaline.
Three ketone bodies from Acetyl-CoA : 2. Glibenclamide:
1. Acetone 1. Use: type 2
2. Acetoacetic acid 2. Sulphonyl urease derivate
3. Stimulates generation of insulin from beta cells.
3. 3-hydroxybutyric acid 3. Metformin:
• Use: type 2 (drug of choice).
An 8-year-old child with a 3-year-long history of diabetes was
hospitalized in hyperglycemic coma. Specify the initial dose of
insulin to be administered:
A. 0,1-0,2 U/kg of body weight per hour
Objective
Dosage of insulin in hyperglycemia coma is: 0,1-
B. 0,05 U/kg of body weight per hour
0,2 U/kg of body weight per hour
C. 0,2-0,3 U/kg of body weight per hour
D. 0,3-0,4 U/kg of body weight per hour
E. 0,4-0,5 U/kg of body weight per hour
98. A 54-year-old woman complains of fogged vision in Objective
her right eye, rainbow circles in her vision, headache, and Glaucoma
nausea. Within the last month she twice experienced a • Is a damage in the optic nerve due to increased
similar condition, but back the all the signs eventually intraocular pressure in the eye.
disappeared and her sight was restored. Currently, all the • Normal intraocular pressure is10 to 21 mmHg
signs have beed persisting for over 2 days. Objectively, the • Pathophysiology: ↑ aqueous humor = ↑
patient has eyelid edema, congestive injection of the intraocular pressure ↑ = damage optic nerve =
eyeball, corneal opacity, shallow anterior chamber of the peripheral vision loss (peripheral retina is more
eye, and dilated pupil that is unresponsive to the light. Her susceptible to damage) = central vision loss
intraocular pressure is 48mmHg. Make the diagnosis: • Types: open angle (chronic) and closed angle
A. Cyclitis (acute)
B. Iridocyclitis
C. Keratitis
D. Iritis
E. Glaucoma
Review
Objective
Glaucoma
A 59-year-old man complains of pain in his left eye and left side
• Is a damage in the optic nerve due to increased
of his head, significant vision impairment of the left eye,
intraocular pressure in the eye.
nausea, and vomiting. Visual acuity of the right eye is 1.0. • Normal intraocular pressure is10 to 21 mmHg
Visual acuity of the left eye is 0.03, attempts at correction bring • Pathophysiology: ↑ aqueous humor = ↑
no improvement. Right eye intraocular pressure - 21 mm Hg, intraocular pressure ↑ = damage optic nerve =
left eye intraocular pressure 65 mm Hg. Congestive injection is peripheral vision loss (peripheral retina is more
observed on the sclera of the left eye. The cornea is thick and susceptible to damage) = central vision loss
swollen. The anterior chamber is shallow, moist, and clear. The • Types: open angle (chronic) and closed angle
pupil is dilated and unresponsive to the light, the fundus of the (acute)
eye is not visible. What is the most likely diagnosis?
A. Acute attack of glaucoma of the left eye
B. Acute iridocyclitis of the left eye
C. Stage II intraocular tumor of the left eye
D. Endophthalmitis of the left eye
E. Panophthalmitis of the left eye
A man diagnosed with closed-angle glaucoma, grade IIa, of the Glaucoma Drugs
right eye is registered for regular medical check-ups. In the • M3 agonists
evening an acute glaucoma attack occurred in his right eye; an • Contract ciliary muscle
ambulance was called. What emergency aid would be optimal • Carbachol, pilocarpine
in this case? • α2 agonists
A. Pilocarpine, Diacarb (Acetazolamide), lytic mixture • Block ciliary epithelium from releasing
B. Atropine eye drops aqueous
C. Antibiotic eye drops, broad-spectrum • Apraclonidine
D. Sulfacetamide sodium eye drops • Β blockers
E. Dexamethasone eye drops • Block ciliary epithelium from releasing
aqueous.
• Timolol, betaxolol, carteolol
• Prostaglandin analogues
• Vasodilate the Canals of Schlemm =
increase outflow
• Bimatoprost
• Carbonic anhydrase inhibitors:
• Decrease synthesis of aqueous
• Acetazolamide
Review
Objective
A parturient woman is 30 years old, stage I of the labor is
Signs of fetal distress → urgent cesarean delivery
ongoing. The fetus is in the cephalic presentation. Auscultation • ↓ Heart rate; normal is 120-160 bpm
of the fetal heart sounds detects bradycardia. Evaluation of • Lack of variability in the fetal heart rate
cardiotocogram yielded the following data: decrease of basal pattern. A normal fetal heart rate pattern
heart rate down to 90/min., variability - monotonous (2 and should have moderate variability, meaning that
less); late decelerations with amplitude of 50/min. Make the there should be fluctuations in the heart rate of
diagnosis and choose the obstetrical tactics necessary in this at least 6-25 beats per minute. monotonous
case: refers to a lack of variability in the fetal heart
A. Fetal distress. Urgent cesarean section delivery rate pattern
B. Fetal distress. Vacuum extraction delivery • Late decelerations: a type of abnormal fetal
C. Normal condition of the fetus. Vaginal birth heart rate pattern that occur in response to
D. Fetal distress. Stimulation of uterine contractions uterine contractions. They are characterized by
a slow and gradual decrease in the fetal heart
E. Fetal distress. Forceps delivery
rate that begins after the onset of the
contraction and returns to baseline after the
contraction ends. Late decelerations may
indicate that the fetus is not receiving enough
oxygen during contractions and can be a sign
of fetal distress.
A 24-year-old pregnant woman on her 37th week of pregnancy Biophysical profile (BPP)
has been brought to the maternity obstetric service with is a prenatal test used to evaluate the health of the fetus
complaints of weak fetal movements. Fetal heartbeats are by ultrasound. The test evaluates five fetal parameters:
95/min. On vaginal examination the uterine cervix is tilted 1. Fetal heart rate
backwards, 2 cm long, external orifice allows inserting a 2. Fetal breathing movements
fingertip. Biophysical profile of the fetus equals 4 points. What 3. Fetal movements
tactics of pregnancy management should be chosen? 4. Amniotic fluid volume
A. Urgent delivery via a cesarean section 5. Fetal muscle tone
B. Treatment of placental dysfunction and repeated analysis of • Each parameter is given a score of 0 or 2 points,
the fetal biophysical profile on the next day depending on whether it is present or absent. The
C. Doppler measurement of blood velocity in the umbilical maximum score is 10 points.
artery • A normal score > 8 while a score 4 or below may
indicate fetal distress syndrome and urgent delivery
D. Urgent preparation of the uterine cervix for delivery
might be required.
E. Treatment of fetal distress; if ineffective, then elective
cesarean section on the next day
99. A pregnant woman of 34 weeks of gestation underwent
Objective
dopperometry of umbilical circulation, which revealed a
Following symptoms suggests that the best
reverse diastolic component. Objectively, the height of the
management in this case is urgent cesarean section.
uterus is 27 cm above the pubic bone, head of the fetus is
1. Reverse diastolic component in the umbilical
mobile and located above the entrance to the lesser pelvis.
circulation→ compromised blood flow to the
Fetal heartbeat is 136/min. Vaginal examination shows that
fetus → inadequate oxygen and nutrients to
the uterine cervix is closed, its length is 3 cm. What tactics
support the fetus → fetal distress syndrome.
should the obstetrician choose?
2. Uterine cervix is closed → cesarean section
A. Repeated dopplerometry next day
B. Urgent cesarean section
C. Labor induction with oxytocin
D. Fetal biophysical profile
E. Ultrasound photometry of the fetus
100. A 10-year-old boy came to the polyclinic with Objective
complaints of stuffy nose. It is known that these signs occur Presentation → allergy.
in the child periodically (in spring and autumn). He has a
history of atopic dermatitis. The father of the child had
bronchial asthma. Objectively, the boy’s face is pale and
slightly swollen. Respirations are 22/min. Auscultation
detects vesicular respiration over the lungs. Rhinoscopy
shows swollen and pale nasal mucosa. What disease can be
suspected?
A. Allergic rhinitis
B. Recurrent respiratory disease
C. Acute rhinitis
D. Acute adenoiditis
E. Acute maxillary sinusitis
172. An 8-year-old boy has a 2-year history of blotchy itchy
rash appearing after eating citrus fruit. The first eruption Atopic dermatitis
occurred at the age of 6 months after the introduction of • What is it? Atopic dermatitis (eczema) is a chronic
juices to the baby’s diet. Father has a history of bronchial inflammatory skin condition that is characterized by
asthma, mother - that of allergic rhinitis. What is the most dry, itchy, and scaly patches on the skin.
likely diagnosis? • Cuase: combination of genetic (family history),
A. Atopic dermatitis environmental, and immunological factors.
B. Psoriasis • Who? Commonly affects children.
C. Pityriasis Rosea • Symptoms: tend to be chronic and often recur, although
D. Urticaria they may vary in severity over time.
E. Quincke’s edema • Who? avoiding triggers, using moisturizers and topical
steroids,

Urticaria (hives)
• What is it? Acute skin condition that is characterized
by raised, red, and itchy wheals or bumps on the skin.
• Cuase: an allergic reaction to an allergen, such as food,
medication, or insect bites, but it can also be triggered
by non-allergic factors such as stress, heat, or exercise.
• Who? Urticaria can occur at any age.
• Treatment: avoiding triggers, antihistamines, and, in
severe cases, oral corticosteroids.
Atopic dermatitis
• What is it? Atopic dermatitis (eczema) is a chronic
inflammatory skin condition that is characterized by
dry, itchy, and scaly patches on the skin.
• Cuase: combination of genetic (family history),
environmental, and immunological factors.
• Who? Commonly affects children.
• Symptoms: tend to be chronic and often recur, although
they may vary in severity over time.
• Who? avoiding triggers, using moisturizers and topical
steroids,

Urticaria (hives)
• What is it? Acute skin condition that is characterized
by raised, red, and itchy wheals or bumps on the skin.
• Cuase: an allergic reaction to an allergen, such as food,
medication, or insect bites, but it can also be triggered
by non-allergic factors such as stress, heat, or exercise.
• Who? Urticaria can occur at any age.
• Treatment: avoiding triggers, antihistamines, and, in
severe cases, oral corticosteroids.
Objective
Acute urticaria is a skin allergic reaction that can
occur to certain type of foods like shrimps in this
case.

After eating shrimps, a 25-year-old man suddenly developed


skin itching, some areas of his skin became hyperemic or
erupted into vesicles. Make the diagnosis:
A. Acute urticaria
B. Hemorrhagic vasculitis (Henoch-Schonlein
C. purpura)
D. Urticaria pigmentosa
E. Psoriasis
F. Scabies
101. After a long drive with the window open a man developed
facial asymmetry; he cannot close his right eye, his right
nasolabial fold is smoothed out, movements of expression are
absent on the right, there is a disturbance of gustatory sensation
in the tongue on the right. No other neurological pathologies
were detected. What disease can be provisionally diagnosed in
this patient?
A. Neuropathy of the facial nerve Facial palsy
B. Neuropathy of the trigeminal nerve (Neuropathy of the facial nerve)
C. Trigeminal ganglionitis • Usually develops after HSV reactivation.
D. Neuropathy of the oculomotor nerve • Treatment: glucocorticoids +/– acyclovir. Most
E. Ischemic stroke patients gradually recover function, but
aberrant regeneration can occur.
• Other causes of peripheral facial palsy include
Lyme disease, herpes zoster (Ramsay Hunt
syndrome), sarcoidosis, tumors (eg, parotid
gland), diabetes mellitus..
Objective
Trigeminal neuralgia
• What is it? Recurrent brief episodes of intense
unilateral pain in trigeminal CN V (usually V2
A 45-year-old woman complains of intolerable paroxysmal and/or V3).
facial pain on the left with attacks that last for 1-2 minutes. • Cause? Most cases are due to compression of
Attacks are provoked by chewing. The disease onset was two CN V root by an aberrant vascular loop.
• Pain is described as electric shock–like or
months ago after the overexposure to cold. Objectively: pain at
stabbing and usually lasts seconds.
the exit points of the trigeminal nerve on the left. Touching near
• Typically triggered by light facial touch or
the wing of the nose on the left induces a pain attack with tonic facial movements (eg, chewing, talking).
spasm of the facial muscles. What is the most likely diagnosis? • Treatment: carbamazepine, oxcarbazepine.
A. Trigeminal neuralgia
B. Glossopharyngeal neuralgia
C. Temporomandibular joint arthritis
D. Facial migraine
E. Maxillary sinusitis
102. A 45-year-old man with thrombophlebitis of the deep veins Thromboembolism
in his legs suddenly after physical exertion developed sharp pain Question sequence
in his thorax on the right, dyspnea, and hemoptysis. Objectively Why do we fear from Deep vein Thrombosis (DVT) % ?
his condition is severe; he presents with acrocyanosis, 1. Thrmobosis in the lower limbs can become mobile →
shortening of pulmonary percussion sound on the right, and 2. Move throught the circulation →
3. Reach the the lungs (Pulmonary arteries) →
weakened respiration. Respiration is 30/min., blood pressure is
4. ischemia in the lungs →
110/80 mm Hg. ECG shows sinus tachycardia, heart rate is
5. Lung stop working →
120/min., electrical axis of the heart deviates to the right, SI-QIII.
6. Death &
What is the most likely diagnosis?
A. Pulmonary embolism
B. Community-acquired right-sided pneumonia
C. Cancer of the right lung
D. Right-sided exudative pleurisy
E. Spontaneous pneumothorax
Related
A 65-year-old woman, who had been suffering from deep vein
thrombophlebitis of the lower leg, suddenly died when awaiting
her appointment with the doctor. Autopsy revealed loose friable
red masses with corrugated dull surface in the main pulmonary
artery and its bifurcation. What pathologic process was
discovered by the pathologist in the pulmonary artery?
A. Fat embolism
B. Foreign body embolism
C. Thrombosis
D. Tissue embolism
E. Thromboembolism

Autopsy of the body of a man, who died during an abdominal


surgery, revealed numerous thrombi in the veins of the lesser Thromboembolism
pelvis. Clinically, thromboembolic syndrome was detected. QuesPon sequence
Where should the doctor search for the embolus? Why do we fear from Deep vein Thrombosis (DVT) % ?
A. Brain 1. Thrmobosis in the lower limbs can become mobile =
B. Pulmonary arteries 2. Move throught the circulaPon =
C. Portal vein 3. Reach the the lungs (Pulmonary arteries) =
D. Left ventricle of heart 4. ischemia in the lungs =
E. Veins of the lower extremities 5. Lung stop working =
6. Death &
103. On laboratory investigation of a pork sample there is 1 Objective
dead trichinella detected in 24 sections. This meat should • No dead trichinella is allowed.
be: • Normal in EU countries is 30 larvae per gram
A. Processed and sold through public catering network
of meat.
B. Allowed for sale with no restrictions
C. Processed for boiled sausage production
D. Handed over for technical disposal
E. Frozen until the temperature of -10 degree C is reached
in the deep layers, with subsequent exposure to cold for
15 days
Related Objective
During meat testing Trichinella was detected in diaphragm crura • No dead trichinella is allowed.
in one of the two muscular tissue samples. What tactics should a • Normal in EU countries is 30 larvae per gram
doctor choose regarding thismeat? of meat.
A. Technolgical disposal
B. Incineration
C. Boiling under 1,5 atmosphere
D. Preservation in 10% salt solution
E. Freezing under -12oC

On laboratory investigation of a pork sample there is 1 dead


trichinella detected in 24 sections. This meat should be:
A. Sent for technical disposal
B. Allowed for sale with no restrictions
C. Processed and sold through public catering network
D. Processed for boiled sausage production
E. Frozen until the temperature of -10oC is reached in the deep
layers, with subsequent exposure to cold for 15 days
104. A surgery unit received a person with an incised stab Objective
wound on the upper third of the right thigh. Examination detects Arterial blood is brighter, deeper and stronger
an incised stab wound 3.0x0.5x2.0 cm in size on the inner
surface of the upper third of the right thigh. Bright-red blood
flows from deep within the wound in a pulsing stream.
Characterize this type of bleeding:
A. Arterial
B. Venous
C. Parenchimatous
D. Capillary
E. Mixed
105. During regular examination of a 2-yearold boy, he
Objective
presents with enlarged left kidney, painless on palpation. The
Absence of one kidney and ureter + hypertrophy
right kidney was undetectable on palpation. Excretory
in the contralateral kidney → Renal Agenesis.
urography shows no contrast on the right. Cytoscopy detected
hemiatrophy of the urinary bladder trigone, the right ureteral
orifice is not detected. What pathology is it?
A. Agenesis of the right kidney
B. Dystopia of the right kidney
C. Hypoplasia of the right kidney
D. Agenesis of the right ureter
E. Ectopic right ureteral orifice
Review

A 47-year-old man is employed at the weaving workshop, has Objective


15-year-long record of service at this factory; his work Occupational deafness because of noise →
conditions are associated with high-frequency and high- Audiometry data and hygienic assessment of
intensity noise. During periodical examination he was working environment
diagnosed with occupational deafness. What are the grounds for
making such a diagnosis?
A. Audiometry data and hygienic assessment of working
environment
B. Record of service at this factory
C. Noise characteristic at this factory
D. Central nervous system examination results
E. Inner ear examination results
Review

An electro-gas welding operator working at a machine Objective


workshop performs welding and cutting of metal, which is Intense UV-radiation → Photoelectric ophthalmia
accompanied by intense UV-radiation. His welding station is
equipped with efficient mechanical ventilation. What
occupational disease is most likely to develop in an electro-gas
welding operator?
A. Photoelectric ophthalmia
B. Heatstroke
C. Vegetative-vascular dystonia
D. Chronic overheating
E. Pneumoconiosis
Review

An emergency situation at a chemical plant caused acute Authorities


occupational intoxication. A doctor who revealed the case of • Medical and preventive treatment facilities
“acute occupational disease (intoxication)” must notify the deals with sociomedical assessment of
following authority: temporary disability. Issues certificate to be
A. Sanitary and epidemiological station eligible for certain disability benefits or
B. Plant administration accommodations, such as time off work or
C. Trade union committee of the plant access to disability services.
D. Medical unit of the plant • Sociomedical expert committee: issues medical
E. Ministry of Public Health of Ukraine certificate for employee.
• Occupational diseases: Local SES
• Acute occupational disease (intoxication)”:
Sanitary and epidemiological station.
Review

A 45-year-old patient (14-year-long work record as a house Objective


painter) upon the contact with synthetic paint develops skin Allergic contact dermatitis: Type IV
reddening, edema, severe itching and oozing lesions on her hypersensitivity reaction secondary to contact
face. Symptoms disappear after the contact with this chemical allergen (eg, nickel D, poison ivy E , neomycin).
substance stops but even the smell of paint alone is enough to
make them reappear each time. Each recurrence is
characterised by increased severity of symptoms. What
provisional diagnosis can be made?
A. Professional eczema
B. Simple contact dermatitis
C. Allergic contact dermatitis
D. Urticaria
E. Toxicodermia
A 28-year-old man complains of skin rash and itching on the both of his
hands. The condition persists for 1.5 years. The exacerbation of his
condition he ascribes to the occupational contact with formaldehyde Objective
resins. Objectively the lesion foci are symmetrically localized on both Allergic contact dermatitis: Type IV
hands. Against the background of erythema with blurred margins there hypersensitivity reaction secondary to contact
are papulae, vesicles, erosions, crusts, and scales. What is the most allergen (eg, nickel D, poison ivy E , neomycin).
likely pathology?
A. Occupational eczema
B. Idiopathic eczema
C. Allergic dermatitis
D. Simple contact dermatitis
E. Erythema multiforme

A 38-year-old woman has been working as a milker for 15 years. She


made an appointment with the doctor due to development of red rashes
on her hands, predominantly in the interdigital space. The rashes are
weeping, itching, and expanding on her skin. Examination of her hands
shows her nail plates to be yellow and brittle. These presentations
aggravate during work. Make the provisional diagnosis:
A. Occupational eczema
B. Scabies
C. Pemphigus
D. Pyoderma
E. Dermatophytosis
106. A 56-year-old woman has been working as a Objective
disinfector for 19 years. She complains of general Presentation → Occupational toxic hepatitis .
weakness, nausea, bitter taste in her mouth, heavy
sensation in her right subcostal area, and rapid fatigability.
Objectively, her body temperature is 37.1° C, the sclerae
are icteric, and the liver is enlarged. Total bilirubin is 40
mcmol/L. What is the likely diagnosis?
A. Chronic cholecystitis
B. Biliary dyskinesia
C. Occupational toxic hepatitis
D. Chronic pancreatitis
E. Acute cholecystitis
A 29-year-old patient works as a motor mechanic. Anamnesis
Objective
shows frequent exposure to cold, exacerbation of chronic
• Lung cancer → low fever, cough, hemoptysis,
bronchitis attended by cough with relativly small amount of
bronchial obstruction, wheezing, pneumonic
mucopurulent sputum, subfebrility, sometimes joined by
“coin” lesion on CXR.
hemoptysis and pain in the right side of chest. Breathing is • Highlighted location → Peripheral
vesicular. X-ray shows shadows and sharp decrease in size of
the lower lobe distinctly visible on the X-ray image as a streak
2-3 cm wide situated at the angle from lung root to the frontal
costodiaphragmatic recess. The most likely diagnosis is:
A. Peripheral lung cancer
B. Bronchiectasis
C. Pneumonia
D. Middle lobe syndrome
E. Interlobular pleurisy
A 64-year-old man complains of cough with expectoration Objective
consisting of blood-streaked mucus, dyspnea, low grade fever • Lung cancer: low fever, cough, hemoptysis,
and general fatigue. He has been presenting with these bronchial obstruction, wheezing, pneumonic
symptoms for 3 months. He has been smoking since early “coin” lesion on CXR.
adolescence. Objectively to is 37.4oC, respirations are 26/min., • Smokeing → Central lung cancer (most likely)
pulse is 82/min., rhythmic, blood pressure is 130/85 mm Hg.
The right side of the thorax lags behind in the respiratory
process, dull percussion sound and acute decrease of breathing
activity are observed there. X-ray shows homogeneous shadow
of the lung field on the right with mediastinum displacement
towards the affected side. What is the most likely diagnosis?
A. Central lung cancer
B. Exudative pleuritis
C. Pleuropneumonia
D. Pulmonary tuberculosis
E. Multiple bronchiectasis
107. A 57-year-old man, a miner, complaints of a pain in
his chest, dyspnea on physical exertion, excessive
sweating, constant sub febrile temperature, and cough that
produces blood- steaked sputum. He has been smoking for
approximately 40 years (2 packs a day) and frequently has”
pneumonias”. Survey chest X-ray shows a triangular
shadow in the middle lobe of the right lung. One of the
apices of the shadow points to the lung root. Cardiac and
mediastinal shadows are displaced toward the affected area.
Make the provisional diagnosis:
A. Right-sided pleuropneumonia
B. Pneumoconiosis
C. Tuberculosis of the right lung
D. Cancer of the right lung
E. Chronic bronchitis
Review
A 30-year-old patient was hospitalized in
an intensive care unit with a diagnosis of
multiple bee stings. Skin is pale and
covered with cold sweat. Pulse can be
palpated only at the carotid arteries and is
110/min.; breathing rate is 24/min.,
rhytmical, weakened. What drug must be
administered immediately?
A. Epinephrine hydrochloride
B. Prednisolone
C. Norepinephrine hydrochloride
D. Dopamine
E. Tavegyl (Clemastine)
108. A 43-year-old man complains of facial
edema, dyspnea, and difficult swallowing
that appeared 3 hours after he was stung by
an insect. Objectively, the skin of his
eyelids, cheeks and lips is acutely
hyperemic and edematous. What medicines
should he be prescribed for emergency aid?
A. Nonsteroidal anti-inflammatory drugs
B. Diuretics
C. Muscle relaxants
D. Antihistamines
E. Glucocorticoids (Glucocorticosteroids)
Review Microsporia – tinea capitis
Mother of a 5-year-old child noticed on the head of her child a • Microsporia, also known as ringworm of the scalp or
round” bald” spot 3 cm in diameter. All the hairs in the focus are tinea capitis, is a fungal infection of the scalp and
broken off at the length of 5-6 mm. The day before the child hair. It is caused by a group of fungi known as
was petting a stray cat. Make the diagnosis: dermatophytes, which can infect the skin, nails, and
A. Microsporia hair.
• Microsporia is more commonly found in children,
B. Superficial trichophytosis
especially those between the ages of 3 and 7. The
C. Deep trichophytosis
infection is usually transmitted through direct contact
D. Psoriasis with an infected person or animal, or through contact
E. Alopecia areata with contaminated objects such as combs, brushes,
hats, or towels.
• Symptoms of microsporia may include circular, scaly
patches on the scalp, broken hair shafts, and itching.
The affected areas may also be red and inflamed. In
some cases, there may be pus-filled sores or swollen
lymph nodes.
• Treatment for microsporia usually involves
antifungal medications, such as griseofulvin,
terbinafine, or itraconazole.
A 19-year-old girl complains of moderate, itching and hair Microsporia – tinea capitis
loss on her head. Objectively, on the skin of her occipital • Microsporia, also known as ringworm of the scalp or
region there is a single round erythematous focus 3 cm in tinea capitis, is a fungal infection of the scalp and
diameter with clear margins. Asbestos- like scales can be hair. It is caused by a group of fungi known as
observed on the surface of the lesion. The hair in the focus dermatophytes, which can infect the skin, nails, and
of the lesion is broken off at the length of 6-8 cm. What is hair.
the most likely diagnosis? • Microsporia is more commonly found in children,
A. Psoriasis especially those between the ages of 3 and 7. The
B. Seborrhea infection is usually transmitted through direct contact
C. Microsporia with an infected person or animal, or through contact
D. Scabies with contaminated objects such as combs, brushes,
E. Trichophytosis hats, or towels.
• Symptoms of microsporia may include circular, scaly
patches on the scalp, broken hair shafts, and itching.
The affected areas may also be red and inflamed. In
some cases, there may be pus-filled sores or swollen
lymph nodes.
• Treatment for microsporia usually involves
antifungal medications, such as griseofulvin,
terbinafine, or itraconazole.
110. A 26-year-old man complains of chills, rhinitis, dry cough, Rubella ‫اﻟﺤﺼﺒﺔ اﻷﻟﻤﺎﻧﯿﺔ‬
and fever up to 38oC. Examination shows him to be in a • Rubella, also known as German measles, is a viral
moderately severe condition; there are small pale pink non- infection that is generally mild in children but can
merging spots on the skin of his back, abdomen, and be more serious in adults, especially in pregnant
extremities. Palpation reveals enlarged occipital and axillary women. The symptoms of rubella in adults may
lymph nodes. No information about vaccination history could be include:
obtained. What is the likely etiology of this disease? 1. Rash: A rash that begins on the face and then
spreads to the rest of the body is a common
A. Rubella virus
symptom of rubella
B. Epstein-Barr virus
2. Fever: Adults with rubella may develop a
C. Streptococcus
low-grade fever that can last for up to a
D. Mumps virus week.
E. Neisseria meningitis 3. Joint pain: Joint pain is a common symptom
of rubella in adults
4. Headache
5. Sore throat and cough: Adults with rubella
may also experience a sore throat or cough.
6. Swollen glands: Swollen glands, especially
behind the ears and at the base of the skull.
Review
Among first-year schoolchildren there was a case of
measles registered. A 7-year-old boy from the same
group was not vaccinated against measles due to refusal
of his parents. His clinical history has no cases of
measles in the past and is not contraindicatory to
immunobiological agents. Choose the most rational
tactics of measles prevention in this schoolboy:
A. Measles-Mumps-Rubella vaccine
B. Isolation for 20 days
C. Antiviral agents
D. Antibiotics
E. Immunomodulators
111. A 3-year-old child has been delivered to a hospital
with complaints of pain in the legs, fever, loss of
appetite. Objectively: pale skin and mucosa,
hemorrhagic rash. Lymph nodes are enlarged, painless,
dense and elastic, not matted together. Bones, joints,
and abdomen are painful. The liver and spleen are
enlarged. Hemogram: Hb- 88 g/l, color index - 1,3,
platelets - 80 · 109/l, leukocytes - 25,8 · 109/l,
lymphoblasts - 70%, ESR- 52 mm/hour. Make the
provisional diagnosis:
111. A 3-year-old child has been delivered to a hospital
with complaints of pain in the legs, fever, loss of
appetite. Objectively: pale skin and mucosa,
hemorrhagic rash. Lymph nodes are enlarged, painless,
dense and elastic, not matted together. Bones, joints,
and abdomen are painful. The liver and spleen are
enlarged. Hemogram: Hb- 88 g/l, color index - 1,3,
platelets - 80 · 109/l, leukocytes - 25,8 · 109/l,
lymphoblasts - 70%, ESR- 52 mm/hour. Make the
provisional diagnosis:
A. Acute leukemia
B. Thrombocytopenic purpura
C. Acute rheumatic fever
D. Infectious mononucleosis
E. Hemorrhagic vasculitis (Henoch-Schonlein
purpura)
112. A 13-year-old girl for a month has been complaining of Objective
fatigability, dull pain in her right subcostal area, abdominal Hypokinetic biliary dyskinesia (HBD) is a
distension, and constipations. Abdominal palpation reveals condition in which the muscles in the bile duct do
positive Kehr, Murphy, and Ortner signs, while Desjardins and not contract properly, leading to a backup of bile
Mayo-Robson points are painless. Total bilirubin is 14.7 in the gallbladder. Bile is a fluid that is produced
mcmol/L, predominantly indirect, ALT- 20 U/L, AST- 40 U/L, in the liver and stored in the gallbladder. It is
amylase - 6.3 mmol/L. Echocholecystography shows practically released into the small intestine to aid in the
digestion of fats.
no contraction of the gallbladder. Make the provisional
diagnosis:
A. Hypokinetic biliary dyskinesia
B. Hyperkinetic biliary dyskinesia
C. Chronic pancreatitis
D. Acute pancreatitis
E. Chronic hepatitis
113. A 20-year-old woman complains of headaches, Objective
vertigo, tearfulness, vomiting, pain in the area of the heart, • Premenstrual syndrome (PMS) refers to a
and tachycardia. The signs appear 6-7 days before collection of physical and emotional
menstruation and disappear in the first days of symptoms that occur in some women in the
menstruation. Make the diagnosis days or weeks leading up to their menstrual
A. Diencephalic syndrome period.
B. Algomenorrhea • PMS symptoms may include mood changes,
C. Metabolic craniopathy irritability, bloating, breast tenderness,
D. Stein-leventhal syndrome headaches, fatigue, and food cravings.
E. Premenstrual syndrome
A 29-year-old woman came to a gynecologist with complaints Objective
of irritability, tearfulness, headache, nausea, occasional • Premenstrual syndrome (PMS) refers to a
vomiting, pain in the heart area, tachycardia attacks, memory collection of physical and emotional
impairment, meteorism. These signs appear 6 days before symptoms that occur in some women in the
menstruation and disappear the day before menstruation or days or weeks leading up to their menstrual
during its first 2 days. On vaginal examination: the uterus and period.
uterine appendages are without alterations. What diagnosis is • PMS symptoms may include mood changes,
the most likely? irritability, bloating, breast tenderness,
A. Premenstrual syndrome headaches, fatigue, and food cravings.
B. Algodismenorrhea
C. Ovarian apoplexy
D. Genital endometriosis
E. Neurosis
A 27-year-old patient complains of irritability, tearfulness,
depression, and sometimes aggressiveness, headache, nausea,
vomiting, swelling of the mammary glands. The mentioned
problems arise 5-6 days before menstruation and gradually
progress until menstruation, 3 days after it the problems
disappear. What is the most likely diagnosis?
A. Premenstrual syndrome
B. Premature pathological climacterium
C. Secondary psychogenic amenorrhea
D. Preclimacterium syndrome
E. Algomenorrhea

An 18-year-old girl complains of breast pain and engorgement,


headaches, irritability, swelling of the lower extremities. These
symptoms have been observed since menarche and occur 3-4
days before the regular menstruation. Gynecological
examination revealed no pathology. Make a diagnosis:
A. Premenstrual syndrome
B. Neurasthenia
C. Renal disease
D. Mastopathy
E. Cardiovascular disorder
Objective
• Climacteric syndrome is an old name for
Review (Related)
menopause Age > 45 ‫ﺳن اﻟﯾﺄس ﻋﻧد اﻟﻧﺳﺎء‬.
A 48-year-old woman complains of disturbed menstrual cycle:
• Findings:
her periods last for 7-9 days and are excessively profuse • Hot flashes (very common).
throughout the last half-year. She notes occasional hot flashes in • Disturbed menstrual cycle.
her head, insomnia, irritability, and headaches. Her skin is of • Mood changes, including irritability,
normal color. Blood pressure - 150/90 mm Hg, pulse - 90/min., depression, and anxiety
rhythmic. The abdomen is soft and painless. Bimanual • Insomnia or sleep disturbances
examination shows no uterine enlargement, the appendages • Decreased libido or sexual desire
cannot be detected. The vaginal fornices are free. What is the • Urinary incontinence or increased
most likely diagnosis? urgency to urinate
A. Climacteric syndrome • Joint and muscle pain
B. Premenstrual syndrome • Weight gain or difficulty losing weight
C. Adrenogenital syndrome • Memory problems or difficulty
D. Stein-Leventhal syndrome (polycystic ovary syndrome) concentrating
E. Uterine myoma
114. What modern organizational method can provide the Objective
patients in the remote settlements with timely access to Telemedicine is the remote delivery of healthcare
quality medical aid and such medical services as services using technology.
consulting, diagnostics, and treatment, especially in the
situations when time and distance are crucial?
A. Telemedicine
B. Field medical services
C. Mobile communication
D. Ambulance services
E. Air medical services
115. 3 hours after a trauma, a young man developed bradycardia Objective
of 46/min., anisocoria D>S, hemi-hyperreflexia S>D, • Neurological symptoms after trauma →
hemihypesthesia on the left, and a convulsive disorder. The suspected brain injury → CT scan or MRI
character of this process needs to be clarified. What method of • After a head trauma, a CT scan can detect
examination will be the most accurate for this purpose? bleeding or swelling in the brain, skull
A. Brain CT fractures, and other types of brain injuries.
B. Skull X-ray • Skull X-ray will mainly limited to skull
fracture only.
C. Electroencephalography
D. Echoencephalography
E. Lumbar puncture
116. A 40-year-old victim of a traffic accident sustained the Objective
following injuries: closed diaphyseal femur fracture, brain Priority for the most serious injuries
concussion, multiple rib fractures, hemopneumothorax,
degloving shin injuries. What injuries require the most urgent
attention?
A. Multiple rib fractures, hemopneumothorax
B. Closed diaphyseal femur fracture
C. Brain concussion
D. Degloving shin injuries
E. All injuries are equivalent
117. After eating shrimps, a 25-year-old man suddenly Objective
developed skin itching, some areas of his skin became Acute urticaria is a skin allergic reaction that can
hyperemic or erupted into vesicles. Make the diagnosis: occur to certain type of foods like shrimps in this
A. Acute urticaria case.
B. Hemorrhagic vasculitis (Henoch-Schonlein purpura)
C. Urticaria pigmentosa
D. Psoriasis
E. Scabies
An 8-year-old boy has a 2-year history of blotchy itchy rash Atopic dermatitis
appearing after eating citrus fruit. The first eruption occurred • What is it? Atopic dermatitis (eczema) is a chronic
at the age of 6 months after the introduction of juices to the inflammatory skin condition that is characterized by
baby’s diet. Father has a history of bronchial asthma, mother - dry, itchy, and scaly patches on the skin.
that of allergic rhinitis. What is the most likely diagnosis? • Cuase: combination of genetic (family history),
A. Atopic dermatitis environmental, and immunological factors.
B. Psoriasis • Who? Commonly affects children.
C. Pityriasis Rosea • Symptoms: tend to be chronic and often recur, although
D. Urticaria they may vary in severity over time.
E. Quincke’s edema • Who? avoiding triggers, using moisturizers and topical
steroids,

Urticaria (hives)
• What is it? Acute skin condition that is characterized
by raised, red, and itchy wheals or bumps on the skin.
• Cuase: an allergic reaction to an allergen, such as food,
medication, or insect bites, but it can also be triggered
by non-allergic factors such as stress, heat, or exercise.
• Who? Urticaria can occur at any age.
• Treatment: avoiding triggers, antihistamines, and, in
severe cases, oral corticosteroids.
Atopic dermatitis
• What is it? Atopic dermatitis (eczema) is a chronic
inflammatory skin condition that is characterized by
dry, itchy, and scaly patches on the skin.
• Cuase: combination of genetic (family history),
environmental, and immunological factors.
• Who? Commonly affects children.
• Symptoms: tend to be chronic and often recur, although
they may vary in severity over time.
• Who? avoiding triggers, using moisturizers and topical
steroids,

Urticaria (hives)
• What is it? Acute skin condition that is characterized
by raised, red, and itchy wheals or bumps on the skin.
• Cuase: an allergic reaction to an allergen, such as food,
medication, or insect bites, but it can also be triggered
by non-allergic factors such as stress, heat, or exercise.
• Who? Urticaria can occur at any age.
• Treatment: avoiding triggers, antihistamines, and, in
severe cases, oral corticosteroids.
118. Employees work in conditions of high dust concentration. Objective
Certain chemical (silicon dioxide content) and physical • Dispersion refers to the extent to which
properties of dust aerosols contribute to the development of particles are distributed throughout the air.
occupational dust-induced diseases. What is the main physical • Aerosol (dust) particles such as silicon dioxide
property of dust aerosols? can have high dispersion levels.
A. Dispersion
B. Magnetization
C. Electric charge
D. Solubility
E. Ionization
119. A 58-year-old man complains of weakness and
tumor-like formations that appeared on the anterior
surface of his neck and in the inguinal region.
Palpation detects soft painless mobile cervical and
inguinal lymph nodes up to 2 cm in diameter. The liver
protrudes by 2 cm from the edge of the costal margin,
the lower splenic pole is at the umbilical level. In
blood: erythrocytes - 3.5 · 1012/L, Hb- 88 g/L,
leukocytes - 86 · 109/L, band neutrophils - 1%,
segmented neutrophils - 10%, lymphocytes - 85%,
eosinophils - 2%, basocytes - 0%, monocytes - 2%,
erythrocyte sedimentation rate - 15 mm/hour,
Gumprecht shadows. What is the most likely
diagnosis?
A. Chronic lymphatic leukemia
B. Lymphocytic leukemoid reaction
C. Acute leukemia
D. Chronic myeloleukemia
E. Lymphogranulomatosis
120. A 55-year-old woman came to a gynecologist with
complaints of leukorrhea and bloody discharge from the vagina
after 5 years of menopause. Anamnesis states no pregnancies. Endometrial carcinoma
Bimanual examination: the uterus and uterine appendages are (uterine corpus cancer)
without changes. During diagnostic curettage of the uterine 1. Menopausal female
cavity the physician scraped off enchephaloid matter. What is 2. Abnormal uterine bleeding
the most likely diagnosis in this case? 3. Uterine enlargement
A. Endometrial carcinoma
B. Adenomyosis
C. Subserous uterine myoma
D. Cervical carcinoma
E. Ovarian carcinoma
Endometrial carcinoma
(uterine corpus cancer)
Menopausal female, abnormal uterine bleeding,
Review uterine enlargement is typical findings in
endometrial carcinoma (uterine corpus cancer)
A 58-year-old woman came to the gynecological clinic. She
complains of bloody discharge from her genital tracts.
Menopause is 8 years. Gynecological examination: the uterus is
slightly enlarged, dense to touch, with limited mobility; the
uterine appendages cannot be detected; parametrium is free.
Fractional curettage of the uterine cavity yields a significant
amount of medullary substance in the scrape. What is the most
likely diagnosis?
A. Uterine corpus cancer
B. Adenomyosis
C. Chorioepithelioma
D. Uterine cervix cancer
E. Hormone-producing ovarian tumor
121. Children from a certain township presents with brittle
teeth, malocclusion, dental enamel erosions, and dental
pigmentation that looks like yellow-brown spots. What is
the likely cause of this presentation?
A. High levels of sulfates in water
B. High levels of nitrates
C. High levels of fluorine in water
D. Low levels of fluorine in water Normal ranges: 0.7 to 1.2 mg/L
E. Low levels of sulfates in water
Good?
• In mderate amounts: Flurine forms fluorapatite
which prevent tooth decay and cavitation of the
tooth.
Bad?
• In excess amount: cause tooth defects.
Caries morbidity rate is 89% among residents of a community.
It is determined that fluorine content in water is 0,1 mg/l. What
preventive measures should be taken?
A. Water fluorination
B. Tooth brushing
C. Fluorine inhalations
D. Sealant application
E. Introduction of more vegetables to the diet Normal ranges: 0.7 to 1.2 mg/L

Examination of a group of persons living on the same territory Good?


revealed the following common symptoms: dark-yellow • In mderate amounts: Flurine forms fluorapatite
pigmentation of the tooth enamel, diffuse osteoporosis of bone which prevent tooth decay and cavitation of the
apparatus, ossification of ligaments and joints, functional tooth.
disorders of the central nervous system. This condition may be Bad?
caused by the excessive concentration of the following • In excess amount: cause tooth defects.
microelement in food or drinking water:
A. Fluorine
B. Copper
C. Nickel
D. Iodine
E. Cesium
122. A 65-year-old man with acute anterior myocardial Objective
infarction developed an asphyxia attack. Examination The presence of asphyxia attacks and wet crackles
detected diffuse cyanosis. In the lungs there are numerous after MI is strongly suggests the complication of
heterogenous wet crackles. Heart rate is 100/min. Blood pulmonary edema.
pressure is 120/100 mm Hg. What complication occurred in
this patient?
A. Cardiogenic shock
B. Ventricular septal rupture
C. Pulmonary edema
D. Hypertensive crisis
E. Pulmonary embolism
Review
During physical exertion a man suddenly developed acute Objective
chest pain on the right and dyspnea. Objectively he • Pneumothorax is accumulation of air in
assumes forced half sitting position in the bed, presents pleural cavity.
with diffuse cyanosis, resting tachypnea of 38/min., the • Chest pain, resonance ‫ﺻدى‬, absence of
right side of the thorax is enlarged and does not respiration →Spontaneous pneumothorax.
participate in the respiratory process; percussion on the
right reveals tympanic resonance and absence of
respiration. What is the most likely diagnosis in this case?
A. Spontaneous pneumothorax
B. Hemothorax
C. Lobar pneumonia
D. Pulmonary embolism
E. Acute pleurisy
123. A 7-year-old boy after a fall from a height presents Objective
with rapid and shallow breathing and cyanotic face. The Presentation → pneumothorax.
right half of his thorax is distended and takes no part in the
respiration. Percussion detects tympanitis in the affected
area, while auscultation detects no breathing there. What
pathology is the most likely cause of this clinical
presentation? What instrumental examination would be
most informative in this case?
A. Tension cyst of the right lung. Tracheobronchoscopy
B. Right-sided tension pneumothorax. Chest X-ray
C. Right-sided hemothorax. Survey X-ray of the chest
D. Airway foreign body. Diagnostic therapeutic
bronchoscopy
E. Mediastinitis. Survey X-ray of the chest
Review Objective
A 45-year-old man diagnosed with acute pulmonary abscess • Pyopneumothorax: pus in the pleural cavity
suddenly developed sharp pain in his chest on the right and • Pneumothorax: air in the pleural cavity
dyspnea up to 30/min. Examination detects facial cyanosis and • Pleuropneumonia: inflammation of lung and
shallow rapid respirations. Auscultation reveals acutely pleura.
weakened respiration throughout the whole right lung;
percussion reveals a vesiculotympanitic (bandbox) resonance at
the lung apex and dullness in the lower lobe. What complication
developed in this patient?
A. Pyopneumothorax
B. Pleuropneumonia
C. Pneumothorax
D. Acute mediastinitis
E. Esophageal perforation
124. The director of a medical facility draws up a financial Public health management
plan for the next year. To improve the economic well-being • Fixed costs are expenses that do not vary with
of his establishment, he decided to increase the amount of the level of production or services provided.
medical services provided. How will it change the fixed They include costs such as rent, utilities, salaries
cost per unit of service? of administrative staff, and other overhead
A. Fluctuate expenses.
B. Decrease • When the amount of medical services provided is
C. No changes increased, the total fixed costs remain the same.
D. There is no correlation between these variables However, since the fixed costs are spread over a
E. Increase larger number of units of service, the fixed cost
per unit of service decreases. This is because the
fixed costs are being distributed across a larger
base of services, reducing the average cost per
unit. For example, if a medical facility incurs
$10,000 in fixed costs per month and provides
100 units of service, the fixed cost per unit would
be $100 ($10,000 divided by 100).
• If the facility then increases the amount of
services provided to 200 units, the fixed cost per
unit would decrease to $50 ($10,000 divided by
200).
125. A multigravida, labor II, 36-37 weeks of gestation, Premature detachment of placenta
has gone into labor. Her waters broke 8 hours ago, the labor
(Placental abruption)
activity continues for the last 4 hours, it is regular, with Following symptoms strongly suggests the
contractions that last 35 seconds and occur every 3 –4
premature:
minutes. The child is in the cephalic presentation, with the
1. Sudden sharp abdominal pain.
head pressed to the entrance into the lesser pelvis. The
2. Fetal heartbeat is muffled → fetal distress.
parturient woman complains of a sudden sharp abdominal
3. Presence of blood in amniotic fluid.
pain. Her pulse is 100/min, blood pressure is 110/70-
100/70 mmHg. The uterus is tense and does not relax
between the contractions. Fetal heartbeat is muffled-
100/min. The amniotic fluid is blood-colored and continues
to leak. What is the most likely diagnosis?
A. Uterine rupture
B. Cervical rupture
C. Rupture of the umbilical vessels
D. Premature detachment of the normally positioned
placenta
E. Partial placenta previa
126. A 52-year-old woman has been suffering for 2 years from
Objective
dull, occasionally exacerbating pain in her right subcostal area,
• Patient presentations is typical for gallbladder
occurring after eating high-fat foods, bitter taste in her mouth in inflammation (cholecystitis).
the morning, constipations, and flatulence. Objectively she has • Ultrasound is the most important diagnostic
excess weight, her body temperature is 36.9oC; there is a device for the diagnosis of cholecystitis.
coating on the root of her tongue; the abdomen is moderately • Why do patients with cholecystitis have pain
distended and painful in the area of gallbladder projection. What after fatty meals?
examination would be the most helpful for diagnosis making?
A. Ultrasound
B. Duodenal intubation
C. Cholecystography
D. Duodenoscopy
E. Liver scanning
A 5-year-old child had acute onset of the disease that
manifested in body temperature up to 39.5° C, marked
chills, weakness, inertness, skin pallor, and headache. 8
hours later a hemorrhagic rash developed on the skin of the
buttocks and legs. The child is sluggish, the body
temperature has dropped, blood pressure is 80/40,
respirations are 28-30/min, diuresis is decreased. Make the
provisional diagnosis:
A. thrombocytopenic purpura
B. Hemorrhagic vasculitis (Henoch-Schonlein purpura)
C. Measles
D. Meningococcemia
E. Reye syndrome

Objective
Hemorrhagic rash and hypotension → Meningococcemia.
127. A 21-year-old man was hospitalized on the 2nd day of
the disease. His general condition is severe, body Adrenal insufficiency
temperature is 39°C. On his skin there are numerous • Definition: inability of adrenal glands to
irregular-shaped hemorrhghic elements. The diagnosis of generate enough glucocorticoids (cortisol) +/−
meningococcemia was made. The next day his body mineralocorticoids (aldosterone) which can be
temperature suddenly decreased, blood pressure was treated with replacement therapy.
80/40mmHg, pulse was 120/min. Acrocyanosis was • Findings:
detected. What complication did the patient develop? • Hyperpigmentation: occurs in primary
A. Acute hemorrhage adrenal insufficiency.
B. Acute liver failure
• Low blood pressure: due to decreased
C. Acute heart failure cortisol and aldosterone.
D. Acute adrenal insufficiency • Weakness and fatigue.
E. Cerebral coma
• Muscle aches, weight loss, GI
disturbances, sugar and/or salt cravings.
• During stress, such surgery or infection →
• The body requirements for cortisol is
increased.
• The dose of cortisol should be increased.
• Remember cortisol is called stress
hormone.
A 36-year-old man has been complaining of marked weakness, Objective
low appetite, and nausea for the last year, year and a half. Low blood pressure + hyperpigmentation are
Within the last year he has lost 10 kg of body mass. The skin is typical for adrenal insufficiency.
darkened, especially on his face, neck, and arms. Skin folds and
nipples are hyperpigmented, there are pigment spots on the Adrenal insufficiency
patient’s inner thighs. Pulse is 60/min., blood pressure is 80/50 • Definition: inability of adrenal glands to
mm Hg. What is the provisional diagnosis? generate enough glucocorticoids (cortisol) +/−
A. Adrenal insufficiency mineralocorticoids (aldosterone) which can be
B. Diabetes mellitus treated with replacement therapy.
C. Chronic gastritis • Findings:
D. Hemochromatosis • Hyperpigmentation: occurs in primary
E. Cholestatic hepatitis adrenal insufficiency.
• Low blood pressure: due to decreased
cortisol and aldosterone.
• Weakness and fatigue.
• Muscle aches, weight loss, GI
disturbances, sugar and/or salt cravings.
• During stress, such surgery or infection →
• The body requirements for cortisol is
increased.
• The dose of cortisol should be increased.
• Remember cortisol is called stress
hormone.
Adrenal insufficiency
• Definition: inability of adrenal glands to
generate enough glucocorticoids (cortisol) +/−
Review mineralocorticoids (aldosterone) which can be
A 39-year-old man suffers from chronic adrenal insufficiency treated with replacement therapy.
and receives replacement glucocorticoid therapy • Findings:
(hydrocortisone - 15 mg/day). He is to undergo elective surgery • Hyperpigmentation: occurs in primary
adrenal insufficiency.
for calculous cholecystitis. What medication adjustment should
• Low blood pressure: due to decreased
be made on the day of the surgery to prevent the development of
cortisol and aldosterone.
acute adrenal insufficiency?
• Weakness and fatigue.
A. Increase the dosage by 2-3 times • Muscle aches, weight loss, GI
B. Cancel the drug for the day of the surgery disturbances, sugar and/or salt cravings.
C. Add a mineralocorticoid • During stress, such surgery or infection →
D. Add an antibiotic • The body requirements for cortisol is
E. Prescribe a large volume intravenous fluid infusion increased.
• The dose of cortisol should be increased.
• Remember cortisol is called stress
hormone.
After tonsillectomy a woman with systemic lupus
erythematosus, who has been taking prednisolone for a year, Adrenal insufficiency
developed acute weakness, nausea, vomiting, pain in the right • Definition: inability of adrenal glands to
iliac area, watery stool up to 5 times per day. Ps- 96/min., BP- generate enough glucocorticoids (cortisol) +/−
80/50 mm Hg. What preventive therapy should have been mineralocorticoids (aldosterone) which can be
administered prior to the surgery? treated with replacement therapy.
A. Increase of prednisolone dosage • Findings:
B. Prescription of desoxycorticosterone acetate • Hyperpigmentation: occurs in primary
C. Prescription of antibiotics adrenal insufficiency.
D. Administration of Ringer’s solution • Low blood pressure: due to decreased
E. Administration of 10% NaCl solution cortisol and aldosterone.
• Weakness and fatigue.
• Muscle aches, weight loss, GI
disturbances, sugar and/or salt cravings.
• During stress, such surgery or infection →
• The body requirements for cortisol is
increased.
• The dose of cortisol should be increased.
• Remember cortisol is called stress
hormone.
128. A 2.5-yearold child is ill for the second day. The onset
Objective
of the disease was associated with the temperature up to Oral rehydration is used to compensate for loss of
37.8°C, a single bout of vomiting, a watery diarrhea up to 5
volume in diarrhea and vomiting to prevent
times per day. During the second day, vomiting occurred
dehydration.
twice, body temperature is 38.0°C, the child has low
appetite, watery diarrhea continues. The treatment of the
child should start with the following:
A. Prescribe nifuroxazide
B. Prescribe polymyxin
C. Prescribe loperamide
D. Prescribe ceftriaxone
E. Prescribe oral rehydration
129. Examination detected vesicles with seropurulent Vesiculopustulosis
content on the neck, back of the head, and buttocks of an • What is it? Vesicles (small) with seropurulent
infant on the 4th day of life. The patient’s condition is content.
satisfactory, the child is active, all newborn reflexes can be • Cause: Staph aureus
fully induced, the umbilical cord is at the stage of • Who: Infants
mummification, the umbilical area is without any • Location: Head, abdomen and buttocks.
peculiarities. What disease can be suspected?
A. Epidermolysis bullosa
B. Phlegmon
C. Vesiculopustulosis
D. Miliaria
E. Neonatal pemphigus

Related 2009
On the 6th day of life a child got multiple vesicles filled with
seropurulent fluid in the region of occiput, neck and buttocks.
General condition of the child is normal. What disease should
be suspected?
A. Vesiculopustulosis
B. Impetigo neonatorum
C. Miliaria
D. Impetigo
E. Epidermolysis bullosa
Pediatric (neonate) - dermatology

A 22-day-old infant developed subcutaneous red nodes from 1.0 Objective


to 1.5 cm in size on the scalp; later the nodes suppurated. • Pseudofurunculosis is a purulent lesion of
Temperature increased up to 37.7oC, intoxication symptoms sweat glands (small) that develops when
appeared, regional lymph nodes enlarged. Complete blood staphylococci or other pathogens enter them.
count: anemia, leukocytosis, neutrocytosis, increased ESR. • Pseudofurunculosis mainly affects newborns
What diagnosis can be made? and children under one year old
• Clinical dermatology refers pseudofurunculosis
A. Pseudofurunculosis
to a group of purulent-inflammatory skin
B. Pemphigus
diseases — pyoderma. The most common
C. Vesiculopustulosis
infectious agent in pseudofurunculosis is
D. Scalp phlegmon Staphylococcus aureus.
E. –
Related
A 3-week-old infant developed large, flaccid vesicles with
Pemphigus neonatorum
purulent contents on the skin of chest and abdomen. The
• Large blisters.
vesicles rupture quickly. Make the provisional diagnosis: • Rupture.
A. Pemphigus neonatorum • Epidermal fragments.
B. Vesiculopustulosis
C. Toxic erythema
D. Pemphigus syphiliticus
E. Pseudofurunculosis

10 days after birth a newborn developed a sudden fever up to


38, 1oC. Objectively: the skin in the region of navel, abdomen
and chest is erythematous; there are multiple peasized blisters
with no infiltration at the base; single bright red moist erosions
with epidermal fragments on the periphery. What is your
provisional diagnosis?
A. Epidemic pemphigus of newborn
B. Syphilitic pemphigus
C. Streptococcal impetigo
D. Vulgar impetigo
E. Atopic dermatitis
A 53-year-old man complains of general weakness, loss of
Objective
appetite, and painful vesicles appearing on his skin. The disease
Acantholytic pemphigus
onset occurred suddenly, after hyperinsolation one week ago. • Terminology:
Examination detects isolated vesicles with wrinkled opercula • Pemphigoid: blister or bubble
and occasional painful erosions on the skin of the patient’s torso • Acantholysis: separation of skin
and limbs. Nikolsky sign is positive. What is the most likely • What is it? Autoimmune disease in which
diagnosis? immune system produces antibodies that attack
A. Acantholytic pemphigus proteins in the skin and mucous membranes,
B. Nonacantholytic pemphigus leading to the formation of blisters and
C. Duhring’s disease (dermatitis herpetiformis) erosions..
D. Herpes
E. Toxicodermia
An 8-day-old boy was delivered to the hospital on the Phlegmon of the newborn
second day after the onset of the disease. His parents
(cellulitis of the newborn)
complain of his fussiness, regurgitation, body temperature
• What is it? A bacterial neonatal infection affects
up to 38.5 C, red skin with infiltration in the lumbar area.
the subcutaneous tissue, fascia, and muscles.
His medical history has no peculiarities. The child is in the
• Cuased by: Staphylococcus aureus and
severe condition, inert, pale, suckles poorly. In the lumbar
streptococcus pyogenes
area, on the sacrum and buttocks there is a tense infiltration
• Symptoms: fever, lethargy, poor feeding, and
with hyperemic and cyanotic areas and with a soft spot 8×7
swelling, redness, infilteration and tenderness of
cm in its center. the stool is 10 times in 24 hours, with
the affected area
green and mucous admixtures. What is the most likely
diagnosis?
A. Erysipelas
B. Congenital soft-tissue tumor
C. Phlegmon of the newborn
D. Adiponecrosis
E. Hemangioma
130. A 20-year-old patient complains of severe headache,
Objective
double vision, weakness, fever, irritability. Objectively: body
• Normal Cerebrospinal Fluid:
temperature is 38.1oC, the patient is reluctant to contact,
• Glucose: 2.2-3.9 mmol/L
sensitive to stimuli. There are ptosis of the left eyelid, • Protein: < 0.4 g/L
exotropia, anisocoria S>D, pronounced meningeal syndrome. • Lymphocytes: 5-10 per mm3.
On lumbar puncture the cerebrospinal fluid flowed out under a • ↑ lymphocytes, ↑ proteins, ↓ glucose are
pressure of 300 mm Hg, the fluid is clear, slightly opalescent. typical for TB and fungal meningitis.
24 hours later there appeared fibrin film. Protein - 1.4 g/L,
lymphocytes - 600/3 per mm3, sugar - 0.3 mmol/L. What is the
provisional diagnosis?
A. Tuberculous meningitis
B. Meningococcal meningitis
C. Lymphocytic (Armstrong’s) meningitis
D. Syphilitic meningitis
E. Mumps meningitis
‫اﻟﺗﮭﺎب ﻓﻲ اﻏﺷﯾﺔ اﻟدﻣﺎغ – اﻷﺳﺑﺎب‬
An 9-year-old child was hospitalized for fever up to 39,8oC, Objective
inertness, moderate headache, vomiting. Examination revealed • Normal Cerebrospinal Fluid:
meningeal symptoms. Lumbar puncture was performed. The • Glucose: 2.2-3.9 mmol/L
obtained fluid was characterised by increased opening pressure, • Protein: < 0.4 g/L
was transparent, with the cell count of 450 cells per 1 mcL • Lymphocytes: 5-10 per mm3.
(mainly lymphocytes - 90%), glucose level of 3,6 mmol/l. What • Normal glucose → viral → Enterovirus
agent could have caused the disease in the child?
A. Enterovirus
B. Neisseria meningitidis
C. Mycobacterium tuberculosis
D. Staphylococcus aureus
E. Streptococcus pneumoniae

‫اﻟﺗﮭﺎب ﻓﻲ اﻏﺷﯾﺔ اﻟدﻣﺎغ – اﻷﺳﺑﺎب‬


A 43-year-old man, who has been abusing alcohol and suffering Objective
from pulmonary tuberculosis, in the course of two weeks • Meningitis symptoms (headache & nuchal
gradually developed general weakness, headache, diplopia, rigidity) → meningitis
vomiting. Objectively: ptosis on the left, anisocoria S>D, • Tuberculosis infection → Tuberculous
exotropia of the left eye, neck stiffness; Kernig’s and
Brudzinski’s signs are positive. In cerebrospinal fluid:
lymphocytic pleocytosis, low glucose, precipitation of
cerebrospinal fluid resulted in production of fibrin film. What is
the most likely diagnosis?
A. Tuberculous meningitis
B. Subarachnoid hemorrhage
C. Brainstem encephalitis
D. Acute myelitis
E. Basal arachnoiditis
A patient is being treated in the tuberculosis clinic.
Throughout the last 3 weeks he has been suffering from
headaches of increasing intensity. Neurological examination
detects nuchal rigidity without focal signs. Make the provisional Objective
• Meningitis symptoms (headache & nuchal
diagnosis:
rigidity) → meningitis
A. Tuberculous meningitis
• Tuberculosis infection → Tuberculous
B. Chorea minor
C. Brain tumor
D. Myelitis
E. Convexital arachnoiditis

You might also like